glossari: retocs
[apunts-acces-uib-majors-25-anys-matematiques.git] / 05-geometria.tex
blob50a153d216413fbceb696a36343e72434f7156ba
1 % SPDX-FileCopyrightText: 2023 Xisco Sebastià, Xavier Bordoy
3 % SPDX-License-Identifier: CC-BY-4.0
5 \part{Geometria analítica}
6 % Definició de colors (per gràfics)
7 \definecolor{qqqqff}{rgb}{0.,0.,1.}
8 \definecolor{cqcqcq}{rgb}{0.752941176471,0.752941176471,0.752941176471}
9 \definecolor{qqttcc}{rgb}{0.,0.2,0.8}
11 % Començ text
13 \chapter{Geometria del pla}\label{seccio:geometria-al-pla}
15 En aquest tema s'estudiaran els vectors i les rectes definits sobre un espai
16 de dues dimensions.
18 \section{Punts}
19 Aquest apartat tracta de l'estudi dels vectors i de les seves operacions a
20 l'espai de dues dimensions. Aquest espai queda representat per uns \term{eixos de coordenades}\index{eixos de coordenades}, que són dues rectes reglades entre les quals hi ha un angle recte (\autoref{fig:pla-cartesia}):
21 \begin{itemize}
22 \item L'eix horitzontal s'anomena \term{eix de les abscises}\index{eix!de les abscises} (o simplement \term{eix de les $X$}) i s'anomena amb la lletra $X$
23 \item L'eix vertical s'anomena \term{eix de les ordenades}\index{eix!de les ordenades} (o simplement \term{eix de les $Y$}) i s'anomena amb la lletra $Y$
24 \end{itemize}
25 En conjunt, els eixos formen el que s'anomena \term{Pla cartesià}\index{pla cartesià}\index{sistema de coordenades}.
27 Cada \term{punt}\index{punt!del pla} del pla queda determinat per les seves projeccions sobre cadascun dels eixos, el que s'anomenen \term{coordenades}\index{coordenades} (\autoref{fig:coordenades-punts}).
29 \begin{figure}[h!]
30 \centering
31 \begin{tikzpicture}[line cap=round,line join=round,>=triangle 45,x=1.0cm,y=1.0cm,scale=0.75]
32 \draw [color=cqcqcq, xstep=1.0cm,ystep=1.0cm] (-6.18,-3.14) grid (6.74,5.9);
33 \draw[->,color=black] (-6.18,0.) -- (6.74,0.);
34 \foreach \x in {-6,-5,-4,-3,-2,-1,1,2,3,4,5,6}
35 \draw[shift={(\x,0)},color=black] (0pt,2pt) -- (0pt,-2pt) node[below] {\small $\x$};
36 \draw[->,color=black] (0.,-3.14) -- (0.,5.9);
37 \foreach \y in {-3,-2,-1,1,2,3,4,5}
38 \draw[shift={(0,\y)},color=black] (2pt,0pt) -- (-2pt,0pt) node[left] {\small $\y$};
39 \draw[color=black] (0pt,-10pt) node[right] {\small $0$};
40 \clip(-6.18,-3.14) rectangle (6.74,5.9);
41 \end{tikzpicture}.
42 \caption{Pla cartesià}
43 \label{fig:pla-cartesia}
44 \end{figure}
46 \begin{figure}[h!]
47 \centering
48 \begin{tikzpicture}[line cap=round,line join=round,>=triangle 45,x=1.0cm,y=1.0cm,scale=0.75]
49 \draw [color=cqcqcq, xstep=1.0cm,ystep=1.0cm] (-6.18,-3.14) grid (6.74,5.9);
50 \draw[->,color=black] (-6.18,0.) -- (6.74,0.);
51 \foreach \x in {-6,-5,-4,-3,-2,-1,1,2,3,4,5,6}
52 \draw[shift={(\x,0)},color=black] (0pt,2pt) -- (0pt,-2pt) node[below] {\small $\x$};
53 \draw[->,color=black] (0.,-3.14) -- (0.,5.9);
54 \foreach \y in {-3,-2,-1,1,2,3,4,5}
55 \draw[shift={(0,\y)},color=black] (2pt,0pt) -- (-2pt,0pt) node[left] {\small $\y$};
56 \draw[color=black] (0pt,-10pt) node[right] {\small $0$};
57 \clip(-6.18,-3.14) rectangle (6.74,5.9);
58 \draw [fill=qqqqff] (2,3) circle (1.5pt);
59 \draw[color=qqqqff] (2,3) node[anchor=south] {$(2,3)$};
60 \draw [fill=qqqqff] (-3,1) circle (1.5pt);
61 \draw[color=qqqqff] (-3,1) node[anchor=south] {$(-3,1)$};
62 \draw [fill=qqqqff] (-1.5,-2.5) circle (1.5pt);
63 \draw[color=qqqqff] (-1.5,-2.5) node[anchor=east] {$(-1.5,2.5)$};
64 \draw [fill=qqqqff] (0,0) circle (1.5pt);
65 \draw[color=qqqqff] (0,0) node[anchor=south west] {$(0,0)$};
66 \draw [fill=qqqqff] (5,0) circle (1.5pt);
67 \draw[color=qqqqff] (5,0) node[anchor=south] {$(5,0)$};
68 \end{tikzpicture}.
69 \caption{Diversos punts al pla cartesià}
70 \label{fig:coordenades-punts}
71 \end{figure}
74 L'\term{origen de coordenades}\index{origen!de coordenades} és el punt de coordenades $(0,0)$.
76 A partir d'aquest moment identificarem un punt amb les seves coordenades.
78 \begin{notation}[notació dels punts] Els punts es poden escriure de dues maneres diferents: $A = (0,1)$ o bé $A(0,1)$.
79 \end{notation}
81 \subsection{Punt mitjà}
82 Donats dos punts del pla, $P = (x_1,y_1)$ i $Q = (x_2,y_2)$, que determinen un segment, podem preguntar-nos quines són les cooordenades del punt mitjà\index{punt!mitjà} d'aquest segment, que podem anomenar $P_M$.
84 Aquest punt queda determinant per la expressió següent:
85 \begin{equation*}
86 P_M=\left( \frac{x_1+x_2}{2},\frac{y_1+y_2}{2}\right),
87 \end{equation*}
88 és a dir, $P_M$ s'obté fent la mitjana aritmètica de les coordenades de $P$ i $Q$.
90 \begin{example}Calculeu les coordenades del punt mitjà del segment determinat pels punts $P = \left( 0,-5\right)$ i $Q = \left( -3,1\right)$.%
91 \begin{equation*}
92 P_{M}=\left( \frac{0+(-3)}{2},\frac{-5+1}{2}\right) =\left( \frac{-3}{2}%
93 ,-2\right)
94 \end{equation*}
95 \end{example}
97 \begin{exercise}Calculeu les coordenades del punt mitjà del segment determinat pels punts $P = \left( -3,7\right)$ i $Q = \left( -5,3\right)$.
98 \end{exercise}
100 \begin{exercise}Donat el punt $P\left( 0,-5\right)$, calculeu les coordenades del punt simètric de $P$ respecte del punt $M = \left( -1,12\right)$.
101 \end{exercise}
103 \begin{claim}Hem de notar que, encara que pareixi que sí, aquest resultat no es pot estendre quan es vol trobar un punt que estigui a distància $1/3$ d'$A$ en el segment $\overline{AB}$ (en general, a distància $d \neq 1/2$). En aquest cas, s'haurà de procedir a raonar amb vectors (\autoref{seccio:vectors-2d}), per exemple trobant el vector $1/3 \cdot \overrightarrow{AB}$ i situant-lo amb origen $A$. El seu extrem final seria el punt desitjat.
104 \end{claim}
106 \section{Vectors}\label{seccio:vectors-2d}
108 \begin{definition}[vector fix]Un \term{vector fix}\index{vector!fix} és un segment orientat a l'espai (és a dir una fletxa), que té un \term{origen}\index{origen!d'un vector} (el punt on comença) i un \term{final}\index{final d'un vector} (punt on acaba). Els dos punts s'anomenen \term{extrems del vector}\index{extrems d'un vector}.
109 \end{definition}
111 Per tant, un vector té:
112 \begin{itemize}
113 \item Una \term{direcció}\index{direcció!d'un vector}: la recta sobre la qual està el vector
114 \item Un \term{sentit}\index{sentit!d'un vector}: cap a on apunta la fletxa. Si $A$ i $B$ són els extrems d'un vector, aleshores aquest vector pot tenir dos sentits:
115 \begin{itemize}
116 \item de $A$ cap a $B$ (punt origen és $A$ i el punt destí és $B$)
117 \item o de $B$ cap a $A$ (punt origen és $B$ i el punt destí és $A$)
118 \end{itemize}
119 \item La seva \term{longitud}\index{longitud!d'un vector}. Formalment s'anomena \term{mòdul} del vector\index{mòdul!d'un vector}
120 \end{itemize}
122 \begin{notation}[notació de vectors]Els vectors es denoten amb una fletxa a damunt del seu nom. D'aquesta manera escriurem $\overrightarrow{AB}$ per denotar el vector que té origen $A$ i final a $B$. Si volem obviar els extrems, podem escriure $\overrightarrow{u}$, per exemple.
123 \end{notation}
125 \begin{example}Siguin els vectors següents (\autoref{fig:diversos-vectors}):
126 \begin{itemize}
127 \item Els extems dels vectors són:
128 \begin{itemize}
129 \item El vector $\overrightarrow{a}$ té origen $(-1,1)$ i fi $(-3,-1)$
130 \item El vector $\overrightarrow{b}$ té origen $(-1,-1)$ i fi $(0,0)$
131 \item El vector $\overrightarrow{c}$ té origen $(-4,3)$ i fi $(-1,3)$
132 \item El vector $\overrightarrow{d}$ té origen $(-4,2)$ i fi $(-4,-1)$
133 \item El vector $\overrightarrow{u}$ té origen $(1,1)$ i fi $(3,3)$
134 \item El vector $\overrightarrow{v}$ té origen $(4,1)$ i fi $(6,3)$
135 \item El vector $\overrightarrow{w}$ té origen $(1,-1)$ i fi $(3,1)$
136 \end{itemize}
137 \item Els vectors $\overrightarrow{a}$, $\overrightarrow{b}$, $\overrightarrow{u}$, $\overrightarrow{v}$ i $\overrightarrow{w}$ tenen la mateixa direcció
138 \item Els vectors $\overrightarrow{b}$, $\overrightarrow{u}$, $\overrightarrow{v}$, $\overrightarrow{w}$ tenen el mateix sentit, però el vector $\overrightarrow{a}$ té sentit contrari
139 \item Els vectors $\overrightarrow{a}$, $\overrightarrow{u}$, $\overrightarrow{v}$, $\overrightarrow{w}$ tenen el mateix mòdul. El mòdul de $\overrightarrow{b}$ és la meitat que el mòdul de $\overrightarrow{u}$. I $\overrightarrow{c}$ i $\overrightarrow{d}$ tenen el mateix mòdul (encara que no tenguin ni la mateixa direcció ni sentit)
140 \end{itemize}
142 \begin{figure}[h!]
143 \centering
144 % Generat amb geogebra. Modificat manualment
145 \begin{tikzpicture}[line cap=round,line join=round,>=triangle 45,x=1.0cm,y=1.0cm]
146 \draw [color=cqcqcq,dash pattern=on 2pt off 2pt, xstep=1.0cm,ystep=1.0cm] (-5.1,-2.1) grid (6.1,4.1);
147 \draw[->,color=black] (-5.5,0) -- (6.2,0);
148 \foreach \x in {-5,-4,-3,-2,-1,1,2,3,4,5,6}
149 \draw[shift={(\x,0)},color=black] (0pt,2pt) -- (0pt,-2pt) node[below] {\footnotesize $\x$};
150 \draw[->,color=black] (0,-2.1) -- (0,4.1);
151 \foreach \y in {-2,-1,1,2,3,4}
152 \draw[shift={(0,\y)},color=black] (2pt,0pt) -- (-2pt,0pt) node[left] {\footnotesize $\y$};
153 \draw[color=black] (0pt,-10pt) node[right] {\footnotesize $0$};
154 \draw [->] (1.,1.) -- (3.,3.);
155 \draw [->] (4.,1.) -- (6.,3.);
156 \draw [->] (1.,-1.) -- (3.,1.);
157 \draw [->] (-1.,1.) -- (-3.,-1.);
158 \draw [->] (-1.,-1.) -- (0.,0.);
159 \draw [->] (-4.,3.) -- (-1.,3.);
160 \draw [->] (-4.,2.) -- (-4.,-1.);
161 \draw[color=black] (2.0038916669677724,2.188648367438086) node {$u$};
162 \draw[color=black] (4.99308545055815,2.188648367438086) node {$v$};
163 \draw[color=black] (2.0038916669677724,0.17924587958011154) node {$w$};
164 \draw[color=black] (-1.9152735159618333,0.09621271892482333) node[anchor=south] {$a$};
165 \draw[color=black] (-0.5037097848219328,-0.3189530843516177) node {$b$};
166 \draw[color=black] (-2.4632923762867356,3.201652927432602) node {$c$};
167 \draw[color=black] (-3.908069371688751,0.6442315792497255) node {$d$};
168 \end{tikzpicture}
169 \caption{Diversos vectors al pla}
170 \label{fig:diversos-vectors}
171 \end{figure}
174 \end{example}
176 \begin{definition}[vector lliure]Un \term{vector lliure}\index{vector!lliure} és un segment orientat al pla, però del qual tenim la llibertat de triar el seu origen. És a dir, vector que tenen la mateixa direcció, sentit i longitud són a partir d'ara iguals per a nosaltres, independentment d'on estiguin situats. Formalment aquests vectors s'anomenen \term{equipolents}\index{vector!equipolent}
177 \end{definition}
179 En general, si no se'ns diu el contrari, o no se'ns dóna l'origen d'un vector, es suposarà que aquest és lliure. A més sempre suposarem que l'origen del vector és l'origen de coordenades i, per tant, escriurem el vector com a $\overrightarrow{v} = \overrightarrow{(3,5)}$ i no $\overrightarrow{v} = \overrightarrow{(0,0)(3,5)}$, obviant el seu origen.
181 A més, de la mateixa manera que pels punts, existeixen dues notacions estàndard: $\overrightarrow{v} = \overrightarrow{(3,5)}$ o bé $\overrightarrow{v}(3,5)$, que podrem usar indistintament.
183 \begin{example}Els vectors $\overrightarrow{u}$, $\overrightarrow{v}$ i $\overrightarrow{w}$ són equipolents (\autoref{fig:diversos-vectors}). És més, tots aquests vectors es consideren el mateix vector que $\overrightarrow{(2,2)}$. En canvi, el vector $\overrightarrow{a}$ no és el mateix vector, ja que té sentit contrari.
184 \end{example}
186 \begin{definition}[coordenades i components d'un vector]Donat un vector $\overrightarrow{v}$, les seves \term{coordenades}\index{coordenada d'un vector} són els nombres que formen el seu producte cartesià, és a dir, si $\overrightarrow{v} = (v_x, v_y)$, aleshores, $v_x$ i $v_y$ són les seves coordenades. $v_x$ es diu \term{coordenada de l'eix de les abscises} i $v_y$, \term{coordenada de l'eix de les ordenades}, o simplement coordenada de l'eix $X$ i coordenada de l'eix $Y$, respectivament.
188 Les coordenades es poden interpretar com a les longituds, amb signe, de les projeccions d'un vector sobre els dos eixos de coordenades. Cadascuna de les dues components d'un vector pot ser positiva o negativa segons que la respectiva projecció apunti cap a la part positiva o negativa del
189 corresponent eix de coordenades (figura~\autoref{fig:components-vector-2D}). En aquest sentit les coordenades s'anomenen \term{components}.
191 \begin{figure}[h!]
192 \centering
193 % Generat amb geogebra. Modificat manualment
194 \begin{tikzpicture}[line cap=round,line join=round,>=triangle 45,x=1.0cm,y=1.0cm]
195 \draw [color=cqcqcq,dash pattern=on 2pt off 2pt, xstep=1.0cm,ystep=1.0cm] (-1.1,-1.1) grid (8,10);
196 \draw[->,color=black] (-1,0) -- (8,0);
197 \draw[->,color=black] (0,-1) -- (0,10);
198 \draw[color=black] (0,0) node[anchor=north west] {$O$};
199 \draw[color=black] (7.5,0) node[anchor=north] {$x$};
200 \draw[color=black] (0,9.5) node[anchor=east] {$y$};
201 \draw [->,color=qqttcc] (1,2) -- (5,7);
202 \draw [->] (1,2) -- (5,2);
203 \draw [->] (1,2) -- (1,7);
204 \draw (3.5,4.8) node {$v$};
205 \draw (3,2) node[anchor=south] {$v_x$};
206 \draw (1,4.5) node[anchor=west] {$v_y$};
207 \end{tikzpicture}
208 \caption{Components d'un vector}
209 \label{fig:components-vector-2D}
210 \end{figure}
213 \end{definition}
215 \begin{example}Siguin els vectors següents:%
216 \begin{equation*}
217 \overrightarrow{u} = (3,-2),\, \overrightarrow{v} = (-5,1)
218 \end{equation*}%
219 El vector $\overrightarrow{v}$ apunta cap a la dreta i cap a baix, i el vector $\overrightarrow{u}$ apunta cap a l'esquerre i cap a dalt. Com que no se'ns diu quins són els seus origens, es considerarà que aquests vectors són lliures, i que, per tant, podem situar-los els on es desitgi.
220 \end{example}
222 \begin{exercise}Representeu gràficament els vectors $\overrightarrow{u} = (-3,4)$, $\overrightarrow{v} = (5,-1)$ i $\overrightarrow{w} = (1,0)$.
223 \end{exercise}
225 \begin{claim}[vector d'extrems donats]Donats els punts $P(x_{1},y_{1})$ i $Q(x_{2},y_{2})$, el vector $\overrightarrow{PQ}$ que té origen en $P$ i final en $Q$ té les components següents:%
226 \begin{equation*}
227 \overrightarrow{PQ}=(x_{2}-x_{1},y_{2}-y_{1}),
228 \end{equation*}
229 és a dir, restem les coordenades del punt final menys les coordenades del punt inicial.
230 \end{claim}
232 \begin{example}Calculeu les components del vector que comença en el punt $P\left(0,-6\right)$ i acaba en el punt $Q\left(-3,2\right)$:
233 \begin{equation*}
234 \overrightarrow{PQ}=\overrightarrow{\left( -3-0,2-\left( -6\right) \right)} =\overrightarrow{\left(-3,8\right)}
235 \end{equation*}
236 \end{example}
238 \begin{exercise}Calculeu les components del vector d'origen $P = \left( -2,1\right)$ i que acaba en el punt $Q = \left( -3,-5\right)$.
239 \end{exercise}
241 \begin{exercise}Els punts $A(3,0)$, $B(-5,4)$ i $C(6,-4)$ són vèrtexos d'un para\lgem{elogram}. Representeu gràficament aquests punts i calculeu les cooordenades de vèrtex restant. Trobeu els vectors que tenen com a extrems els vèrtex del para\lgem{elogram}.
242 \end{exercise}
244 \begin{definition}[mòdul d'un vector]El \term{mòdul} d'un vector\index{mòdul!d'un vector} és la seva longitud. El mòdul del vector $\overrightarrow{u} = (a,b)$, que es representa per $\left\vert \overrightarrow{u}\right\vert$, es calcula amb la fórmula:%
245 \begin{equation*}
246 \left\vert \overrightarrow{u}\right\vert =\left\vert (a,b)\right\vert =\sqrt{a^{2}+b^{2}}.
247 \end{equation*}
248 \end{definition}
250 \begin{example}El mòdul del vector $\overrightarrow{u} = \overrightarrow{(3,-2)}$ és:%
251 \begin{equation*}
252 \left\vert \overrightarrow{u}\right\vert =\sqrt{3^{2}+\left( -2\right) ^{2}}=\sqrt{13}.
253 \end{equation*}
254 \end{example}
256 \begin{exercise}
257 Calculeu el valor del mòdul del vector $\overrightarrow{u}(-5,1)$.
258 \end{exercise}
260 \bigskip
261 Acabem amb unes quantes definicions:
263 \begin{definition}[vector unitari] Un vector és \term{unitari}\index{vector!unitari} quan té mòdul 1.
264 \end{definition}
266 \begin{definition}[ortogonalitat, ortonormalitat]Donats dos vectors $\overrightarrow{u}$ i $\overrightarrow{v}$, direm que $\overrightarrow{u}$ és \term{ortogonal}\index{ortogonalitat}\index{vector!ortogonal} a $\overrightarrow{v}$ simplement quan $\overrightarrow{u}$ sigui perpendicular\index{vector!perpendicular} a $\overrightarrow{v}$, és a dir, quan ambdós formen un angle de 90 graus.
268 Si a més, $\overrightarrow{u}$ és unitari, aleshores direm que $\overrightarrow{u}$ és \term{ortonormal}\index{ortonormalitat}\index{vector!ortonormal} a $\overrightarrow{v}$.
269 \end{definition}
272 \subsection{Operacions amb vectors}
274 Definim aquí les diferents operacions que es poden fer amb vectors.
276 \subsubsection{Suma de dos vectors}
278 \begin{definition}[suma de dos vectors]. Siguin $\overrightarrow{u} = \overrightarrow{(a,b)}$ i $\overrightarrow{v} = \overrightarrow{(c,d)}$ dos vectors. La seva \term{suma}\index{suma!de vectors} es defineix com:
279 \begin{equation*}
280 \overrightarrow{u}+\overrightarrow{v}=\overrightarrow{(a,b)}+\overrightarrow{(c,d)}=\overrightarrow{\left( a+c,b+d\right)}
281 \end{equation*}
282 \end{definition}
284 \begin{example}
285 Donats els vectors $\overrightarrow{u} = \overrightarrow{(3,-2)}$ i $\overrightarrow{v} = \overrightarrow{(-5,1)}$, la seva suma és:%
286 \begin{equation*}
287 \overrightarrow{u}+\overrightarrow{v}=\overrightarrow{(3,-2)}+\overrightarrow{(-5,1)}=\overrightarrow{\left( 3-5,-2+1\right)}=\overrightarrow{\left( -2,-1\right)}
288 \end{equation*}
289 \end{example}
291 Noteu que, per a què es puguin sumar dos vectors aquests han de tenir el mateix origen o bé ser lliures. En aquest cas, la suma de dos vectors es pot calcular gràficament: en el dibuix següent es representa la suma gràfica de $\overrightarrow{u}$ i $\overrightarrow{v}$ (\autoref{fig:regla-del-parallelogram}):
293 \begin{figure}[h!]
294 \centering
295 % Generat amb geogebra. Modificat manualment
296 \begin{tikzpicture}[line cap=round,line join=round,>=triangle 45,x=1.0cm,y=1.0cm]
297 \draw [->] (-3.,1.) -- (-2.,3.);
298 \draw [->] (-2.,1.) -- (1.,2.);
299 \draw (-3.08,2.48) node[anchor=north west] {$\overrightarrow{u}$};
300 \draw (-0.5,1.66) node[anchor=north west] {$\overrightarrow{v}$};
301 \end{tikzpicture}
302 \definecolor{ccqqqq}{rgb}{0.8,0.,0.}
303 \definecolor{qqqqcc}{rgb}{0.,0.,0.8}
304 \begin{tikzpicture}[line cap=round,line join=round,>=triangle 45,x=1.0cm,y=1.0cm]
305 \draw [->] (-2.,1.) -- (-1.,3.);
306 \draw [->] (-2.,1.) -- (1.,2.);
307 \draw (-2.12,2.54) node[anchor=north west] {$\overrightarrow{u}$};
308 \draw (-0.5,1.66) node[anchor=north west] {$\overrightarrow{v}$};
309 \draw [->,color=qqqqcc] (-1.,3.) -- (2.,4.);
310 \draw [->,color=qqqqcc] (1.,2.) -- (2.,4.);
311 \draw [->,color=ccqqqq] (-2.,1.) -- (2.,4.);
312 \draw (-0.92,3.26) node[anchor=north west] {$\overrightarrow{u} + \overrightarrow{v}$};
313 \end{tikzpicture}
314 \caption{Regla del para\lgem{elogram} per al càlcul de la suma de vectors}
315 \label{fig:regla-del-parallelogram}
316 \end{figure}
318 Es pot procedir de manera anàloga per a qualssevol vectors. Aquesta manera gràfica d'aconseguir la suma es coneix com \term{regla del para\lgem{elogram}}\index{regla!del para\lgem{elogram}}.
320 \begin{exercise}Calculeu gràficament i analítica la suma dels vectors $\overrightarrow{u} = \overrightarrow{(-5,4)}$ i $\overrightarrow{v} = \overrightarrow{(3,-1)}$.
321 \end{exercise}
323 \subsubsection{Diferència de dos vectors}
325 \begin{definition}[diferència de dos vectors] Donats dos vectors $\overrightarrow{u} = \overrightarrow{(a,b)}$ i $\overrightarrow{v} = \overrightarrow{(c,d)}$, la seva \term{diferència}\index{diferència!de dos vectors} es defineix com:
326 \begin{equation*}
327 \overrightarrow{u}-\overrightarrow{v}=\overrightarrow{(a,b)}-\overrightarrow{(c,d)}=\overrightarrow{\left( a-c,b-d\right)}
328 \end{equation*}
329 \end{definition}
331 \begin{example}
332 Donats els vectors $\overrightarrow{u} = \overrightarrow{(3,-2)}$ i $\overrightarrow{v} = \overrightarrow{(-5,1)}$, la seva diferència és:%
333 \begin{equation*}
334 \overrightarrow{u}-\overrightarrow{v} = \overrightarrow{(3,-2)}-\overrightarrow{(-5,1)} = \overrightarrow{\left( 3+5,-2-1\right)} = \overrightarrow{\left( 8,-3\right)}
335 \end{equation*}
336 \end{example}
338 \begin{exercise}
339 Calculeu $\overrightarrow{u}-\overrightarrow{v}$ i $\overrightarrow{v}-\overrightarrow{u}$, amb $\overrightarrow{u} = \overrightarrow{(-5,4)}$ i $\overrightarrow{v} = \overrightarrow{(3,-1)}$.
340 \end{exercise}
342 \subsubsection{Producte d'un escalar per un vector}
344 \begin{definition}[escalar]Quan parlem de vectors, els nombres s'anomenen \term{escalars}\index{escalar}
345 \end{definition}
347 \begin{definition}[producte d'un escalar per un vector] Donat un nombre $k \in \mathbb{R}$ i un vector $\overrightarrow{u}(a,b)$, \term{el producte de $k$ per $\overrightarrow{A}$}\index{producte!d'un escalar per vector}, $k \cdot \overrightarrow{u}$, es defineix com:
348 \begin{equation*}
349 k \cdot \overrightarrow{u} = k\cdot \overrightarrow{(a,b)}=\overrightarrow{\left( ka, kb\right)}
350 \end{equation*}
351 \end{definition}
353 \begin{example}
354 Donats el vector $\overrightarrow{u} = \overrightarrow{(3,-2)}$ i el número $k=-5$, es té que el seu producte és:%
355 \begin{equation*}
356 k\text{$\cdot $}\overrightarrow{u}=-5\cdot \overrightarrow{(3,-2)}=\overrightarrow{\left( -5\cdot 3,-5\cdot \left( -2\right) \right)} = \overrightarrow{\left( -15,10\right)}
357 \end{equation*}
358 \end{example}
360 En el dibuix següent es veu un exemple gràfic del producte d'un nombre (en aquest cas el $3$) per un vector (\autoref{fig:producte-escalar-per-vector}):%
362 \begin{figure}[h!]
363 \centering
364 % Generat amb geogebra. Modificat manualment
365 \definecolor{ttzzqq}{rgb}{0.2,0.6,0.}
366 \definecolor{qqqqff}{rgb}{0.,0.,1.}
367 \definecolor{ffqqqq}{rgb}{1.,0.,0.}
368 \begin{tikzpicture}[line cap=round,line join=round,>=triangle 45,x=1.0cm,y=1.0cm]
369 \clip(-1.,-1.) rectangle (10.,4.);
370 \draw [->] (0.,0.) -- (2.,1.);
371 \draw [->] (3.,0.) -- (9.,3.);
372 \draw (0.46,1.26) node[anchor=north west] {$\overrightarrow{u}$};
373 \draw (4.94,2.2) node[anchor=north west] {$3\overrightarrow{u}$};
374 \draw [line width=1.2pt,color=ffqqqq] (3.,0.)-- (5.,1.);
375 \draw [line width=1.2pt,color=ffqqqq] (4.10733126291999,0.5536656314599954) -- (4.067082039324995,0.3658359213500132);
376 \draw [line width=1.2pt,color=ffqqqq] (4.10733126291999,0.5536656314599954) -- (3.932917960675006,0.6341640786499876);
377 \draw [line width=1.2pt,color=qqqqff] (5.,1.)-- (7.,2.);
378 \draw [line width=1.2pt,color=qqqqff] (6.214662525839981,1.60733126291999) -- (6.174413302244984,1.4195015528100081);
379 \draw [line width=1.2pt,color=qqqqff] (6.214662525839981,1.60733126291999) -- (6.040249223594997,1.6878297101099824);
380 \draw [line width=1.2pt,color=qqqqff] (6.,1.5) -- (5.959750776405003,1.312170289890018);
381 \draw [line width=1.2pt,color=qqqqff] (6.,1.5) -- (5.825586697755016,1.5804984471899926);
382 \draw [line width=1.2pt,color=ttzzqq] (7.,2.)-- (9.,3.);
383 \draw [line width=1.2pt,color=ttzzqq] (8.107331262919992,2.553665631459995) -- (8.067082039324994,2.365835921350013);
384 \draw [line width=1.2pt,color=ttzzqq] (8.107331262919992,2.553665631459995) -- (7.932917960675007,2.634164078649988);
385 \draw [line width=1.2pt,color=ttzzqq] (7.89266873708001,2.4463343685400054) -- (7.852419513485014,2.2585046584300232);
386 \draw [line width=1.2pt,color=ttzzqq] (7.89266873708001,2.4463343685400054) -- (7.718255434835026,2.526832815729998);
387 \draw [line width=1.2pt,color=ttzzqq] (8.32199378875997,2.660996894379985) -- (8.281744565164972,2.4731671842700025);
388 \draw [line width=1.2pt,color=ttzzqq] (8.32199378875997,2.660996894379985) -- (8.147580486514986,2.741495341569977);
389 \draw (1.66,0.68) node[anchor=north west] {$\times 3 =$};
390 \end{tikzpicture}
391 \caption{Exemple d'un producte d'un escalar per un vector}
392 \label{fig:producte-escalar-per-vector}
393 \end{figure}
395 \begin{exercise}
396 Calculeu gràficament i analítica el producte $-3\cdot \overrightarrow{u}$, amb $\overrightarrow{u} = \overrightarrow{(3,-1)}$.
397 \end{exercise}
399 \begin{claim}L'operació del producte d'un escalar per un vector dóna sempre un vector para\lgem{el} al vector inicial. És més, el vector resultant és una allargament (si $k > 1$) o una escurçament (si $k < 1$) del vector original.
400 \end{claim}
402 \begin{claim}Sempre podem obtenir un vector para\lgem{el} a un donat i que sigui unitari, multiplicant per la inversa del seu mòdul. És a dir, $\frac{1}{\lvert \overrightarrow{u} \rvert} \overrightarrow{u}$ sempre és un vector unitari para\lgem{el} a $\overrightarrow{u}$.
403 \end{claim}
405 \begin{proposition}[Condició de pare\lgem{elisme} entre dos vectors]\label{resultat:proposicio-parallelisme-vectors}Siguin $\overrightarrow{u}(a,b)$ i $\overrightarrow{v}(c,d)$ dos vectors qualssevol amb $a, b \neq 0$:%
406 \begin{equation*}
407 \overrightarrow{u} \text{ és para\lgem{el} a } \overrightarrow{v} \iff \frac{c}{a}=\frac{d}{b}
408 \end{equation*}
409 \end{proposition}
411 Expressat en paraules, això ens diu que si dos vectors són para\lgem{els}, aleshores el quocient entre les seves respectives components dóna el mateix resultat, i viceversa, és a dir, que si el quocient entre les respectives components de dos vectors dóna el mateix resultat, aleshores aquests dos vectors són para\lgem{els}.
413 \begin{remark}Si alguns dels vectors anteriors té alguna component igual a zero, l'altre vector per ser-hi proporcional ha de tenir la mateixa component igual a zero. Per exemple $\overrightarrow{u} = \overrightarrow{(5,0)}$ és proporcional a $\overrightarrow{v} = \overrightarrow{(-7,0)}$ (perquè $\frac{-7}{5} \cdot \overrightarrow{u} = \overrightarrow{v}$) però no ho és a $\overrightarrow{w_1} = \overrightarrow{(0,3)}$ ni a $\overrightarrow{w_2} = \overrightarrow{(5,3)}$.
414 \end{remark}
416 \begin{example}Determineu, a cadascun dels apartats següents, si els vectors són para\lgem{els} entre si:
417 \begin{enumerate}[label=\emph{\alph*})]
418 \item $\overrightarrow{a} = \overrightarrow{\left( 2,-3\right)}$ i $\overrightarrow{b} = \overrightarrow{\left( 4,-6\right)}$:%
419 \begin{equation*}
420 \frac{2}{4}=\frac{-3}{-6}
421 \end{equation*}%
422 Per tant, aquests dos vectors són para\lgem{els} entre si.
424 \item $\overrightarrow{c} = \overrightarrow{\left( 2,-1\right)}$ i $\overrightarrow{d}= \overrightarrow{\left( 4,-3\right)}$:%
425 \begin{equation*}
426 \frac{2}{4}\neq \frac{-1}{-3}
427 \end{equation*}%
428 Així, aquests dos vectors no són para\lgem{els} entre si.
429 \end{enumerate}
430 \end{example}
432 \begin{exercise}
433 Determineu si els vectors següents són para\lgem{els} entre si:
434 \begin{enumerate}[label=\emph{\alph*})]
435 \item $\overrightarrow{a} = \overrightarrow{\left( 1,-3\right)}$ i $\overrightarrow{b} = \overrightarrow{\left( 5,-6\right)}$
437 \item $\overrightarrow{c} = \overrightarrow{\left( 3,-1\right)}$ i $\overrightarrow{d}= \overrightarrow{\left( -6,2\right)}$
439 \item $\overrightarrow{e} = \overrightarrow{\left( 3,0\right)}$ i $\overrightarrow{f} = \overrightarrow{\left( 5,0\right)}$
440 \end{enumerate}
441 \end{exercise}
443 \paragraph{Producte escalar de dos vectors}
445 \begin{definition}[producte escalar de dos vectors]El \term{producte escalar de dos vectors}\index{producte!escalar}, $\overrightarrow{u}(a,b)$ i $\overrightarrow{v}(c,d)$, que es denota per $\overrightarrow{u}\cdot \overrightarrow{v}$, es defineix de la manera següent:%
446 \begin{equation}\label{eq:producte-escalar}
447 \overrightarrow{u}\cdot\overrightarrow{v}=(a,b)\cdot(c,d)=a\cdot c+b\cdot d
448 \end{equation}
449 \end{definition}
451 Com es veu, el producte escalar de dos vectors és un nombre.
453 \begin{example}El producte escalar dels vectors $\overrightarrow{u} = \overrightarrow{(2,0)}$ i $\overrightarrow{v} = \overrightarrow{(-3,1)}$ és igual a:%
454 \begin{equation*}
455 \overrightarrow{u}\cdot\overrightarrow{v}=(2,0)\cdot(-3,1)=2\cdot\left( -3\right) +0\cdot 1=-6
456 \end{equation*}
457 \end{example}
459 \begin{exercise}Calculeu $\overrightarrow{u}\cdot \overrightarrow{v}$, amb $\overrightarrow{u} = \overrightarrow{(-3,4)}$ i $\overrightarrow{v} = \overrightarrow{(-2,-8)}$.
460 \end{exercise}
462 \begin{exercise}Per a quins valors de $\lambda$ el producte escalar $\overrightarrow{(3,-4)} \cdot \overrightarrow{(\lambda, 2)}$ serà $5$?
463 \end{exercise}
465 \begin{exercise}Sigui $\overrightarrow{u} = \overrightarrow{(2,-3)}$. Trobeu un vector para\lgem{el} i dieu-li $\overrightarrow{v}$. Calculeu el producte $\overrightarrow{u} \cdot \overrightarrow{v}$.
466 \end{exercise}
468 \begin{exercise}Trobeu un vector unitari que sigui para\lgem{el} a $\overrightarrow{u} = \overrightarrow{\left( -8, -15 \right)}$.
469 \end{exercise}
471 \paragraph{Angle entre dos vectors}
473 \begin{proposition}[Relació entre producte escalar i angle entre dos vectors]\label{resultat:angle-producte-esclar}Es pot provar que es cumpleix que la relació:%
474 \begin{equation}\label{eq:angle-producte-escalar}
475 \overrightarrow{u}\cdot\overrightarrow{v}=\left\vert \overrightarrow{u}\right\vert \cdot \left\vert \overrightarrow{v} \right\vert \cdot \cos \alpha,
476 \end{equation}%
477 on $\alpha$ és l'angle que formen entre si els vectors $\overrightarrow{u}$ i $\overrightarrow{v}$.
479 Això permet calcular l'angle $\alpha$ entre dos vectors, o qualsevol altre variable desconeguda d'aquesta fórmula \eqref{eq:angle-producte-escalar} si es coneixen les altres. Recordeu que el producte escalar es pot calcular amb seva fórmula \eqref{eq:producte-escalar}. Per tant, l'equació anterior és equivalent a:
480 \begin{equation}
481 ac+bd=\sqrt{a^{2}+b^{2}}\text{$\cdot $}\sqrt{c^2+d^2}\cdot\cos \alpha
482 \end{equation}
483 \end{proposition}
485 \begin{claim}Recordeu que el cosinus d'un angle es defineix com la projecció del radi definit per l'angle sobre el diàmetre horitzontal de la circumferència de radi unitat.
487 Els valors del cosinus dels angles més usuals es mostren a continuació (taula~\autoref{tab:taula-valors-cosinus}):
489 \begin{table}[ht!]
490 \centering
491 \begin{tabular}{r|c|c|c|c|c|c|c|}
492 \cline{2-8}
493 & $0$ \si{\radian} & $\pi/6$ \si{\radian} & $\pi/4$ \si{\radian} & $\pi/3$ \si{\radian} & $\pi/2$ \si{\radian} & $\pi$ \si{\radian} & $3\pi/2$ \si{\radian} \\
494 \cline{2-8}
495 & $0\degree$ & $30\degree$ & $45\degree$ & $60\degree$ & $90\degree$ & $180\degree$ & $270\degree$\\
496 \hline
497 \multicolumn{1}{|r|}{$\cos \alpha$} & $1$ & $\sqrt{3}/2$ & $\sqrt{2}/2$ & $1/2$ & $0$ & $-1$ & $0$\\
498 \hline
499 \end{tabular}%
500 \caption{Valors dels cosinus pels angles més usuals}
501 \label{tab:taula-valors-cosinus}
502 \end{table}
504 Encara que es pot calcular el cosinus d'un angle arbitrari amb la calculadora científica.
505 \end{claim}
509 \begin{example}Què val l'angle format pels vectors $\overrightarrow{u} = \overrightarrow{(2,0)}$ i $\overrightarrow{v} = \overrightarrow{(-3,1)}$?
512 \bigskip
513 Si aplicam la darrera fórmula i denotam l'angle per $\alpha $, es té que%
514 \begin{align*}
515 2\cdot\left( -3\right) +0\cdot 1 &= \sqrt{2^{2}+0^{2}} \cdot \sqrt{\left( -3\right) ^{2}+1^{2}}\cdot \cos \alpha
517 -6 &=2 \cdot \sqrt{10} \cdot \cos \alpha \\
518 \cos \alpha &= \frac{-6}{2\sqrt{10}}=\frac{-3}{\sqrt{10}} \\
519 \alpha &= \arccos \frac{-3}{\sqrt{10}}\simeq 161,565\degree%
520 \end{align*}
521 \end{example}
523 \begin{exercise}Calculeu l'angle que formen entre si els vectors $\overrightarrow{u} = \overrightarrow{(-2,-5)}$ i $\overrightarrow{v} = \overrightarrow{(-3,2)}$.
524 \end{exercise}
527 \bigskip
528 Vegem a continuació les propietats del producte escalar.
530 \begin{theorem}[Propietats del producte escalar]\label{resultat:propietats-del-producte-esclar} Donats vectors $\overrightarrow{u},\overrightarrow{v}$ i $\overrightarrow{w}$ i un nombre $k$ qualssevol, el producte escalar té les propietats següents:
532 \begin{enumerate}[label=\emph{\alph*})]
533 \item $\left\vert \overrightarrow{u}\right\vert =\sqrt{\overrightarrow{u}\cdot\overrightarrow{u}}$. És a dir, el mòdul d'un vector es pot calcular amb l'arrel quadrada del producte escalar per si mateix.
535 \item $\overrightarrow{u}\cdot \overrightarrow{v}=\overrightarrow{v}\cdot \overrightarrow{u}$ (propietat conmutativa)
537 \item $\left( k\overrightarrow{u}\right) \cdot \overrightarrow{v}=k\left( \overrightarrow{u}\text{$\cdot $}\overrightarrow{v}\right)$ (propietat associativa)
539 \item $\overrightarrow{u}\cdot \left( \overrightarrow{v}+\overrightarrow{w}\right) =\overrightarrow{u}\cdot \overrightarrow{v}+\overrightarrow{u}\cdot \overrightarrow{w}$ (propietat distributiva)
541 \item \term{Condició de perpendicularitat entre dos vectors}\index{condició!de perpendicularitat entre dos vectors}: si el producte escalar de dos vectors és $0$, aleshores aquests dos vectors són perpendiculars entre si, i viceversa, és a dir, que si dos vectors són perpendiculars entre si, aleshores el seu producte escalar és $0$. Matemàticament,%
542 \begin{equation*}
543 \overrightarrow{u}\cdot\overrightarrow{v}=0\iff \overrightarrow{u} \bot \overrightarrow{v}
544 \end{equation*}
545 \end{enumerate}
546 \end{theorem}
548 \begin{example}Per exemple, els vectors $\overrightarrow{u} = \overrightarrow{\left( 30,-9\right)}$ i $\overrightarrow{v} = \overrightarrow{\left( 3,10\right)}$ són perpendiculars, ja que $\overrightarrow{u}\cdot \overrightarrow{v}=30\cdot 3+\left( -9\right) \cdot 10=0$.
549 \end{example}
551 \begin{exercise}En cada cas, calculeu $x$ per a què els vectors $\overrightarrow{u}= \overrightarrow{\left( 8,-15\right)}$ i $\overrightarrow{v} = \overrightarrow{\left( 2,x\right)}$ siguin:
552 \begin{enumerate}[label=\emph{\alph*})]
553 \item para\lgem{els},
555 \item perpendiculars,
556 \end{enumerate}
557 \end{exercise}
559 \begin{exercise}Donat el vector $\overrightarrow{u} = \overrightarrow{\left( 5,12\right)}$, trobeu:
560 \begin{enumerate}[label=\emph{\alph*})]
561 \item un vector para\lgem{el},
563 \item un vector perpendicular.
564 \end{enumerate}
565 \end{exercise}
567 \section{La recta en el pla}
569 En aquest apartat farem un estudi de la recta en un espai de dues dimensions.
571 Una \term{recta}\index{recta}, en particular, és una co\lgem{ecció} de punts. Per tant, un objectiu principal serà trobar les coordenades de tots els seus punts. La manera més senzilla de trobar-la és usar vectors.
573 Donada una recta $r$, sempre podem obtenir un punt qualsevol $P$ i un vector $\overrightarrow{v}$ sobre aquesta --- per exemple, si sabéssim dos punts $A$ i $B$ sobre la recta, aleshores tendríem un punt, $A$ o $B$, i un vector amb aquestes condicions, $\overrightarrow{AB}$ o qualsevol múltiple seu. Per tant, per a qualsevol punt $X$ sobre la recta, aquest forma el vector $\overrightarrow{OX}$, que té com a origen l'origen de coordenades i com a destí $X$. Aquest vector es pot posar com a suma del vector $\overrightarrow{OP}$ i un múltiple del vector $\overrightarrow{v}$ (vegeu la figura \autoref{fig:equacio-vectorial-recta-2d}), és a dir, existeix un nombre $\lambda$ tal que:
574 \begin{equation}\label{eq:equacio-vectorial-recta-2d}
575 \overrightarrow{OX} = \overrightarrow{OP} + \lambda \cdot \overrightarrow{v}.
576 \end{equation}
578 \begin{figure}[h!]
579 \centering
580 % Generat amb geogebra. Modificat manualment
581 \definecolor{ffqqqq}{rgb}{1.,0.,0.}
582 \definecolor{uuuuuu}{rgb}{0.26666666666666666,0.26666666666666666,0.26666666666666666}
583 \definecolor{xdxdff}{rgb}{0.49019607843137253,0.49019607843137253,1.}
584 \begin{tikzpicture}[line cap=round,line join=round,>=triangle 45,x=1.0cm,y=1.0cm]
585 \draw[->,color=black] (-3.,0.) -- (5.5,0.);
586 \foreach \x in {-3,-2,-1,1,2,3,4,5}
587 \draw[shift={(\x,0)},color=black] (0pt,2pt) -- (0pt,-2pt) node[below] {\footnotesize $\x$};
588 \draw[->,color=black] (0.,-1.) -- (0.,5.);
589 \foreach \y in {-1,1,2,3,4}
590 \draw[shift={(0,\y)},color=black] (2pt,0pt) -- (-2pt,0pt) node[left] {\footnotesize $\y$};
591 \draw[color=black] (0pt,-10pt) node[right] {\footnotesize $0$};
592 \clip(-3.,-1.) rectangle (5.5,5.);
593 \draw [dash pattern=on 3pt off 3pt,domain=-3.:5.5] plot(\x,{(--6.--2.*\x)/4.});
594 \draw [->] (0.,0.) -- (1.704,2.352);
595 \draw [->] (0.,0.) -- (4.128,3.564);
596 \draw [->,color=ffqqqq] (-1.,1.) -- (0.632,1.816);
597 \draw (-1.9,1.44) node[anchor=north west] {$r$};
598 \draw (0,0) node[anchor=north east] {$O$};
599 \begin{scriptsize}
600 \draw [fill=xdxdff] (1.704,2.352) circle (1.5pt);
601 \draw[color=xdxdff] (1.84,2.64) node {P};
602 \draw [fill=xdxdff] (4.128,3.564) circle (1.5pt);
603 \draw[color=xdxdff] (4.26,3.84) node {X};
604 \draw [fill=uuuuuu] (0.,0.) circle (1.5pt);
605 \draw[color=ffqqqq] (-0.16,1.64) node {v};
606 \end{scriptsize}
607 \end{tikzpicture}
608 \caption{Visualització de l'equació vectorial d'una recta}
609 \label{fig:equacio-vectorial-recta-2d}
610 \end{figure}
612 Aquest equació \eqref{eq:equacio-vectorial-recta-2d} s'anomena \term{equació vectorial de la recta}\index{equació!vectorial!d'una recta} i al vector $\overrightarrow{v}$ se li diu \term{vector director} de $r$\index{vector!director}.
614 \begin{claim}Noteu que realment no fa falta que el vector director $v$ estigui sobre la recta. Basta qualsevol que tengui la mateixa direcció, ja que suposem que feim feina amb vectors lliures. En aquest sentit parlarem de {\em el} vector director de la recta $r$ i no d'{\em un} vector director, per a qualsevol d'aquests vectors, ja que els haurem identificat.
615 \end{claim}
617 \begin{example}Trobeu l'equació vectorial de la recta que passa pels punts $A = (2,3)$ i $B = (4,5)$.
619 Hem de prendre un punt de la recta i un vector director. Ja tenim el punt: podem prendre $A$ o $B$. Agafarem $A = (2,3)$.
621 Per trobar el vector director, calcularem $\overrightarrow{AB} = \overrightarrow{(4-2,5-3)} = \overrightarrow{(2,2)}$.
623 Per tant, l'equació vectorial de la recta en qüestió és:
624 \begin{equation*}
625 \overrightarrow{OX} = \overrightarrow{OA} + \lambda \overrightarrow{(2,2)}
626 \end{equation*}
628 Si denotam $X=(x,y)$ les coordenades del punt $X$, tenim que aquesta equació es transforma en:
629 \begin{equation*}
630 \overrightarrow{(x,y)} = \overrightarrow{(2,3)} + \lambda \overrightarrow{(2,2)}.
631 \end{equation*}
632 \end{example}
634 \begin{claim}
635 A part d'aquesta equació, n'hi ha d'altres però totes provénen d'aquesta. L'ús d'una o de l'altra dependrà de l'exercici concret que volguem resoldre i de la nostra comoditat.
636 \end{claim}
638 \subsection{Equació paramètrica de la recta}
640 Sigui $r$ una recta donada pel punt $P = (x_{1},y_{1})$ i el vector director $\overrightarrow{v_r} = \overrightarrow{(v_x,v_y)}$, aleshores l'equació vectorial de la recta $r$ ve donada per
641 \begin{equation*}
642 \overrightarrow{OX} = \overrightarrow{OP} + \lambda \cdot \overrightarrow{v},
643 \end{equation*}
644 on $X = (x,y)$ és un punt qualsevol de la recta. Si desenvolupem aquesta equació obtenim que
645 \begin{equation*}
646 \overrightarrow{(x,y)} = \overrightarrow{(x_1,y_1)} + \lambda \cdot \overrightarrow{(v_x, v_y)},
647 \end{equation*}
648 és a dir,
649 \begin{equation*}
650 \overrightarrow{(x,y)} = \overrightarrow{(x_1 + \lambda \cdot v_x, y_1 + \lambda v_y)}.
651 \end{equation*}
653 Dos vectors són iguals si, i només si, les seves components són iguals. Per tant, $(x,y) = (x_1 + \lambda \cdot v_x, y_1 + \lambda v_y)$, és a dir, s'han de complir simultàniament les equacions següents:
654 \begin{equation*}
655 \left\{\begin{aligned}
656 x & = x_1 + \lambda \cdot v_x,\\
657 y & = y_1 + \lambda \cdot v_y.
658 \end{aligned}
659 \right.
660 \end{equation*}
662 Hem obtingut l'\term{equació paramètrica}\index{equació!paramètrica!d'una recta}. L'equació paramètrica d'una recta dóna les coordenades de tots els punts d'una recta depenent d'un paràmetre $\lambda$ (d'aquí el seu nom). Per a cada valor de $\lambda$ obtenim un punt de la recta.
664 Recapitulant, si $r$ és una recta que passa pel punt $P = (x_1, y_1)$ i té com a vector director $\overrightarrow{v_r} = \overrightarrow{(v_x,v_y)}$, aleshores l'equació paramètrica de $r$ ve donada per:
665 \begin{equation}\label{eq:equacio-parametrica-recta-2d}
666 r:\left\{\begin{aligned}
667 x & =x_{1}+\lambda v_{x} \\
668 y & =y_{1}+\lambda v_{y}%
669 \end{aligned}%
670 \right.,
671 \end{equation}%
672 amb $\lambda \in \mathbb{R}$.
674 \begin{example}\label{exemple:equacio-parametria-recta-2d}Si una recta passa pel punt $\left( 0,-1\right)$ i el seu vector director és $\overrightarrow{v} = \overrightarrow{\left(-3,2\right)}$, aleshores la seva equació paramètrica és la següent:%
675 \begin{equation*}
676 \left. \begin{aligned}
677 x & =0+\lambda \left( -3\right) \\
678 y & =-1+\lambda \cdot 2
679 \end{aligned}%
680 \right\} ; \left.
681 \begin{aligned}
682 x & =-3\lambda \\
683 y & =-1+2\lambda%
684 \end{aligned}%
685 \right\}
686 \end{equation*}
687 \end{example}
689 Per trobar més punts d'aquesta recta basta substituir $\lambda $ per un nombre qualsevol a les expressions anteriors.
691 \begin{example}Si a la recta anterior feim $\lambda =2$, tenim que%
692 \begin{equation*}
693 \left. \begin{aligned}
694 x & =-6 \\
695 y & =-1+4=3%
696 \end{aligned}
697 \right\},
698 \end{equation*}%
699 i, per tant, que $\left( -6,3\right)$ és un altre punt de la recta.
700 \end{example}
702 \begin{exercise}Calculeu l'equació paramètrica de la recta que passa per $A = (-3,0)$ i segueix la direcció $\overrightarrow{v} = \overrightarrow{\left( 5,-1\right)}$. Trobeu tres punts més d'aquesta recta.
703 \end{exercise}
705 \begin{claim}Per saber si un punt pertany a una recta donada, només hem de veure si aquest punt verifica l'equació de la recta. Per exemple, si volem saber si $P = (5,3)$ pertany o no a la recta de l'\autoref{exemple:equacio-parametria-recta-2d}, només hem de substituir a les equacions:
706 \begin{equation*}
707 \left\{ \begin{aligned}
708 5 & = -3 \lambda\\
709 3 & = -1 + 2 \lambda
710 \end{aligned}\right. ,
711 \end{equation*}
712 i hem de resoldre aquest sistema. Si aquest sistema té solució, és a dir, existeix $\lambda$, aleshores $P$ pertanyarà a la recta; sinó, no ho farà. En el nostre cas, $\lambda = -5/3$ de la primera equació i $\lambda = 2$ de la segona. Per tant, $P$ no és de la recta.
714 Aquest fet també ens servirà per a les altres equacions de la recta.
715 \end{claim}
717 \subsection{Equació contínua de la recta}
719 Si aïllem $\lambda $ a cadascuna de les equacions de la recta en forma paramètrica \eqref{eq:equacio-parametrica-recta-2d}, obtenim
720 \begin{equation*}
721 \lambda =\frac{x-x_{1}}{v_{x}},\text{ }\lambda =\frac{y-y_{1}}{v_{y}}.
722 \end{equation*}
723 Si ara igualam les dues equacions, s'obté \term{l'equació contínua de la recta}\index{equació!contínua!d'una recta}
724 \begin{equation}\label{eq:equacio-continua-recta-2d}
725 r:\frac{x-x_{1}}{v_{x}}=\frac{y-y_{1}}{v_{y}},
726 \end{equation}
727 on $P = (x_{1},y_{1})$ és qualsevol punt de la recta i $\overrightarrow{v_{r}} = \overrightarrow{(v_x,v_u)}$ és el vector director de la recta.
729 \begin{example}Seguint amb la recta de l'exemple anterior, \autoref{exemple:equacio-parametria-recta-2d}, la seva equació contínua és:%
730 \begin{equation*}
731 \frac{x}{-3}=\frac{y+1}{2}
732 \end{equation*}
733 \end{example}
735 \begin{claim}Notem que si alguna component del vector director $\overrightarrow{v_r}$ és zero, aleshores no existeix la fracció corresponent a l'equació \eqref{eq:equacio-continua-recta-2d} (no es pot dividir per zero). Ara bé, en aquest cas es veu l'equació \eqref{eq:equacio-continua-recta-2d} com a {\em notació}.
737 Per exemple, la recta que passa pel punt $(2,3)$ i té com a vector director $\overrightarrow{(5,0)}$, té com a equació contínua:
738 \begin{equation*}
739 \frac{x-2}{5} = \frac{y-3}{0}
740 \end{equation*}
741 \end{claim}
743 \subsection{Equació general de la recta}\label{seccio:equacio-general-de-la-recta-2d}
745 Si a l'equació de la recta en forma contínua llevam els denominadors i ho transposam tot al primer membre, l'equació de la recta s'escriu de la manera següent:%
746 \begin{equation}\label{eq:equacio-general-recta-2d}
747 Ax+By+C=0,
748 \end{equation}%
749 amb $A$, $B$ i $C$ nombres reals. Aquesta equació rep el nom d'\term{equació general de la recta} o \term{equació implícita de la recta}\index{equació!general!d'una recta}\index{equació!implícita!d'una recta}.
751 \begin{example}Seguint amb la recta anterior, \autoref{exemple:equacio-parametria-recta-2d}, la seva equació general és:%
752 \begin{equation*}
753 r \equiv 2x=-3\left( y+1\right),
754 \end{equation*}
755 que simplificada és:
756 \begin{equation*}
757 r \equiv 2x+3y+3=0.
758 \end{equation*}
759 \end{example}
761 \begin{claim}Notem que, si a l'exemple anterior, feim $x = \lambda$, llavors
762 \begin{equation*}
763 y = \left(-3-2x\right)/3 = -1 + 2/3 \lambda,
764 \end{equation*}
765 per la qual cosa
766 \begin{equation*}
767 \begin{pmatrix}
770 \end{pmatrix} = \begin{pmatrix}
773 \end{pmatrix} + \begin{pmatrix}
775 -2/3
776 \end{pmatrix} \cdot \lambda.
777 \end{equation*}
778 Això implica que $r$ passa pel punt $(0,-1)$ i té com a vector director $\overrightarrow{(1,-2/3)}$. Noteu que aquest darrer vector director és equivalent a $(-3,2)$ (aquest darrer és el primer multiplicat per $3$), el qual és el que teníem a l'exemple \autoref{exemple:equacio-parametria-recta-2d}.
779 \end{claim}
781 \begin{exercise}Trobeu les equacions contínua i general de la recta que passa per $P = (2,-5)$ i segueix la direcció del vector director $\overrightarrow{v} = \overrightarrow{\left(-2,7\right)}$.
782 \end{exercise}
784 \begin{exercise}Donada la recta d'equació $5x-y+6=0$, trobeu les cooordenades de dos dels seus punts. A partir d'aquests, calculeu el seu vector director.
785 \end{exercise}
787 \subsubsection{Vector director a partir de l'equació general}
789 \begin{proposition}\label{prop:vector-director-a-partir-eq-implicita}Donada una recta en forma general, és a dir, $Ax+By+C=0$, el seu vector director és $\overrightarrow{v}=\overrightarrow{\left(-B,A\right)}$.
790 \end{proposition}
792 \begin{demonstration}Una recta genèrica $r$ que passa pel punt $P = (x_1, y_1)$ i que té com a vector director $\overrightarrow{v_r}(v_x, v_y)$ té l'equació contínua
793 \begin{equation*}
794 r \equiv \frac{x-x_1}{v_x} = \frac{y-y_1}{v_y}
795 \end{equation*}
796 Per tant, $v_y \cdot (x-x_1) = v_x \cdot (y-y_1)$. Aleshores, $v_y x - v_x y + (-v_y x_1 + v_x y_1) = 0$. Per la qual cosa, $A=v_y$, $B=-v_x$ i $C=-v_y x_1 + v_x y_1$. Per tant, el vector director és $\overrightarrow{(v_x, v_y)} = \overrightarrow{(-B, A)}$.
797 \end{demonstration}
799 \begin{proposition}Donada una recta $r \equiv Ax + By + C = 0$ en forma implícita, tenim que el vector $\overrightarrow{(A, B)}$ és perpendicular a la recta.
800 \end{proposition}
802 \begin{demonstration}El vector $(A,B)$ és perpendicular al vector $(-B,A)$ --- ja que el seu producte escalar és $0$. Per tant, el vector $(A,B)$ és un
803 vector perpendicular a la recta d'equació $Ax+By+D=0$.
804 \end{demonstration}
806 \begin{example}El vector director de la recta $5x-2y+1=0$ és $\overrightarrow{v}=\left(2,5\right)$.
807 \end{example}
809 \begin{exercise}Calculeu el vector director de les rectes següents:
810 \begin{enumerate}[label=\emph{\alph*})]
811 \item $4x-3y+1=0$
813 \item $-y+5=0$
814 \end{enumerate}
815 \end{exercise}
817 \begin{exercise}Donada la recta $x-5y+8=0$, trobeu:
818 \begin{enumerate}[label=\emph{\alph*})]
819 \item l'equació de la recta para\lgem{ela} que passa pel punt $(2,-7)$,
821 \item l'equació de la recta perpendicular que passa pel punt $(2,-7) $.
822 \end{enumerate}
823 \end{exercise}
825 \begin{claim}La \autoref{prop:vector-director-a-partir-eq-implicita}, serveix per a passar de l'equació general a l'equació contínua o bé a l'equació paramètrica: directament es pot obtenir el seu vector director $\overrightarrow{v_r}$. I després subtituïnt $x$ o $y$, podem trobar un punt seu.
826 \end{claim}
828 \begin{example}Obteniu l'equació contínua de la recta $s$ que té equació general $s \equiv 5x -9y -2 = 0$.
831 \bigskip
832 Per la \autoref{prop:vector-director-a-partir-eq-implicita}, tenim que el vector director de $s$ és $\overrightarrow{v_s} = \overrightarrow{(9,5)}$.
834 D'altra banda, trobarem un punt de $s$. Prendre'm $x = 0$, per exemple, amb el que obtenim $y = -2/9$. Per tant $(0,-2/9) \in s$.
836 Amb tot, tenim que l'equació contínua de $s$ serà:
837 \begin{equation*}
838 s \equiv \frac{x}{9} = \frac{y+\frac{2}{9}}{5}
839 \end{equation*}
840 \end{example}
842 \begin{exercise}Donada la recta $r \equiv 2x - 9y +5 = 0$, trobeu les equacions contínua, paramètrica i vectorial.
843 \end{exercise}
845 \begin{example}Trobeu el punt de tall de les rectes $r \equiv 2x -5y +10 = 0$ i $s \equiv \frac{x-2}{5} = \frac{y-3}{8}$.
848 \bigskip
849 Diem $P(a,b)$ al punt de tall de $r$ i $s$. Si $P \in r \cap s$, aleshores $P$ verifica les equacions de $r$ i $s$ simultàniament. Per tant, s'ha de verificar el sistema:
850 \begin{equation*}
851 \left\{\begin{aligned}
852 2a-5b + 10 = 0\\
853 \frac{a-2}{5} = \frac{b-3}{8}
854 \end{aligned}\right.
855 \end{equation*}
857 Aplicant el mètode de reducció multiplicant la segona equació per $40$ (vegeu \autoref{apendix:seccio:metodes-de-resolucio-sistemes}), tenim que
858 \begin{equation*}
859 \left\{\begin{aligned}
860 2a-5b &= - 10\\
861 8a -5b & = 1
862 \end{aligned}\right.
863 \end{equation*}
864 Per tant, $a = 3/2$ i $b = 13/5$. Llavors el punt de tall és $P(\frac{3}{2}, \frac{13}{5})$.
866 Noteu que no sempre dues rectes tendran punt de tall: quan aquestes siguin para\lgem{eles}, aleshores no existiran punts de tall. En aquest cas, el sistema no tendria solució. Vegeu l'apartat referent a la posició relativa de dues rectes (\autoref{subseccio:posicio-relativa-rectes-2d}).
867 \end{example}
869 \begin{exercise}Donades les rectes $r \equiv 5x - 2y + 8 = 0$ i $s \equiv \frac{x-2}{3} = \frac{y}{5}$, trobeu:
870 \begin{enumerate}[label=\emph{\alph*})]
871 \item dues rectes para\lgem{eles} a $r$
872 \item dues rectes para\lgem{eles} a $s$
873 \item una recta perpendicular a $s$ que passi per $(10,10)$
874 \item una recta perpendicular a $r$ que passi per $(0,0)$
875 \item el punt de tall de $r$ i $s$
876 \item el punt de tall de $r$ i la recta perpendicular a $s$ que passa per $(5,20)$
877 \end{enumerate}
878 \end{exercise}
880 \subsection{Equació explícita de la recta}
882 Si de l'equació general d'una recta \eqref{eq:equacio-general-recta-2d} aillam la $y$ ens queda una equació de la forma:%
883 \begin{equation}\label{eq:equacio-explícita}
884 y=mx+b,
885 \end{equation}%
886 amb $m$ i $b$ nombres reals. Aquesta equació es coneix amb el nom de \term{equació explícita de la recta}\index{equació!explícita!d'una recta}. S'anomena \term{pendent}\index{pendent d'una recta} al coeficient $m$ i \term{ordenada a l'origen}\index{ordenada a l'origen} al nombre $b$. La interpretació gràfica d'aquests dos paràmetres és la següent:
887 \begin{itemize}
888 \item La pendent de la recta és la inclinació d'aquesta:
889 \begin{itemize}
890 \item Si $m > 0$, aleshores la recta és \term{creixent}\index{recta!creixent} (quan els valors de $x$ creixen, els valors de $y$ creixen)
891 \item Si $m < 0$, aleshores la recta és \term{decreixent}\index{recta!decreixent} (quan les valors de $x$ creixen, els valors de $y$ decreixen)
892 \item Si $m=0$, aleshores la recta és \term{constant}\index{recta!constant}. Té una forma completament horitzontal.
893 \end{itemize}
894 D'altra banda, quan $\lvert m \rvert$ és major, la inclinació de la recta és major en el sentit que és més vertical. Per exemple, $y = 3x+2$ tendrà més inclinació que $y=x+2$, i $y=-5x+10$ tendrà més inclinació que $y = -2x+10$.
896 \item L'ordenada a l'origen $b$ és el valor que de l'eix de les $Y$ quan $x=0$. És a dir, l'ordenada a l'origen ens diu en quin punt talla la recta a l'eix $OY$. En altres paraules, $(0,b)$ és el punt de tall de la recta amb l'eix $OY$.
897 \end{itemize}
899 \begin{example}Representeu gràficament la recta $r \equiv y=-2x+3$ i trobeu els seus punts de tall amb els eixos.
901 Sabem que $r$ és decreixent perquè $-2 < 0$. I que passa per $(0,3)$. Per representar-la només ens fa falta un altre punt (una recta ve determinada per dos punts). Substituïm, per exemple, per $x = 2$: $y = -2 \cdot 2 +3 = -1$. Per tant, $(2,-1) \in r$. Aleshores, $r$ té la representació següent (\autoref{fig:equacio-explicita-recta-2d}):
903 \begin{figure}[h!]
904 \centering
905 % Generat amb geogebra. Modificat manualment
906 \definecolor{xdxdff}{rgb}{0.49019607843137253,0.49019607843137253,1.}
907 \definecolor{uuuuuu}{rgb}{0.26666666666666666,0.26666666666666666,0.26666666666666666}
908 \begin{tikzpicture}[line cap=round,line join=round,>=triangle 45,x=1.0cm,y=1.0cm]
909 \draw[->,color=black] (-1,0) -- (3,0);
910 \foreach \x in {-1,1,2}
911 \draw[shift={(\x,0)},color=black] (0pt,2pt) -- (0pt,-2pt) node[below] {\footnotesize $\x$};
912 \draw[->,color=black] (0,-2) -- (0,4);
913 \foreach \y in {-2,-1,1,2,3}
914 \draw[shift={(0,\y)},color=black] (2pt,0pt) -- (-2pt,0pt) node[left] {\footnotesize $\y$};
915 \draw[color=black] (0pt,-10pt) node[right] {\footnotesize $0$};
916 \clip(-1,-2) rectangle (3,4);
917 \draw [domain=-1:3] plot(\x,{(--3-2*\x)/1});
918 \draw (1,2.24) node[anchor=north west] {$r$};
919 \begin{scriptsize}
920 \draw [fill=uuuuuu] (0,3) circle (1.5pt);
921 \draw[color=uuuuuu] (0.14,3.28) node {$A$};
922 \draw [fill=xdxdff] (2,-1) circle (1.5pt);
923 \draw[color=xdxdff] (2.14,-0.72) node {$B$};
924 \end{scriptsize}
925 \end{tikzpicture}
926 \caption{Visualització de l'equació explícita d'una recta}
927 \label{fig:equacio-explicita-recta-2d}
928 \end{figure}
931 Només fa falta trobar el punt de tall amb l'eix de les abscises. En aquest cas, $y=0$. Per tant, $0=-2x+3$, el que implica que $x = 3/2$. Per tant, el punt $(\frac{3}{2},0)$ és el punt de la recta que està sobre l'eix $OX$.
932 \end{example}
934 \subsubsection{Càlcul de la pendent mitjançant dos punts}
936 Donats dos punts $A = (x_1,y_1)$ i $B = (x_2,y_2)$, per calcular la pendent de la recta $r \colon y = mx + b$ que els conté, podem substituir ambdós punts a l'equació de la recta i trobar $m$ i $b$. O bé, podem emprar la fórmula següent per a calcular la pendent de $r$:
937 \begin{equation*}
938 m = \frac{y_2 - y_1}{x_2-x_1}
939 \end{equation*}
940 i després substituir un dels punts a l'equació de la recta per a trobar $b$.
942 \begin{example} Trobeu l'equació explícita de la recta $r$ que passa pels punts $A = (2,3)$ i $B = (10,15)$.
944 Sigui $r \colon y = mx + n$ l'equació explícita de la recta $r$. Hem de determinar $m$ i $n$. Facem-ho de dues maneres:
945 \begin{itemize}
946 \item Substituïnt els dos punts a l'equació explícita.
948 Com que $A$ i $B$ són punts de la recta $r$, verifiquen la seva equació. Per tant,
949 \begin{equation*}
950 \left\{ \begin{aligned}
951 3 & = m \cdot 2 + n\\
952 15 & = m \cdot 10 + n
953 \end{aligned} \right.
954 \end{equation*}
955 Si resolem aquest sistema per $m$ i $n$, obtenim $m = 3/2$ i $n = 0$.
957 \item Emprant la fórmula de la pendent
959 Podem calcular la pendent amb la fórmula:
960 \begin{equation*}
961 m = \frac{15-3}{10-2} = \frac{12}{8} = \frac{3}{2}
962 \end{equation*}
963 Per tant, $r \colon y = \frac{3}{2} x + n$. Prenem un punt qualsevol de la recta, per exemple $A$, i substituïm-lo a aquesta equació: $3 = \frac{3}{2} \cdot 2 + n$. D'aquí tenim que $n=0$.
964 \end{itemize}
965 \end{example}
968 \subsubsection{Pendents de rectes para\lgem{eles} i perpendiculars}
970 Existeix una relació entre les pendents de rectes para\lgem{eles} o perpendiculars:
972 \begin{proposition}[relació entre les pendents de rectes para\lgem{eles} o perpendiculars] Siguin $r\colon y = m_r x + n_r$ i $s \colon y = m_s x + n_s$ dues rectes en el pla. Aleshores:
973 \begin{itemize}
974 \item $r \parallel s \iff \text{r, s tenen la mateix pendent} \iff m_r = m_s$
976 \item $r \bot s \iff m_{r}=-\frac{1}{m_s}$
977 \end{itemize}
978 \end{proposition}
980 Aquest teorema no es podrà generalitzar a la geometria a l'espai.
982 \begin{example}Si el pendent d'una recta donada val $-5$, la pendent de qualsevol recta para\lgem{ela} val tamb\'{e} $-5$, i la de qualsevol recta perpendicular val $1/5$.
983 \end{example}
985 \begin{exercise}Donada la recta $x+5y-3=0$, calculeu la seva pendent, la de una recta para\lgem{ela} i la de una recta perpendicular.
986 \end{exercise}
988 \subsection{Equació de la recta determinada per dos punts}
990 \begin{proposition}[equació de la recta determinada per dos punts donats]Donats dos punts coneguts $A(x_{1},y_{1})$ i $B(x_{2},y_{2})$, si volem conèixer la recta que determinen, podem emprar la fórmula següent:
991 \begin{equation*}
992 \frac{x-x_{1}}{x_{2}-x_{1}}=\frac{y-y_{1}}{y_{2}-y_{1}},
993 \end{equation*}
994 que ens dóna l'equació contínua de la recta.
995 \end{proposition}
997 Amb aquesta proposició, ens evitam haver de cercar el vector director i plantejar una equació.
999 \begin{example}L'equació de la recta que passa pels punts $A = \left( 0,-2\right) $ i $B = \left( -4,1\right)$ es la següent:%
1000 \begin{equation*}
1001 \frac{x}{-4}=\frac{y+2}{3}
1002 \end{equation*}
1003 \end{example}
1005 \begin{exercise}Calculeu l'equació de la recta que passa pels punts $A = \left( 3,-5\right)$ i $B = \left( -1,7\right)$.
1006 \end{exercise}
1008 \subsection{Exemples de càlcul d'equacions de rectes}
1010 \begin{example}Trobeu la recta que passa pels punts $A = (5,9)$ i $B = (-10,8)$.
1012 Ho farem de diverses maneres:
1013 \begin{itemize}
1014 \item Calculant el vector director i amb un punt:
1016 El vector director pot ser $\overrightarrow{v} = \overrightarrow{(-10-5, 8-9)} = \overrightarrow{(-15,-1)}$. Qualsevol múltiple seu també és vector diretor de la recta. Per tant, triarem $\overrightarrow{v} = \overrightarrow{(15,1)}$ per evitar els signes.
1018 D'aquí podem obtenir diverses equacions de la recta fàcilment:
1019 \begin{itemize}
1020 \item L'equació vectorial: $\overrightarrow{OX} = \overrightarrow{(5,9)} + \lambda \cdot \overrightarrow{(15,1)}$
1021 \item L'equació paramètrica: $r \colon \left\{
1022 \begin{aligned}
1023 x & = 5 + 15 \lambda\\
1024 y & = 9 + \lambda
1025 \end{aligned}\right.$
1026 \item L'equació contínua: $r \colon \frac{x-5}{15} = y-9$
1027 \end{itemize}
1028 \item Trobant la pendent:
1030 Amb la fórmula, $m = \frac{8-9}{-10-5} = 1/15$. Per tant $r \colon y = 1/15 x + n$. Substituïnt, per exemple, $A$ a l'equació de la recta, tenim que $9 = 1/15 \cdot 5 + n$. Pel que $n = 26/3$. Per tant, $r \colon y = 1/15 x + 26/3$.
1031 \item A partir de la contínua o a partir de la explícita, podem trobar l'equació general\footnote{No és recomanable fer-ho amb un sistema d'equacions substituïnt els punts.}.
1033 \end{itemize}
1034 \end{example}
1036 \begin{example}Trobeu totes les equacions de la recta que passa pel punt $P = \left(3,-2\right)$ i que té per vector director $\overrightarrow{v} = \overrightarrow{\left(1,-4\right)}$.
1038 \begin{itemize}
1039 \item L'equació paramètrica és: $r:\left\{
1040 \begin{aligned}
1041 x & =3+\lambda \\
1042 y & =-2-4\lambda%
1043 \end{aligned}%
1044 \right. $
1046 \item L'equació contínua és $r:x-3=\frac{y+2}{-4}$
1048 \item L'equació general és $-4\cdot \left( x-3\right) = y+2$, és a dir, $-4x-y+10=0$
1049 \item Si aïllam la $y$ tenim: $y=-4x+10$. Aleshores, la pendent d'aquesta recta és $-4$.
1050 \end{itemize}
1051 \end{example}
1054 \subsection{Posició relativa entre dues rectes}\label{subseccio:posicio-relativa-rectes-2d}
1056 \begin{proposition}[posició relativa entre dues rectes]Dues rectes al pla cartesià poden ser (vegeu \autoref{fig:posicio-relativa-recta-2d}):
1057 \begin{itemize}
1058 \item \term{secants}\index{rectes!secants}, és a dir, que es tallen a un punt
1059 \item \term{para\lgem{eles}}\index{rectes!para\lgem{eles}}. Per tant, no es tallen a cap punt.
1060 \item \term{coincidents}\index{rectes!coincidents}, és a dir, són la mateixa recta.
1061 \end{itemize}
1063 Cadascuna d'aquestes posicions s'anomenen la \term{posició relativa} entre les dues rectes\index{posició relativa!entre dues rectes}.
1065 \begin{figure}[h!]
1066 \centering
1067 % Generat amb geogebra. Modificat manualment
1068 \definecolor{qqqqff}{rgb}{0.,0.,1.}
1069 \definecolor{ffqqqq}{rgb}{1.,0.,0.}
1070 \begin{tikzpicture}[line cap=round,line join=round,>=triangle 45,x=1.0cm,y=1.0cm,scale=.75]
1071 \begin{scope}[shift={(-2,0)}]
1072 \clip(0.,0.) rectangle (18.,6.);
1073 \draw [color=ffqqqq,domain=0.:18.] plot(\x,{(-0.--2.52*\x)/2.7});
1074 \draw [color=qqqqff,domain=0.:18.] plot(\x,{(-4.158--2.52*\x)/2.7});
1075 \draw [color=ffqqqq,domain=0.:18.] plot(\x,{(-17.3304--2.52*\x)/2.7});
1076 \draw [color=qqqqff,domain=0.:18.] plot(\x,{(-29.9192--2.84*\x)/0.22});
1077 \draw [color=ffqqqq,domain=0.:18.] plot(\x,{(-31.4244--2.52*\x)/2.7});
1078 \draw (3.54,4.3) node[anchor=north west] {$r$};
1079 \draw (4.86,2.74) node[anchor=north west] {$s$};
1080 \draw (9.08,3.26) node[anchor=north west] {$r$};
1081 \draw (11.04,3.02) node[anchor=north west] {$s$};
1082 \draw (15.44,4.36) node[anchor=north west] {$r \text{ i } s$};
1083 \begin{scriptsize}
1084 \draw [fill=qqqqff] (10.82,3.68) circle (1.5pt);
1085 \end{scriptsize}
1086 \end{scope}
1087 \end{tikzpicture}
1088 \caption{Les diferents posicions relatives possibles entre dues rectes}
1089 \label{fig:posicio-relativa-recta-2d}
1090 \end{figure}
1091 \end{proposition}
1093 La proposició següent ens diu quan dues rectes són secants, para\lgem{eles} o coincidents.
1095 \begin{proposition}[criteri de posició relativa] Per a dues rectes $r$ i $s$ en el pla es compleix:
1096 \begin{itemize}
1097 \item Si $r$ i $s$ tenen diferent pendent, aleshores són secants
1098 \item Si $r$ i $s$ tenen la mateixa pendent, aleshores són para\lgem{eles} o coincidents. En aquest cas:
1099 \begin{itemize}
1100 \item Si $r$ i $m$ tenen diferents ordenades a l'origen, llavors són para\lgem{eles}
1101 \item Si $r$ i $m$ tenen la mateixa ordenada a l'origen, llavors són coincidents
1102 \end{itemize}
1103 \end{itemize}
1104 \end{proposition}
1106 Aquest criteri usant vectors directors és el següent:
1108 \begin{proposition}[criteri de posició relativa] Per a dues rectes $r$ i $s$ en el pla es compleix:
1109 \begin{itemize}
1110 \item Si $r$ i $s$ no tenen vectors directors proporcionals, aleshores són secants
1111 \item Si $r$ i $s$ tenen vectors directors proporcionals\footnote{Hem de notar aquí que si dues rectes tenen vectors directors que són proporcionals, sempre podem prendre el mateix vector director a les dues rectes. Per exemple $r \colon \frac{x-1}{2} = \frac{y-3}{4}$ té com a vector director $\overrightarrow{(2,4)}$ i $s \colon \frac{x-5}{1} = \frac{y-8}{2}$ té com a vector director $\overrightarrow{(1,2)}$, els quals són proporcionals. Per tant, podríem prendre com a equació contínua de la recta $r$ l'equació $\frac{x-1}{1} = \frac{y-3}{2}$.}, aleshores són para\lgem{eles} o coincidents. En aquest cas, si $r$ i $s$ passen per un punt en comú, aleshores són coincidents. Altrament, són para\lgem{eles}
1112 \end{itemize}
1113 \end{proposition}
1115 \begin{exercise}Calculeu la posició relativa de les rectes:
1116 \begin{enumerate}[label=\emph{\alph*})]
1117 \item $r \colon y = 2x -5$ i $s \colon y = 3x+4$
1118 \item $r \colon 2x + 5y -4 = 0$ i $s \colon \left\{
1119 \begin{aligned}
1120 x & = 2 + 5 k\\
1121 y & = -3 + 7k
1122 \end{aligned}\right.$
1123 \item $r \colon \frac{x-1}{1} = \frac{y + 2}{2}$ i $s \colon x - 2y + 5 = 0$
1124 \end{enumerate}
1125 \end{exercise}
1127 \subsubsection{Càlcul dels punts de tall}
1129 Fins ara hem vist quina és la posició relativa entre dues rectes. Això vol dir que podem saber si dues rectes es tallen però {\em encara} no sabem com trobar el seu \term{punt de tall}\index{punt!de tall}, és a dir, el punt d'intersecció entre les dues rectes.
1131 Per a trobar el punt de tall entre dues rectes, només hem de notar que si $P$ pertany a les dues rectes, aleshores ha de complir ambdues equacions. D'aquesya manera obtindrem un sistema d'equacions de dues incògnites i dues equacions, que podem resoldre fàcilment per reducció, igualació o substitució (vegeu \autoref{apendix:seccio:metodes-de-resolucio-sistemes}).
1133 \begin{example}\label{exemple:calcul-punt-tall-rectes}Calculeu els punts de tall, si existeixen, entre les rectes $r:x-3y+1=0$ i $s:-4x+7y=0$.
1135 Hem de resoldre el sistema%
1136 \begin{equation*}
1137 \left\{
1138 \begin{aligned}
1139 x-3y+1 & = 0 \\
1140 -4x+7y & = 0%
1141 \end{aligned}%
1142 \right.
1143 \end{equation*}%
1145 Si aplicam el mètode de substitució (vegeu \autoref{apendix:seccio:metodes-de-resolucio-sistemes}), aïllant la $x$ de la primera equació: $x=3y-1$, tenim que $-4\left( 3y-1\right) +7y=0$, és a dir, $y=\frac{4}{5}$. Per tant, si substituïm a la primera equació: $x-3\cdot \frac{4}{5}+1=0$, és a dir, $x=\frac{7}{5}$.
1147 Aleshores, el punt de tall entre ambdues rectes \'{e}s el punt $\left( \frac{7}{5},\frac{4}{5}\right)$.
1148 \end{example}
1150 \begin{exercise}Calculeu els punts de tall, si existeixen, entre les rectes $r:2x-3y+1=0$ i $s:-4x+7y=0$.
1151 \end{exercise}
1153 \begin{exercise}Calculeu els punts de tall, si existeixen, entre les rectes
1154 \begin{enumerate}[label=\emph{\alph*})]
1155 \item $r \colon y = 2x +4$ i $s \colon y = 7x-5$
1156 \item $r \colon y = 2x +4$ i $s \colon x -y -5 = 0$
1157 \item $r \colon 2x - y - 2 = 0$ i $s \colon \frac{x-1}{2} = \frac{x+5}{2}$
1158 \item $r \colon 2x - 5y - 4 = 0$ i $s \colon x - y - 2 = 0$
1159 \item $r \colon \left\{
1160 \begin{aligned}
1161 x & = 2 - 5 \lambda\\
1162 y & = 1 + \lambda
1163 \end{aligned}\right.$ i $s \colon \left\{
1164 \begin{aligned}
1165 x & = 2 \lambda\\
1166 y & = 3 - 2\lambda
1167 \end{aligned}\right.$
1168 \item $r \colon \left\{
1169 \begin{aligned}
1170 x & = 1 + 2 \lambda\\
1171 y & = 2 - 3\lambda
1172 \end{aligned}\right.$ i $s \colon \frac{x-1}{2} = \frac{y}{3}$
1173 \end{enumerate}
1174 \end{exercise}
1177 \begin{exercise}Calculeu els punts de tall, si existeixen, entre les rectes $r:2x-3y+1=0$ i $s:-4x+\alpha y=0$ en funció del paràmetre $\alpha$.
1178 \end{exercise}
1180 \begin{remark}En general, una manera senzilla de trobar els punts de tall de dues rectes és, en primer lloc, passar-les a l'equació explícita i resoldre el sistema per igualació (vegeu \autoref{apendix:seccio:metodes-de-resolucio-sistemes}).
1182 En l'exemple anterior (\autoref{exemple:calcul-punt-tall-rectes}), tenim que les rectes $r$ i $s$ es transformen en:
1183 \begin{equation*}
1184 \left\{
1185 \begin{aligned}
1186 r \colon y & = \frac{-1-x}{-3} \\
1187 s \colon y & = \frac{4x}{7}%
1188 \end{aligned}%
1189 \right.
1190 \end{equation*}%
1191 Per tant, igualant les dues expressions:
1192 \begin{equation*}
1193 \frac{-1-x}{-3} = \frac{4x}{7},
1194 \end{equation*}
1195 el que implica que $-7 - 7x = -12x \Rightarrow x = 7/5$. Subtituint a qualsevol de les equacions anteriors i operant, obtenim que $y=4/5$.
1196 \end{remark}
1198 \newpage
1199 \section{Exercicis proposats}
1201 \subsection*{Punts i vectors}
1203 \begin{exercise}Els punts $A\left( 3,-2\right) ,$ $B\left( 5,0\right) $ i $C\left(-1,-3\right)$ són vèrtexs d'un para\lgem{elogram}. Calculeu la posició de l'altre vèrtex $D$. I trobeu el seu perímetre. Calculeu analíticament $\overrightarrow{AD}$ i el seu mòdul.
1204 \end{exercise}
1206 \begin{exercise}\label{exer:espuig-punts-1}Donats els punts $A(3,1)$, $B(-5,1)$, $C(-4,-2)$ i $D(0,-3)$, calculeu, analíticament, les components i el mòdul dels vectors:
1207 \begin{multicols}{4}
1208 \begin{enumerate}[label=\emph{\alph*})]
1209 \item $\overrightarrow{AB}$
1210 \item $\overrightarrow{BA}$
1211 \item $\overrightarrow{BC}$
1212 \item $\overrightarrow{CB}$
1213 \item $\overrightarrow{CD}$
1214 \item $\overrightarrow{AD}$
1215 \item $\overrightarrow{BD}$
1216 \item $\overrightarrow{CA}$
1217 \end{enumerate}
1218 \end{multicols}
1219 \end{exercise}
1221 \begin{exercise}\label{exer:espuig-punts-2}Calculeu les coordenades de $B$ si sabem que el vector $\overrightarrow{AB}=\overrightarrow{(3,-4)}$ i $A = (2,-5)$; trobeu les coordenades del punt $C$ si sabem que $D(-5,2)$ i $\overrightarrow{CD} = \overrightarrow{(-5,1)}$.
1222 \end{exercise}
1224 \begin{exercise}\label{exer:espuig-punts-3}Donats els punts $A(3,0)$, $B(2,3)$, $C(-2,1)$ i $D(7,2)$, esbrineu si els vectors següents són equipolents:
1225 \begin{multicols}{3}
1226 \begin{enumerate}[label=\emph{\alph*})]
1227 \item $\overrightarrow{AB}$, $\overrightarrow{CD}$
1228 \item $\overrightarrow{AC}$, $\overrightarrow{DB}$
1229 \item $\overrightarrow{BC}$, $\overrightarrow{DA}$
1230 \end{enumerate}
1231 \end{multicols}
1232 \end{exercise}
1234 \begin{exercise}\label{exer:espuig-punts-4}Les coordenades del punt $A$ són el doble de les del punt $B$. Sabent que $\overrightarrow{AB}=\overrightarrow{(-2,5)}$, calculeu les coordenades dels punts $A$ i $B$.
1235 \end{exercise}
1237 \begin{exercise}\label{exer:espuig-punts-5}Donats els vectors $\overrightarrow{u} = \overrightarrow{(7,-4)}$, $\overrightarrow{v} = \overrightarrow{(-5,-2)}$ i $\overrightarrow{w} = \overrightarrow{(-6,0)}$, calculeu:
1238 \begin{multicols}{3}
1239 \begin{enumerate}[label=\emph{\alph*})]
1240 \item $5\overrightarrow{u} - 2\overrightarrow{v}$
1241 \item $3\overrightarrow{u}-\frac{2}{3}\overrightarrow{w}$
1242 \item $-\overrightarrow{w}-3(\overrightarrow{u} - \overrightarrow{v})$
1243 \item $-3\overrightarrow{v}+5\overrightarrow{u}+\overrightarrow{w}$
1244 \end{enumerate}
1245 \end{multicols}
1246 i calculeu-ne els seus mòduls.
1247 \end{exercise}
1249 \begin{exercise}\label{exer:espuig-punts-6}Trobeu quatre vectors para\lgem{els} i tres perpendiculars al vector $\overrightarrow{u}(-5,4)$. En podeu trobar d'unitaris?
1250 \end{exercise}
1252 \begin{exercise}\label{exer:espuig-punts-7}Calculeu l'angle que formen els vectors següents i extreis conclusions sobre la seva direcció i sentit:
1253 \begin{multicols}{2}
1254 \begin{enumerate}[label=\emph{\alph*})]
1255 \item $\overrightarrow{u}(5,2)$ i $\overrightarrow{v}(10,4)$
1256 \item $\overrightarrow{u}(-3,15)$ i $\overrightarrow{v}(2,-10)$
1257 \item $\overrightarrow{u}(3,4)$ i $\overrightarrow{v}(-50,40)$
1258 \item $\overrightarrow{u}(-3,4)$ i $\overrightarrow{v}(-2,10)$
1259 \end{enumerate}
1260 \end{multicols}
1261 \end{exercise}
1263 \begin{exercise}\label{exer:espuig-punts-8}Donats els punts $A(2,3)$ i $B(-5,4)$, trobeu els punts que divideixen el segment $AB$ en dues parts iguals, en tres parts iguals i en quatre parts iguals.
1264 \end{exercise}
1266 \subsection*{Rectes}
1268 \begin{exercise}\label{exer:espuig-1}Trobeu la recta determinada per:
1269 \begin{enumerate}[label=\emph{\alph*})]
1270 \item Els punts $A(-2,-1)$ i $B(2,4)$
1271 \item El punt $P(1,-4)$ i el vector director $\overrightarrow{v}(5,-3)$
1272 \item El punt $P(1,-2)$ i l'angle que forma amb l'eix $OX$ és $\alpha = 135 \degree$
1273 \item El punt $P(1,-1)$ i la pendent $m=2$
1274 \item La pendent $m=2$ i l'ordenada a l'origen $-5$
1275 \end{enumerate}
1276 \end{exercise}
1278 \begin{exercise}\label{exer:espuig-2}Donada la recta $r$ que passa pel punt $P(-5,-3)$ i que té vector director $\overrightarrow{v}(12,8)$:
1279 \begin{enumerate}[label=\emph{\alph*})]
1280 \item Trobeu les equacions vectorial i paramètrica de la recta
1281 \item Trobeu tres punts que pertanyin a $r$
1282 \item Esbrineu si els punts $(-11,-7)$ i $(2,-1)$ pertanyen a la recta.
1283 \end{enumerate}
1284 \end{exercise}
1286 \begin{exercise}\label{exer:espuig-3}Donada la recta $s$ que passa pel punt $P(4,-3)$ i que té vector director $\overrightarrow{v}(2,-7)$:
1287 \begin{enumerate}[label=\emph{\alph*})]
1288 \item Trobeu l'equació contínua de la recta
1289 \item Trobeu tres punts que pertanyin a $s$
1290 \item Esbrineu si els punts $(8,-7)$ i $(0,11)$ pertanyen a la recta.
1291 \end{enumerate}
1292 \end{exercise}
1294 \begin{exercise}\label{exer:espuig-4}Donada la recta $s$ que passa pel punt $P(-2,3)$ i que té vector director $\overrightarrow{v}(-1,4)$:
1295 \begin{enumerate}[label=\emph{\alph*})]
1296 \item Trobeu l'equació general de la recta
1297 \item Trobeu tres punts que pertanyin a $s$
1298 \item Esbrineu si els punts $(-5,15)$ i $(4,3)$ pertanyen a la recta.
1299 \end{enumerate}
1300 \end{exercise}
1302 \begin{exercise}Trobeu l'equació general de la recta que passa pels punts $A(2,3)$ i $B(-3,-2)$.
1303 \end{exercise}
1305 \begin{exercise}Trobeu l'equació explícita de la recta que:
1306 \begin{enumerate}[label=\emph{\alph*})]
1307 \item passa pel punt $A(-3,-1)$ i té pendent $m=-2$
1308 \item passa pels punts $A(-4,-2)$ i $B(-3,-1)$
1309 \item passa pel punt $A(-5,2)$ i té ordenada a l'origen $-4$.
1310 \end{enumerate}
1311 \end{exercise}
1313 \begin{exercise}\label{exer:espuig-5}Trobeu un punt i el vector director de cadascuna d'aquestes rectes:
1314 \begin{multicols}{2}
1315 \begin{enumerate}[label=\emph{\alph*})]
1316 \item $\overrightarrow{(x,y)} = \overrightarrow{(-5,-2)} + k \overrightarrow{(-9,7)}$
1317 \item $\frac{x-15}{-1} = \frac{y+2}{6}$
1318 \item $4x-10y +6=0$
1319 \item $y = -5x+10$
1320 \item $\left\{\begin{aligned}
1321 x & = 2 - 8k\\
1322 y & = 3 + 6k
1323 \end{aligned}\right.$
1324 \item $x-5 = \frac{y+4}{12}$
1325 \item $x+3y+1 = 0$
1326 \item $y = -\frac{3}{2}x-2$
1327 \item $\left\{\begin{aligned}
1328 x & = -7 - k\\
1329 y & = 11 + k
1330 \end{aligned}\right.$
1331 \item $\frac{-x-5}{-1} = \frac{y+1}{2}$
1332 \item $-2x -y-12=0$
1333 \item $y=x+4$
1334 \end{enumerate}
1335 \end{multicols}
1336 \end{exercise}
1338 \begin{exercise}\label{exer:espuig-6}Indiqueu si els punts següents estan alineats:
1339 \begin{enumerate}[label=\emph{\alph*})]
1340 \item $A(-1,1)$, $B(2,1)$ i $C(8,5)$
1341 \item $D(-1,2)$, $E(0,0)$ i $F(2,-2)$
1342 \end{enumerate}
1344 En cas negatiu, obteniu-ne un que hi estigui.
1345 \end{exercise}
1347 \begin{exercise}\label{exer:espuig-7}Esbrineu la posició relativa de les rectes següents:
1348 \begin{multicols}{2}
1349 \begin{enumerate}[label=\emph{\alph*})]
1350 \item $r \colon 6x-15y+1=0$ i $s\colon -10x+25y+1=0$
1351 \item $r \colon 2x-10y+8 = 0$ i $s \colon x + 5y +4=0$
1352 \item $r \colon y=2x +3$ i $s \colon y=2x+1$
1353 \item $r \colon \frac{x-1}{1} = \frac{y-5}{4}$ i $s\colon \frac{x-2}{-3} = \frac{y+4}{-12}$
1354 \item $r \colon 2x + 6y +4=0$ i $s \colon -3x-9y -6 = 0$
1355 \columnbreak
1356 % https://tex.stackexchange.com/questions/8683/how-do-i-force-a-column-break-in-a-multi-column-page
1357 \item $r \colon y = x+1$ i $s \colon y=-x+1$
1358 \item $r \colon y = 3x+\frac{1}{2}$ i $s \colon 6x-2y +1 =0$
1359 \item $r \colon \frac{x-1}{1} = \frac{y-5}{4}$ i $s \colon \left\{\begin{aligned}
1360 x & = -10 - k\\
1361 y & = 2 + k
1362 \end{aligned}\right.$
1363 \item $r \colon \frac{x-1}{1} = \frac{y-5}{4}$ i $s\colon 2x-y +5=0$
1364 \end{enumerate}
1365 \end{multicols}
1366 \end{exercise}
1368 \begin{exercise}\label{exer:espuig-8}Trobeu el punt d'intersecció de les rectes secants de l'exercici anterior.
1369 \end{exercise}
1371 \begin{exercise}\label{exer:espuig-9}Trobeu la recta para\lgem{ela} a la recta $r$ que passa pel punt $P$ en els casos següents:
1372 \begin{multicols}{2}
1373 \begin{enumerate}[label=\emph{\alph*})]
1374 \item $r \colon 4x-5y +3=0$, $P(-3,5)$
1375 \item $r \colon \frac{x+5}{-2} = \frac{y+1}{-3}$, $P(4,-10)$
1376 \item $r \colon y = -5x + 3$, $P(-1,1)$
1377 \end{enumerate}
1378 \end{multicols}
1379 \end{exercise}
1381 \begin{exercise}\label{exer:espuig-10}Indiqueu si els parells de rectes següents són perpendiculars:
1382 \begin{enumerate}[label=\emph{\alph*})]
1383 \item $r \colon x-5y +1 = 0$, $s\colon 10x+2y-3 =0$
1384 \item $r \colon y=2x+4$, $s\colon y=-\frac{1}{2} x + 8$
1385 \item $r \colon \frac{x-1}{1}=\frac{y-5}{4}$, $s \colon \frac{x-2}{4} = \frac{y+4}{-1}$
1386 \item $r \colon x+y+4=0$, $s \colon -x-y-1 =0$
1387 \item $r \colon y=x+1$, $s \colon y = -x-2$
1388 \item $r \colon \frac{x-1}{3} = \frac{y-5}{7}$, $s \colon 7x+3y+5=0$
1389 \end{enumerate}
1390 \end{exercise}
1392 \begin{exercise}\label{exer:espuig-11}Trobeu l'equació de la recta perpendicular a la recta $r$ que passa pel punt $P$:
1393 \begin{multicols}{2}
1394 \begin{enumerate}[label=\emph{\alph*})]
1395 \item $r \colon 4x-5y+3 =0$, $P(-3,5)$
1396 \item $r \colon \frac{x+5}{-2} = \frac{y+1}{-3}$, $P(4,-10)$
1397 \item $r \colon y=-5x+3$, $P(-1,1)$
1398 \item $r \colon \left\{ \begin{aligned}
1399 x & = 3 + 5 \lambda\\
1400 y & = -2 - 6 \lambda
1401 \end{aligned}\right.$, $P(1,1)$
1402 \end{enumerate}
1403 \end{multicols}
1404 \end{exercise}
1406 \begin{exercise}\label{exer:espuig-12}Calculeu el valor de $a$ per a què les rectes $r \equiv x+ay+4=0$ i $s \equiv 3x+4y-1=0$:
1407 \begin{multicols}{2}
1408 \begin{enumerate}[label=\emph{\alph*})]
1409 \item Siguin para\lgem{eles}
1410 \item Siguin perpendiculars
1411 \item Formin un angle de 180 graus
1412 \item Formin un angle de 60 graus
1413 \end{enumerate}
1414 \end{multicols}
1415 \end{exercise}
1417 \begin{exercise}\label{exer:espuig-13}Calculeu l'angle que formen les rectes següents:
1418 \begin{enumerate}[label=\emph{\alph*})]
1419 \item $r \colon x-y+1=0$, $s\colon 7x+2y-3=0$
1420 \item $r \colon y=-3x+4$, $s \colon y=-x+1$
1421 \item $r \colon 2x+y+4=0$, $s \colon -3x+2y-1=0$
1422 \item $r \colon \frac{x-1}{2} = \frac{y-5}{3}$, $s\colon \frac{x-2}{3} = \frac{y+4}{-2}$
1423 \end{enumerate}
1424 \end{exercise}
1426 \begin{exercise}Donada la recta $r\colon 2x-3y+1=0$, calculeu:
1427 \begin{enumerate}[label=\emph{\alph*})]
1428 \item el seu vector director i un vector perpendicular,
1430 \item l'equació de la recta que passa pel punt $A\left( 3,-5\right)$ i que és perpendicular a la recta $r$,
1432 \item el punt simètric del punt $A$ respecte de la recta $r$.
1433 \end{enumerate}
1434 \end{exercise}
1436 \begin{exercise}Calculeu la pendent i l'ordenada a l'origen de les rectes següents:
1437 \begin{enumerate}[label=\emph{\alph*})]
1438 \item $x+3y=4$
1439 \item $4y+5=-x$
1440 \item $2x - 7y = 0$
1441 \item $-8y = 8$
1442 \end{enumerate}
1443 \end{exercise}
1446 \begin{exercise}Calculeu l'equació de la recta que passa pels punts $A\left( -1,0\right)$ i $B\left( -4,-1\right) $. Calculeu el seu vector director i altres dos punts més de la recta.
1447 \end{exercise}
1449 \begin{exercise}Calculeu totes les equacions de la recta que passa pel punt $A\left(3,-5\right)$ i que segueix la direcció $\overrightarrow{v}(-1,7)$. Calculeu la seva pendent.
1450 \end{exercise}
1452 \begin{exercise}Calculeu totes les equacions de la recta que passa pels punts $A(2,3)$ i $B(-5,1)$.
1453 \end{exercise}
1456 \begin{exercise}Donada la recta $y=2x+8$, calculeu:
1457 \begin{enumerate}[label=\emph{\alph*})]
1458 \item el seu vector director,
1460 \item l'equaci\'{o} de la recta paral\textperiodcentered lela que passa
1461 pel punt $\left( 0,-8\right) $,
1463 \item un vector perpendicular a la recta,
1465 \item l'equaci\'{o} de la recta perpendicular que passa pel punt $\left( 0,-8\right)$.
1466 \end{enumerate}
1467 \end{exercise}
1470 \begin{exercise}Calculeu els punts de tall dels parells de rectes següents:
1471 \begin{enumerate}[label=\emph{\alph*})]
1472 \item $r \colon \frac{x-3}{2} = \frac{y+1}{4}$ i $s \colon 3x - 5y + 2 = 0$
1473 \item $r \colon y = 6x - 10$ i $s \colon 9x - 3y + 27 = 0$
1474 \item $r \colon \left\{\begin{aligned}
1475 x & = 3 + 2 \lambda\\
1476 y & = -1 + 10 \lambda
1477 \end{aligned} \right.$ i $s \colon y = -x + 2$
1478 \item $r \colon \left\{\begin{aligned}
1479 x & = 3 + 2 \lambda\\
1480 y & = -1 + 10 \lambda
1481 \end{aligned} \right.$ i $s \colon \left\{\begin{aligned}
1482 x & = - 5 \lambda\\
1483 y & = 2 - 6 \lambda
1484 \end{aligned} \right.$
1485 \item $r \colon \frac{x-3}{2} = \frac{y+1}{4}$ i $s \colon \frac{x}{10} = \frac{y+8}{-1}$
1486 \item $r \colon 3x - 2y + 6 = 0$ i $s \colon -8x + 7y + 2 = 0$
1487 \item $r \colon y = 4x - 2$ i $s \colon y = 10x - 8$
1488 \item $r \colon y = 4x -2$ i $s \colon y= 4x-10$
1489 \item $r \colon \left\{\begin{aligned}
1490 x & = 3 + 2 \lambda\\
1491 y & = -1 + 10 \lambda
1492 \end{aligned} \right.$ i $s \colon \frac{x-2}{3} = \frac{y+2}{3}$
1493 \item $r \colon \left\{\begin{aligned}
1494 x & = 3 + 2 \lambda\\
1495 y & = -1 + 10 \lambda
1496 \end{aligned} \right.$ i $s \colon 10x - 2y + 3 = 0$
1497 \item $r \colon \frac{x-2}{3} = \frac{y+10}{-2}$ i $s \colon y = 10x -12$
1498 \end{enumerate}
1499 \end{exercise}
1501 \chapter{Geometria de l'espai}
1503 \section{Sistema de coordenades espacials}
1505 De forma anàloga al pla cartesià, a l'espai tridimensional tenim tres eixos de coordenades, $x$, $y$ i $z$, els quals són perpendiculars i parteixen d'un punt, anomenat \term{origen de coordenades}\index{origen!de coordenades}. La forma més usual de representar aquests eixos dibuixant l'eix $x$ en la direccció dreta-esquerre, l'eix $y$ en la direcció davant-darrera i l'eix $z$ en la direcció dalt-baix (\autoref{fig:sistema-de-coordenades-3d})
1507 \begin{figure}[h!]
1508 \centering
1509 % Generat amb TikZ
1510 \begin{tikzpicture}[thick]
1511 \draw[->] (0,0,0) -- (3,0,0);
1512 \draw[->] (0,0,0) -- (0,3,0);
1513 \draw[->] (0,0,0) -- (0,0,3);
1514 \draw (3,0,0) node[anchor=west] {$x$};
1515 \draw (0,3,0) node[anchor=south] {$z$};
1516 \draw (0,0,3) node[anchor=north east] {$y$};
1517 \draw (0,0,0) node[anchor=north] {$O$};
1518 \end{tikzpicture}
1519 \caption{Representació usual del sistema de coordenades cartesianes}
1520 \label{fig:sistema-de-coordenades-3d}
1521 \end{figure}
1523 En aquest sistema de coordenades\index{sistema de coordenades}\index{espai cartesià}, un punt\index{punt!a l'espai} qualsevol $P$ ve localitzat per les projeccions als eixos de coordenades. De la mateixa manera que el cas bidimensional, direm que $P$ té \term{coordenades}\index{coodenades} $(x, y, z)$ i el podrem denotar com $P(x,y,z)$. Per exemple, el punt de coordenades $(1,2,3)$ correspon al punt $A$ de la figura següent (\autoref{fig:punt-coordenades-3d}).
1525 \begin{figure}[h!]
1526 \centering
1527 % Generat amb TikZ
1528 \begin{tikzpicture}[thick]
1529 \draw[->] (0,0,0) -- (2,0,0);
1530 \draw[->] (0,0,0) -- (0,4,0);
1531 \draw[->] (0,0,0) -- (0,0,3);
1532 \draw (2,0,0) node[anchor=west] {$x$};
1533 \draw (0,4,0) node[anchor=south] {$z$};
1534 \draw (0,0,3) node[anchor=north east] {$y$};
1535 \draw[dashed] (1,0,2) -- (1,3,2);
1536 \draw[loosely dotted] (1,0,2) -- (0,0,2);
1537 \draw[loosely dashed] (1,0,2) -- (1,0,0);
1538 \fill[color=blue] (1,3,2) circle (2pt);
1539 \draw (1,3,2) node[anchor=west] {$A(1,2,3)$};
1540 \end{tikzpicture}
1541 \caption{Representació del punt $A(1,2,3)$}
1542 \label{fig:punt-coordenades-3d}
1543 \end{figure}
1545 \begin{exercise}Representeu gràficament en els eixos de coordenades els punts $A\left(3,-2,4\right)$ i $B\left( 5,0,-2\right)$.
1546 \end{exercise}
1548 \section{Vectors}
1550 Les definicions relatives a vectors que hem estudiat a l'apartat de Geometria del pla (vector fix, vector lliure, extrems d'un vector, etc.; vegeu \autoref{seccio:geometria-al-pla}) poden adaptar-se fàcilment a l'espai només afegint una altra coordenada als vectors. A l'igual que al pla, suposarem que tots els vectors són lliures\index{vector!lliure}.
1552 \begin{example}Són vectors el següents:%
1553 \begin{equation*}
1554 \overrightarrow{B}(3,-2,6),\;\overrightarrow{C}(-5,1,-8)
1555 \end{equation*}%
1556 Amb la convenció d'eixos del dibuix anterior (\autoref{fig:sistema-de-coordenades-3d}), el vector $\overrightarrow{B}$ apunta cap a la dreta, s'allunya del lector, i cap a dalt, i el vector $\overrightarrow{C}$ apunta cap a l'esquerre, cap al lector, i cap a baix. Com que no se'ns diu quins són els seus origens, es consider que aquests vectors són lliures i que, per tant, els seus punts d'origen es poden situar on es desitgi.
1557 \end{example}
1559 \subsection{Base estàndard de vectors}
1561 \begin{definition}[base estàndard de vectors]A l'espai cartesià, existeixen tres vectors que formen el que s'anomena \term{base estàndard de vectors}\index{base estàndard de vectors} la qual està formada per tres vectors $\overrightarrow{i}$, $\overrightarrow{j}$ i $\overrightarrow{k}$, que tenen les coordenades següents:
1562 \begin{equation*}
1563 \overrightarrow{i}=\overrightarrow{(1,0,0)},\; \overrightarrow{j}=\overrightarrow{(0,1,0)}, \; \overrightarrow{k}=\overrightarrow{(0,0,1)}
1564 \end{equation*}%
1566 Aquests vectors són unitaris, ortogonals (perpendiculars entre si) i formen un base: qualsevol vector es pot posar com a combinació lineal de $\overrightarrow{i}$, $\overrightarrow{j}$ i $\overrightarrow{k}$\footnote{Les definicions de base d'un espai vectorial escapen a l'abast d'aquest text.}. És a dir, si $\overrightarrow{v}$ és un vector, aleshores existeixen nombres $a$, $b$ i $c$ de manera que $\overrightarrow{v} = a \overrightarrow{i} + b \overrightarrow{j} + c \overrightarrow{k}$. Per les definicions de $\overrightarrow{i}$, $\overrightarrow{j}$ i $\overrightarrow{k}$ és clar que aquests $a$, $b$ i $c$ són els valors de les coordenades de $v$.
1567 \end{definition}
1569 \begin{example}El vector $\overrightarrow{v} = (3,2,2)$ compleix que $\overrightarrow{v} = 3 \overrightarrow{i} +2 \overrightarrow{j} + 2 \overrightarrow{k}$. Es pot veure la seva representació a la figura següent (\autoref{fig:exemple-base-estandard}).
1571 \begin{figure}[h!]
1572 \centering
1573 % Generat amb TikZ
1574 \begin{tikzpicture}[thick]
1575 \draw[->] (0,0,0) -- (4,0,0);
1576 \draw[->] (0,0,0) -- (0,3,0);
1577 \draw[->] (0,0,0) -- (0,0,3);
1578 \draw (4,0,0) node[anchor=west] {$x$};
1579 \draw (0,3,0) node[anchor=south] {$z$};
1580 \draw (0,0,3) node[anchor=north east] {$y$};
1581 \draw[dashed] (3,0,2) -- (3,2,2);
1582 \draw[loosely dotted] (3,0,2) -- (0,0,2);
1583 \draw[loosely dashed] (3,0,2) -- (3,0,0);
1584 \draw[color=blue,very thick,->] (0,0,0) -- (3,2,2);
1585 \draw (3,2,2) node[anchor=west] {$\overrightarrow{v}(3,2,2)$};
1586 \draw[color=orange,very thick,->] (0,0,0) -- (3,0,0);
1587 \draw[color=orange,very thick,->] (0,0,0) -- (0,2,0);
1588 \draw[color=orange,very thick,->] (0,0,0) -- (0,0,2);
1589 \draw (3,0,0) node[anchor=south west] {$3 \cdot \overrightarrow{i}$};
1590 \draw (0,2,0) node[anchor=east] {$2\cdot \overrightarrow{k}$};
1591 \draw (0,0,2) node[anchor=east] {$2 \cdot \overrightarrow{j}$};
1592 \end{tikzpicture}
1593 % Needed \protect http://www.latex-community.org/forum/viewtopic.php?f=44&t=4475
1594 \caption{Descomposició lineal del vector $\protect\overrightarrow{v}(3,2,2)$ respecte de la base estàndard}
1595 \label{fig:exemple-base-estandard}
1596 \end{figure}
1597 \end{example}
1599 \subsection{Operacions amb vectors anàlogues al pla}
1601 El mòdul d'un vector i les operacions de suma i resta de vectors, producte d'un escalar per un vector i producte escalar de dos vectors es defineixen de manera anàloga al pla:
1602 \begin{itemize}
1603 \item El mòdul d'un vector $\overrightarrow{u}(a,b,c)$ es calcula com
1604 \begin{equation*}
1605 \lvert \overrightarrow{u} \rvert = \sqrt{a^2 + b^2 +c^2}
1606 \end{equation*}
1608 Per exemple, $\lvert \overrightarrow{(3,-2,6)} \rvert = \sqrt{3^2 + (-2)^2 + 6^2} = \sqrt{49} = 7$.
1610 \item Donats dos vectors $\overrightarrow{u}(u_1,u_2,u_3)$ i $\overrightarrow{v}(v_1,v_2,v_3)$, la seva suma es defineix com $\overrightarrow{u}+\overrightarrow{v} = (u_1+v_1,u_2+v_2,u_3+v_3)$.
1612 Per exemple, si $\overrightarrow{u} = (2,-3,4)$ i $\overrightarrow{v} = (-2,-7,0)$, aleshores $\overrightarrow{u}+\overrightarrow{v} = (0,-10,4)$.
1614 \item La resta de dos vectors $\overrightarrow{u}(u_1,u_2,u_3)$ i $\overrightarrow{v}(v_1,v_2,v_3)$ es defineix com $\overrightarrow{u}-\overrightarrow{v} = (u_1-v_1,u_2-v_2,u_3-v_3)$.
1616 Per exemple, si $\overrightarrow{u} = (2,-3,4)$ i $\overrightarrow{v} = (-2,-7,0)$, aleshores $\overrightarrow{u}-\overrightarrow{v} = (4,4,4)$.
1618 \item Si $k \in \mathbb{R}$ és un nombre qualsevol i $\overrightarrow{u}(u_1,u_2,u_3)$ és un vector, llavors el producte $k \cdot \overrightarrow{u} = (k \cdot u_1, k \cdot u_2, k \cdot u_3)$.
1620 Per exemple, si $\overrightarrow{u} = (2,-3,4)$ i $k = -3$, aleshores $k \cdot \overrightarrow{u} = (-6,+9,-12)$.
1622 \item Dos vectors $\overrightarrow{u}(u_1,u_2,u_3)$ i $\overrightarrow{v}(v_1,v_2,v_3)$ són para\lgem{els} si, i només si, $\frac{v_1}{u_1} = \frac{v_2}{u_2} = \frac{v_3}{u_3}$ (vegeu \autoref{resultat:proposicio-parallelisme-vectors})
1624 \item Si $\overrightarrow{u}(u_1,u_2,u_3)$ i $\overrightarrow{v}(v_1,v_2,v_3)$ són vectors, el seu producte escalar es defineix com
1625 \begin{equation*}
1626 \overrightarrow{u} \cdot \overrightarrow{v} = u_1 \cdot v_1 + u_2 \cdot v_2 + u_3 \cdot v_3
1627 \end{equation*}
1629 Així per exemple, $\overrightarrow{(3,-2,4)} \cdot \overrightarrow{(-1,0,-5)} = -3 + 0 -20 = -23$.
1631 Es verifica el resultat relatiu a l'angle entre dos vectors (\autoref{resultat:angle-producte-esclar}) i les propietats del producte escalar (\autoref{resultat:propietats-del-producte-esclar}).
1632 \end{itemize}
1635 \begin{exercise}Determineu si els vectors són para\lgem{els} entre si:
1637 \begin{enumerate}[label=\emph{\alph*})]
1638 \item $\overrightarrow{u} = \overrightarrow{\left( 2,-3,1\right)}$ i $\overrightarrow{v} = \overrightarrow{\left( 4,-6,2\right)}$
1640 \item $\overrightarrow{u} = \overrightarrow{\left( 2,-3,1\right)}$ i $\overrightarrow{v}= \overrightarrow{\left( 4,-6,3\right)}$
1641 \end{enumerate}
1642 \end{exercise}
1644 \begin{exercise}Què val l'angle format entre els vectors $\overrightarrow{u}(2,0,-3)$ i $\overrightarrow{v}(-3,1,2)$?
1645 \end{exercise}
1647 \subsection{Producte vectorial}
1649 Vegem tot seguit una operació nova: el producte vectorial entre vectors. Noteu la paraula {\em vectorial} (que no escalar) a aquesta expressió. La importància d'aquesta paraula és perquè el producte vectorial donarà com a resultat un vector mentre que el producte escalar dóna com a resultat un nombre.
1651 \begin{definition}[producte vectorial de vectors]El \term{producte vectorial de dos vectors}\index{producte!vectorial}, $\overrightarrow{u}(u_1,u_2,u_3)$ i $\overrightarrow{w}(w_1,w_2,w_3)$, que es denota per $\overrightarrow{u}\wedge \overrightarrow{w}$ o $\overrightarrow{u}\times \overrightarrow{w}$, respecte de la base estàndard, es defineix com:
1652 \begin{equation*}
1653 \begin{split}
1654 \overrightarrow{u}\wedge \overrightarrow{w} & =(u_1,u_2,u_3)\wedge
1655 (w_1, w_2, w_3) \\
1656 & =\left\vert
1657 \begin{array}{ccc}
1658 u_1 & u_2 & u_3 \\
1659 w_1 & w_2 & w_3 \\
1660 \overrightarrow{i} & \overrightarrow{j} & \overrightarrow{k}%
1661 \end{array}%
1662 \right\vert
1663 \end{split}
1664 \end{equation*}
1666 Aquest determinant es pot fer aplicant la regla de Sarrus (\autoref{alg:regla-de-Sarrus}).
1668 \end{definition}
1670 Notem que el producte vectorial de dos vectors és, per tant, un altre vector.
1672 \begin{example}Siguin els vectors $\overrightarrow{u}(2,0,-3)$ i $\overrightarrow{v}(-3,1,2)$. Aleshores, el seu producte vectorial és:
1673 \begin{equation*}
1674 \begin{split}
1675 \overrightarrow{u}\wedge \overrightarrow{v} &= \left\vert
1676 \begin{array}{ccc}
1677 2 & 0 & -3 \\
1678 -3 & 1 & 2 \\
1679 \overrightarrow{i} & \overrightarrow{j} & \overrightarrow{k}%
1680 \end{array}%
1681 \right\vert \\
1682 & = 2\overrightarrow{k} + 0 + 9 \overrightarrow{j} + 3 \overrightarrow{i} + 0 - 4 \overrightarrow{j}\\
1683 & = 3\overrightarrow{i} + 5 \overrightarrow{j} + 2 \overrightarrow{k}\\
1684 & =\left( 3,5,2\right)
1685 \end{split}
1686 \end{equation*}
1687 \end{example}
1689 \begin{exercise}Calculeu $\overrightarrow{u}\wedge \overrightarrow{v}$, amb $\overrightarrow{u}(0,-2,3)$ i $\overrightarrow{v}(-3,-5,4)$.
1690 \end{exercise}
1692 Hi ha una proposició que permet calcular el mòdul del producte vectorial sense calcular el producte vectorial en si mateix. Aquesta proposició és la següent.
1693 \begin{proposition}[mòdul del producte vectorial]Donat dos vectors $\overrightarrow{u}$ i $\overrightarrow{v}$, el mòdul del seu producte vectorial $\overrightarrow{u} \wedge \overrightarrow{v}$ compleix que \begin{equation*}
1694 \left\vert \overrightarrow{u}\wedge \overrightarrow{v}\right\vert = \lvert \overrightarrow{u} \rvert \cdot \lvert \overrightarrow{v} \rvert \cdot \sin \widehat{uv},
1695 \end{equation*}%
1696 on $\widehat{uv}$ denota l'angle que formen els vectors $\overrightarrow{u}$ i $\overrightarrow{v}$.
1697 \end{proposition}
1699 Notem que el sinus d'un angle es pot calcular de forma exacta amb qualsevol calculadora científica moderna. Així i tot, existeixen taules del sinus pels angles més usuals (vegeu \autoref{apendix:valors-raons-trigonometriques}).
1701 \subsubsection{Propietats del producte vectorial}
1703 Donats els vectors $\vec{a},\vec{b}$ i $\vec{c}$ i el nombre $k$ qualssevol, el producte vectorial verifica les propietats següents:
1705 \begin{enumerate}[label=\emph{\alph*})]
1706 \item $\overrightarrow{a}\wedge \overrightarrow{b}=-\overrightarrow{b}%
1707 \wedge \overrightarrow{a}$ (anticommutativa)
1709 \item $\overrightarrow{a}\wedge \overrightarrow{a}=\vec{0}$
1711 \item $k\left( \overrightarrow{a}\wedge \overrightarrow{b}\right) =\left( k\overrightarrow{a}\right) \wedge \overrightarrow{b}=\overrightarrow{a}\wedge \left( k\overrightarrow{b}\right) $
1713 \item $\overrightarrow{a}\wedge \left( \overrightarrow{b}+\vec{c}%
1714 \right) =\overrightarrow{a}\wedge \overrightarrow{b}+\overrightarrow{a}%
1715 \wedge \overrightarrow{c}$
1717 \item En general, $\overrightarrow{a}\wedge \left( \overrightarrow{b} \wedge \overrightarrow{c}\right) \neq \left( \overrightarrow{a}\wedge
1718 \overrightarrow{b}\right) \wedge \overrightarrow{c}$
1720 \item El vector $\overrightarrow{a}\wedge \overrightarrow{b}$ és perpendicular tant al vector $\overrightarrow{a}$ com al vector $\overrightarrow{b}$.
1722 \item El mòdul del producte vectorial de dos vectors ens dóna l'àrea del parale\lgem{ògram} definit per aquest dos vectors (\autoref{fig:calcul-area-parallelogram-producte-vectorial}):
1723 \begin{figure}[h!]
1724 \centering
1725 % Generat amb TikZ
1726 \definecolor{qqzzcc}{rgb}{0.,0.6,0.8}
1727 \begin{tikzpicture}[line cap=round,line join=round,>=triangle 45,x=1.0cm,y=1.0cm]
1728 \coordinate (A) at (0,1);
1729 \coordinate (B) at (3,4);
1730 \coordinate (C) at (9,3);
1731 \coordinate (D) at (6,0);
1732 % punt mitjà de A i B
1733 \coordinate (M1) at (1.5,2.5);
1734 % punt mijtà de A i D
1735 \coordinate (M2) at (3,0.5);
1736 \fill[color=qqzzcc,fill=qqzzcc,fill opacity=0.05] (A) -- (B) -- (C) -- (D) -- cycle;
1737 \draw [->] (A) -- (B);
1738 % vector AB = (3,3)
1739 \draw [->] (A) -- (D);
1740 % vector AD = (6,-1)
1741 \draw (M1) node[anchor=south east] {$\overrightarrow{u}$};
1742 \draw (M2) node[anchor=north east] {$\overrightarrow{v}$};
1743 \draw [color=qqzzcc] (B)-- (C);
1744 \draw [color=qqzzcc] (C)-- (D);
1745 \draw (3.72,2.32) node[anchor=north west] {$\vert \overrightarrow{u} \wedge \overrightarrow{v} \vert$};
1746 \end{tikzpicture}
1747 % Needed \protect http://www.latex-community.org/forum/viewtopic.php?f=44&t=4475
1748 \caption{Àrea del para\lgem{elogram} determinat pels vectors $\protect\overrightarrow{u}$ i $\protect\overrightarrow{v}$}
1749 \label{fig:calcul-area-parallelogram-producte-vectorial}
1750 \end{figure}
1752 \end{enumerate}
1754 \begin{claim}
1755 Com que $\overrightarrow{a} \wedge \overrightarrow{b}$ i $\overrightarrow{b} \wedge \overrightarrow{a}$ són perpendiculars a $\overrightarrow{a}$ i $\overrightarrow{b}$, això vol dir que $\overrightarrow{a} \wedge \overrightarrow{b}$ i $\overrightarrow{b} \wedge \overrightarrow{a}$ estan a la mateixa línia, un apuntant cap a baix i un apuntant cap a dalt. Per determinar l'orientació d'aquests dos vectors de forma gràfica existeix un procediment, anomenat {\em regla del llevataps}\index{regla!del llevataps} \cite{llevataps}: per determinar si $\overrightarrow{a} \wedge \overrightarrow{b}$ apunta cap a baix o cap a dalt, hem de representar els dos vectors en un mateix pla amb el mateix origen\footnote{Recordem que podem moure els vectors i situar-los amb un mateix origen perquè són vectors lliures.} i observar si el moviment que desplaçaria $\overrightarrow{a}$ cap a $\overrightarrow{b}$ es fa amb sentit horari o bé amb sentit antihorari:
1756 \begin{itemize}
1757 \item Si es fa amb sentit horari, aleshores $\overrightarrow{a} \wedge \overrightarrow{b}$ apuntarà cap a baix.
1758 \item Si per contra, es fa amb sentit antihorari, aleshores apuntarà cap a dalt.
1759 \end{itemize}
1761 \bigskip
1762 Per tant, el producte vectorial és útil si es vol trobar un vector perpendicular a dos donats.
1763 \end{claim}
1765 \begin{exercise}Trobeu un vector que sigui perpendicular als vectors $\overrightarrow{a}(-4,0,3)$ i $\overrightarrow{b}(-3,-1,0)$, simultàniament.
1766 \end{exercise}
1768 \begin{exercise}Trobeu un vector ortogonal a $\overrightarrow{u}(4,-2,5)$ i $\overrightarrow{v}(3,0,-5)$. Trobeu un altre vector ortonormal.
1769 \end{exercise}
1771 \begin{exercise}Trobeu l'àrea del para\lgem{elogram} que formen els vectors $\overrightarrow{u}(-2,0,4)$ i $\overrightarrow{v}(1,3,-1)$.
1772 \end{exercise}
1774 \subsection{Producte mixt}
1776 Tot seguit veurem una nova operació, entre tres vectors, la qual tendrà la principal aplicació de calcular volums de determinats prismes i piràmides (\autoref{resultat:calcul-volum-parallelepiped}, \autoref{resultat:calcul-volum-tetraedre}).
1778 \begin{definition}[producte mixt]Donats tres vectors $\overrightarrow{u}$, $\overrightarrow{v}$ i $\overrightarrow{w}$, el seu \term{producte mixt}\index{producte!mixt}, que es denota per $\left[ \overrightarrow{u},\overrightarrow{v},\overrightarrow{w}\right]$, es defineix com
1779 \begin{equation*}
1780 \left[ \overrightarrow{u},\overrightarrow{v},\overrightarrow{w}\right] =\overrightarrow{u}\cdot (\overrightarrow{v}\wedge \overrightarrow{w}).
1781 \end{equation*}
1782 \end{definition}
1784 Notem que el producte mixt no és, en general, una operació commutativa. És a dir, el valor numèric del producte mixt depèn fortament de l'ordre dels vectors involucrats.
1786 \begin{example}Donats els vectors $\vec{u}\left( 0,-1,5\right) ,$ $\vec{v}(2,0,-3)$ i $\vec{w}(-3,1,2)$, calculeu el producte mixt $\left[ \overrightarrow{u},\overrightarrow{v},\overrightarrow{w}\right]$.
1788 Tenim que
1789 \begin{equation*}
1790 \begin{split}
1791 \left[ \overrightarrow{u},\overrightarrow{v},\overrightarrow{w}\right] & = \overrightarrow{u}\cdot \left(\overrightarrow{v}\wedge \overrightarrow{w}\right) \\
1792 & = \left( 0,-1,5\right) \cdot \left( (2,0,-3)\wedge (-3,1,2)\right) \\
1793 & = \left( 0,-1,5\right) \cdot \left( 3,5,2\right)\\
1794 & =0-5+10=5.
1795 \end{split}
1796 \end{equation*}
1797 \end{example}
1799 \begin{proposition}[Càlcul del producte mixt usant determinants]Per a qualssevol vectors $\overrightarrow{u} = (u_1, u_2, u_3)$, $\overrightarrow{v} = (v_1, v_2, v_3)$ i $\overrightarrow{w} = (w_1, w_2, w_3)$, el producte mixt $\left[ \overrightarrow{u},\overrightarrow{v},\overrightarrow{w}\right]$ també es pot calcular amb la fórmula següent:%
1800 \begin{equation*}
1801 \left[ \overrightarrow{u},\overrightarrow{v},\overrightarrow{w}\right]
1802 =\left\vert
1803 \begin{array}{rrr}
1804 u_1 & u_2 & u_3 \\
1805 v_1 & v_2 & v_3\\
1806 w_1 & w_2 & w_3%
1807 \end{array}%
1808 \right\vert,
1809 \end{equation*}
1810 és a dir, els vectors $\overrightarrow{u}$, $\overrightarrow{v}$ i $\overrightarrow{w}$ disposats per fileres al determinant.
1811 \end{proposition}
1813 \begin{example}Donats els vectors $\vec{u}\left( 0,-1,5\right)$, $\vec{v}(2,0,-3)$ i $\vec{w}(-3,1,2)$, calculeu $\left[ \overrightarrow{u},\overrightarrow{v},\overrightarrow{w}\right]$:%
1814 \begin{equation*}
1815 \left[ \overrightarrow{u},\overrightarrow{v},\overrightarrow{w}\right]
1816 =\left\vert
1817 \begin{array}{rrr}
1818 0 & -1 & 5 \\
1819 2 & 0 & -3 \\
1820 -3 & 1 & 2%
1821 \end{array}%
1822 \right\vert =-9+10+4=5.
1823 \end{equation*}
1824 \end{example}
1826 \begin{exercise}Calculeu $\left[ \overrightarrow{u},\overrightarrow{v},\overrightarrow{w}\right]$, amb $\vec{u}\left( 3,1,-2\right)$, $\vec{v}(-2,10,0)$ i $\vec{w}(0,-1,-5)$.
1827 \end{exercise}
1829 \begin{definition}[para\lgem{elepípede}]Un \term{para\lgem{elepípede}}\index{para\lgem{elepípede}} és un prisme la base del qual és un para\lgem{elogram} (\autoref{fig:wikipedia-parallelepipede}).
1831 \begin{figure}[h!]
1832 \centering
1833 \includegraphics[scale=0.25]{./graphics/wk-Parallellopipedum.png}
1834 \caption{Un para\lgem{elepípede}}
1835 \label{fig:wikipedia-parallelepipede}
1836 \end{figure}
1838 \end{definition}
1840 \begin{proposition}[càlcul del volum d'un para\lgem{elepípede}]\label{resultat:calcul-volum-parallelepiped}Donat el para\lgem{elepípede}\index{volum!para\lgem{elepípede}} definit pels vectors $\overrightarrow{u}$, $\overrightarrow{v}$ i $\overrightarrow{w}$ (vegeu \autoref{fig:calcul-volum-parallelepípede}), el seu volum $V_p$ és igual al valor absolut del producte mixt, és a dir,
1841 \begin{equation*}
1842 V_p=\left\vert \left[ \overrightarrow{u},\overrightarrow{v},\overrightarrow{w}\right] \right\vert
1843 \end{equation*}%
1845 \begin{figure}[h!]
1846 \centering
1847 % Generat amb TikZ
1849 \begin{tikzpicture}[line join=bevel,scale=1.2]
1850 %\rotateRPY{45}{0}{45}
1851 \begin{scope}
1852 \coordinate (A1) at (0,0,0);
1853 \coordinate (A2) at (-2,0,-3);
1854 \coordinate (A3) at (-4,0,-3);
1855 \coordinate (A4) at (-2,0,0);
1857 \coordinate (B1) at (0.2,-1,0);
1858 \coordinate (B2) at (-1.8,-1,-3);
1859 \coordinate (B3) at (-3.8,-1,-3);
1860 \coordinate (B4) at (-1.8,-1,0);
1862 % Punts mitjans
1863 \coordinate (M1) at (-0.5,-1,0);
1864 \coordinate (M2) at (-0.8,-1,-1.5);
1865 \coordinate (M3) at (0.1,-0.5,0);
1867 \draw (A1) -- (A2) -- (A3) -- (A4) -- cycle;
1868 \draw[loosely dashed] (B1) -- (B2) -- (B3);
1869 \draw (B3) -- (B4)-- (B1);
1870 \draw (A1) -- (B1);
1871 \draw[loosely dashed] (A2) -- (B2);
1872 \draw (A3) -- (B3);
1873 \draw (A4) -- (B4);
1874 %\draw (A1) node {$A1$};
1875 %\draw (A2) node {$A2$};
1876 %\draw (A3) node {$A3$};
1877 %\draw (A4) node {$A4$};
1878 %\fill [fill opacity=0.7,fill=green!20] (A1) -- (A2) -- (B2) -- (B1)-- cycle;
1879 %\fill [fill opacity=0.7,fill=orange!20] (A2) -- (A3) -- (B3) -- (B2)-- cycle;
1880 \fill [fill opacity=0.7,fill=purple!20] (A3) -- (A4) -- (B4) -- (B3)-- cycle;
1881 \fill [fill opacity=0.7,fill=red!40] (A4) -- (A1) -- (B1) -- (B4)-- cycle;
1882 \fill [fill opacity=0.7,fill=yellow!20] (A1) -- (A2) -- (A3) -- (A4)-- cycle;
1883 %\fill [fill opacity=0.7,fill=blue!20] (B1) -- (B2) -- (B3) -- (B4)-- cycle;
1884 \draw[ultra thick, color=blue,->] (B1) -- (A1);
1885 \draw[ultra thick, color=blue,->,loosely dashed] (B1) -- (B2);
1886 \draw[ultra thick, color=blue,->] (B1) -- (B4);
1887 \draw (M1) node[anchor=north] {$\overrightarrow{u}$};
1888 \draw (M2) node[anchor=east] {$\overrightarrow{v}$};
1889 \draw (M3) node[anchor=west] {$\overrightarrow{w}$};
1890 \end{scope}
1891 \end{tikzpicture}
1893 % Needed \protect http://www.latex-community.org/forum/viewtopic.php?f=44&t=4475
1894 \caption{Volum del para\lgem{elepípede} definit pels vectors $\protect\overrightarrow{u}$, $\protect\overrightarrow{v}$ i $\protect\overrightarrow{w}$}
1895 \label{fig:calcul-volum-parallelepípede}
1896 \end{figure}
1898 Això vol dir que heu de calcular el producte mixt dels vectors que defineixen els para\lgem{elepípede} i després positivar el seu resultat.
1899 \end{proposition}
1901 \begin{definition}[tetraedre] Un \term{tetraedre}\index{tetraedre} és una piràmide amb totes les cares iguals entre si i iguals a triangles equilàters.
1903 \begin{figure}[h!]
1904 \centering
1905 % Generat amb TikZ
1906 % De https://tex.stackexchange.com/a/498775
1908 \tdplotsetmaincoords{70}{100}
1909 \begin{tikzpicture}[line join=round,tdplot_main_coords,declare function={a=5;}]
1910 \begin{scope}[canvas is xy plane at z=0,transform shape]
1911 \path foreach \X [count=\Y] in {A,B,C}
1912 {(\Y*120:{a/(2*cos(30))}) coordinate(\X)};
1913 \end{scope}
1914 \path (0,0,{a*cos(30)}) coordinate (D);
1915 \draw foreach \X/\Y [remember=\X as \Z (initially D)] in {A/B,B/C,C/D,D/A}
1916 {(\X) -- (\Z) -- (\Y)};
1917 \end{tikzpicture}
1919 \caption{Dibuix d'un tetraedre}
1920 \label{fig:calcul-volum-tetraedre}
1921 \end{figure}
1924 \end{definition}
1927 \begin{proposition}[càlcul del volum d'un tetraedre]\label{resultat:calcul-volum-tetraedre}Donat el tetraedre\index{volum!tetraedre} format pels vectors $\overrightarrow{u},\overrightarrow{v}$ i $\overrightarrow{w}$, el seu volum $V_t$ és igual a:
1928 \begin{equation*}
1929 V_t=\frac{1}{6}\left\vert \left[ \overrightarrow{u},\overrightarrow{v},\overrightarrow{w}\right] \right\vert ,
1930 \end{equation*}
1931 és a dir, calcular el producte mixt dels vectors corresponents, positivar-lo i multiplicar-lo per un sisè.
1932 \end{proposition}
1934 \begin{exercise}Calculeu el volum d'un tetraedre definit pels vectors $\overrightarrow{(1, 2, -2)}$, $\overrightarrow{(-1, 0, 1)}$ i $\overrightarrow{(2, 1, 0)}$. Comproveu que el volum del para\lgem{elepípede} definit pels mateixos vectors és sis vegades més gran.
1935 \end{exercise}
1937 \section{La recta a l'espai}
1939 En aquest apartat farem un estudi de la recta\index{recta} en un espai de tres dimensions.
1941 Si d'una recta coneixem un punt qualsevol d'aquesta i un vector tengui la mateixa direcció (és a dir, que estigui situat sobre ella o bé que estigui situat sobre una recta para\lgem{ela}), llavors tenim elements suficients per a determinar-la completament, és a dir, per a determinar les coordenades de qualsevol punt.
1943 En altres paraules, basta que coneguem un punt de la recta $P_0$ i el seu \term{vector director}\index{vector!director} $\overrightarrow{v}$.
1945 \begin{definition}[equació vectorial de la recta]Una recta $r$ es pot determinar per un punt $P_0(x_0,y_0,z_0)$ de la recta i un vector director $\overrightarrow{v}$, de manera que, per a qualsevol punt $P(x,y,z)$ pertanyent a la recta, es té que
1946 \begin{equation}\label{eq:eq-vectorial-recta-3d}
1947 \overrightarrow{OP} = \overrightarrow{OP_0} + \lambda \overrightarrow{v},
1948 \end{equation}
1949 per qualque $\lambda \in \mathbb{R}$ (\autoref{fig:equacio-vectorial-recta-3d-exemple}). Aquesta equació (\ref{eq:eq-vectorial-recta-3d}) es coneix com a \term{equació vectorial de la recta}\index{equació!vectorial!d'una recta}.
1952 \begin{figure}[h!]
1953 \centering
1954 % Generat amb TikZ
1955 \begin{tikzpicture}[thick]
1956 \coordinate (O) at (0,0,0);
1957 \coordinate (P0) at (3,2,2);
1958 \coordinate (R1) at (-2,1,2);
1959 \coordinate (P) at (5.5,2.5,2);
1960 \coordinate (P2) at (8,3,2);
1961 \coordinate (M) at (4.5,2.3,2);
1962 \coordinate (M2) at (3.75,2.15,2);
1965 % eixos de coordenades
1966 \draw[->] (0,0,0) -- (4,0,0);
1967 \draw[->] (0,0,0) -- (0,3,0);
1968 \draw[->] (0,0,0) -- (0,0,3);
1969 \draw (4,0,0) node[anchor=west] {$x$};
1970 \draw (0,3,0) node[anchor=south] {$z$};
1971 \draw (0,0,3) node[anchor=north east] {$y$};
1973 % Punts i rectes
1974 %\draw[dashed] (3,0,2) -- (3,2,2);
1975 %\draw[loosely dotted] (3,0,2) -- (0,0,2);
1976 %\draw[loosely dashed] (3,0,2) -- (3,0,0);
1978 \draw[color=orange,very thick] (R1) -- (P0) -- (P) -- (P2);
1979 \draw (R1) node[anchor=north] {$r$};
1980 \draw[color=blue,very thick,->] (O) -- (P0);
1981 \draw[color=blue,very thick,->] (O) -- (P);
1982 \draw (P0) node[anchor=south] {$P_0$};
1983 \filldraw[color=green] (P0) circle (2pt);
1984 \draw (P) node[anchor=south] {$P$};
1985 \filldraw[color=green] (P) circle (2pt);
1986 \draw[ultra thick,color=blue,->] (P0) -- (M);
1987 \draw (M2) node[anchor=south] {$\overrightarrow{v}$};
1988 \end{tikzpicture}
1989 % Needed \protect http://www.latex-community.org/forum/viewtopic.php?f=44&t=4475
1990 \caption{Representació de la recta que té vector director $\protect\overrightarrow{v}$ i que passa per $P_0$.}
1991 \label{fig:equacio-vectorial-recta-3d-exemple}
1992 \end{figure}
1994 \end{definition}
1997 \subsection{Equació paramètriques de la recta}
1999 Sigui $r$ una recta determinada per $P_0(x_0,y_0,z_0)$ un punt qualsevol i $\overrightarrow{v}(v_x,v_y,v_z)$ és el seu vector director. Aleshores un punt $P(x,y,z)$ de la recta compleix l'equació vectorial (\autoref{eq:eq-vectorial-recta-3d}), és a dir, $\overrightarrow{OP} = \overrightarrow{OP_0} + \lambda \overrightarrow{v}$, per qualque $\lambda \in \mathbb{R}$, o sigui, $\overrightarrow{(x,y,z)} = \overrightarrow{(x_0,y_0,z_0)} + \lambda \overrightarrow{(v_x, v_y, v_z)}$. Operant, tenim que s'ha de verificar que
2000 \begin{equation*}
2001 \overrightarrow{(x,y,z)} = \overrightarrow{(x_0 + \lambda v_x,y_0 + \lambda v_y ,z_0+ \lambda v_z)}.
2002 \end{equation*}
2003 Si dos vectors són iguals, llavors component a component són iguals. El que implica que
2004 \begin{equation}\label{eq:equacio-parametrica-recta-3d}
2005 r \colon \left\{
2006 \begin{array}{l}
2007 x=x_0+\lambda v_x \\
2008 y=y_0+\lambda v_y \\
2009 z=z_0+\lambda v_z%
2010 \end{array}%
2011 \right. ,
2012 \end{equation}
2013 on $\lambda \in \mathbb{R}$. Aquesta equació (\autoref{eq:equacio-parametrica-recta-3d}), reb el nom de \term{equació paramètrica de la recta}\index{equació!paramètrica!d'una recta}.
2015 \begin{example}\label{exemple:equacio-parametrica-3d-1}Si una recta passa pel punt $(0,-1,3)$ i el seu vector director és el $\overrightarrow{v}(-3,2,0)$, llavors la seva equació paramètrica és la següent:%
2016 \begin{equation*}
2017 \left\{
2018 \begin{array}{l}
2019 x=0+\lambda \cdot \left( -3\right) \\
2020 y=-1+\lambda \text{$\cdot $}2 \\
2021 z=3+\lambda \text{$\cdot $}0%
2022 \end{array}%
2023 \right. \text{, és a dir, }\left\{
2024 \begin{array}{l}
2025 x=-3\lambda \\
2026 y=-1+2\lambda \\
2027 z=3%
2028 \end{array}%
2029 \right.
2030 \end{equation*}
2031 \end{example}
2033 Per trobar més punts d'aquesta recta basta substituir $\lambda$ per qualsevol nombre a les expressions anteriors.
2035 \begin{example}Si a la recta anterior (\autoref{exemple:equacio-parametrica-3d-1}), feim $\lambda=2$, tenim que%
2036 \begin{equation*}
2037 \left.
2038 \begin{array}{l}
2039 x=-6 \\
2040 y=-1+4=3 \\
2041 z=3%
2042 \end{array}%
2043 \right\} ,
2044 \end{equation*}%
2045 i, per tant, $(-6,3,3)$ és un altre punt de la recta.
2046 \end{example}
2048 \begin{exercise}Escriviu les equacions paramètriques de les rectes següents:
2049 \begin{enumerate}[label=\emph{\alph*})]
2050 \item recta que passa pel punt $(-1,0,2)$ i que té la direcció donada pel vector director $\overrightarrow{v}(1,3,-5)$
2051 \item recta que passa per l'origen de coordenades i que té com a vector director $\overrightarrow{(
2052 1,-2,0)}$
2053 \item recta que passa pels punts $(3,-5,2)$ i $(2,-7,-3)$
2054 \end{enumerate}
2055 Trobeu dos punts més de cada recta.
2056 \end{exercise}
2058 \subsection{Equació contínua de la recta}
2060 Si aïllam $\lambda$ en cadascuna de les equacions de la recta en forma
2061 paramètrica (\autoref{eq:equacio-parametrica-recta-3d}), tenim que%
2062 \begin{equation*}
2063 \lambda =\frac{x-x_0}{v_x},\; \lambda =\frac{y-y_0}{v_y},\;\lambda =\frac{z-z_0}{v_z}
2064 \end{equation*}%
2065 Si igualam les expressions, obtenim el que s'anomena \term{equació en forma contínua de la recta} (o simplement \term{equació contínua})\index{equació!contínua!d'una recta}:
2066 \begin{equation}\label{eq:equacio-recta-forma-continua-3d}
2067 r\colon \frac{x-x_0}{v_x}=\frac{y-y_0}{v_y}=\frac{z-z_0}{v_z},
2068 \end{equation}%
2069 on $P(x_0,y_0,z_0)$ és un punt qualsevol de la recta i $\overrightarrow{v}(v_x,v_y,v_z)$ és el seu vector director.
2071 \begin{example}\label{exemple:recta-equacio-continua-1}La recta $r$ que passa pel punt $(0,-1,3)$ i té com a vector director $\overrightarrow{v}(-3,2,0)$ té com a equació contínua:%
2072 \begin{equation*}
2073 \frac{x}{-3}=\frac{y+1}{2}=\frac{z-3}{0}
2074 \end{equation*}
2075 \end{example}
2077 \begin{claim}Observem que en aquest exemple ha aparegut un denominador igual a $0$. A pesar de què la divisió per $0$ no és una operació que
2078 estigui definida, en el context de l'equació contínua d'una recta, aquesta expressió està permesa.
2079 \end{claim}
2081 \begin{exercise}\label{exercici:equacions-continues-rectes-3d}Escriviu les equacions contínues de la rectes següents:
2082 \begin{enumerate}[label=\emph{\alph*})]
2083 \item recta que passa per $(0,-5,3)$ i que té vector director $\overrightarrow{v}(1,-2,2)$
2084 \item recta que passa pels punts $(6,-2,0)$ i $(2,-1,-1)$
2085 \item recta que passa pel punt $(-1,-1,2)$ i que té com a vector director $\overrightarrow{v}(2,0,-3)$.
2086 \end{enumerate}
2087 \end{exercise}
2089 \subsection{Equació implícita de la recta}\label{subseccio:equacio-implicita-recta}
2091 Si a les equacions de la recta en forma contínua (\autoref{eq:equacio-recta-forma-continua-3d}) llevam els denominadors i transposem tots els termes al primer membre, obtindrem dues equacions de la forma:%
2092 \begin{equation}\label{eq:equacio-implicita-recta-3d}
2093 \left\{
2094 \begin{aligned}
2095 Ax+By+Cz+D & = 0 \\
2096 A^{\prime}x+B^{\prime}y+C^{\prime}z+D^{\prime} & = 0%
2097 \end{aligned}%
2098 \right.,
2099 \end{equation}%
2100 on $A$, $B$, $C$, $D$, $A'$, $B'$, $C'$ i $D'$ són nombres reals. Aquestes equacions reben el nom d'\term{equació implícita de la
2101 recta}\index{equació!implícita!d'una recta}.
2103 \begin{example}La recta $r$ que passa pel punt $(0,-1,3)$ i que té vector director $\overrightarrow{v}(-3,2,0)$ té l'equació contínua:
2104 \begin{equation*}
2105 r \colon \frac{x}{-3}=\frac{y+1}{2}=\frac{z-3}{0}
2106 \end{equation*}
2107 (vegis' \autoref{exemple:recta-equacio-continua-1}). Per tant, fent els productes creuats de cada igualtat, obtenim que:%
2108 \begin{equation*}
2109 r \colon \left\{
2110 \begin{array}{rll}
2111 2x & = & -3\left( y+1\right) \\
2112 0\left( y+1\right) & = & 2\left( z-3\right)\\
2113 0 & = & -3 (z-3)%
2114 \end{array}%
2115 \right.
2116 \end{equation*}
2117 Així, operant, obtenim un sistema de tres equacions i tres incògnites:
2118 \begin{equation*}
2119 r \colon \left\{
2120 \begin{array}{r}
2121 2x+3y+3=0 \\
2122 2z-6=0\\
2123 -3z + 9 = 0%
2124 \end{array}%
2125 \right.
2126 \end{equation*}
2127 Per la naturalesa de la recta aquest sistema té rang 2, és a dir, té un grau de llibertat. Això vol dir que d'aquestes tres equacions, n'hem de triar dues que no siguin linealment dependents. En el nostre cas, es veu que la segona i la tercera són linealment independents. Per tant, l'equació implícita de la recta queda:
2128 \begin{equation*}
2129 r \colon \left\{
2130 \begin{array}{r}
2131 2x+3y+3=0 \\
2132 2z-6=0%
2133 \end{array}%
2134 \right.
2135 \end{equation*}
2136 \end{example}
2138 \begin{claim}De vegades és necessari calcular tots els productes creuats per a passar de l'equació contínua a la implícita perquè els productes creuats de la primera i segona fracció i de la segona i tercera fracció poden ser linealment dependents. Vegeu per exemple l'exercici \autoref{exercici:eq-recta} apartat \ref{exercici:eq-recta-itemc}.
2139 \end{claim}
2141 \begin{claim}Noteu que en principi no podem obtenir l'equació implícita d'una recta directament amb el seu vector director i un punt d'aquesta. En aquest cas, hem de passar per l'equació contínua per obtenir l'equació implícita.
2142 \end{claim}
2144 \begin{claim}L'equació general de la recta en el pla és de la forma $Ax+By+D =0$ (\autoref{seccio:equacio-general-de-la-recta-2d}). Per tant, es podria pensar que la generalització d'aquesta equació a l'espai seria $Ax + By + Cz + D = 0$. Però això no és així: aquesta equació correspon a un pla (vegeu \autoref{seccio:el-pla-a-lespai}). La raó d'aquest fet és que, si fos així, tendríem una equació amb dues incògnites, la qual cosa ens donaria dos graus de llibertat, el que correspon a un pla. En aquest sentit, l'equació implícita d'una recta (\autoref{eq:equacio-implicita-recta-3d}) es pot veure com a la intersecció de dos plans.
2145 \end{claim}
2147 \begin{exercise}Trobeu les equacions implícites de les rectes de l'\autoref{exercici:equacions-continues-rectes-3d}.
2148 \end{exercise}
2150 \begin{solution}Una solució podria ser aquestes: \begin{enumerate*}[label=\emph{\alph*})] \item $\left\{ -2x -y-5 = 0, 2y + 2z + 4 = 0 \right\}$ \linebreak\item $\left\{ x+4y +2 = 0, -y-z-2 = 0 \right\}$ \item $\left\{2y + 2 = 0, -3x -2z +1 = 0\right\}$ \end{enumerate*}
2151 \end{solution}
2153 \subsubsection{Vector director a partir de l'equació implícita}\label{subseccio:vector-director-equacio-implicita}
2155 La proposició següent dóna una manera per trobar el vector director d'una recta que ve donada mitjançant l'equació implítica.
2158 \begin{proposition}[vector director a partir de l'equació implícita]\label{prop:v-d-a-partir-eq-implicita-3d}Sigui $r$ una recta donada amb l'equació implícita:
2159 \begin{equation*}
2160 r \colon \left\{
2161 \begin{aligned}
2162 Ax+By+Cz+D & = 0 \\
2163 A'x+B'y+C'z+D' & = 0%
2164 \end{aligned}%
2165 \right.
2166 \end{equation*}%
2167 El seu vector director, $v_r$, es pot calcular amb la fórmula:%
2168 \begin{equation}\label{eq:calcul-vector-director-producte-vectorial}
2169 \overrightarrow{v_{r}}=(A,B,C)\wedge (A',B',C')
2170 \end{equation}
2171 \end{proposition}
2173 \begin{example}El vector director de la recta%
2174 \begin{equation*}
2175 r \colon \left\{
2176 \begin{array}{r}
2177 2x+3y+3=0 \\
2178 2z-6=0%
2179 \end{array}%
2180 \right.
2181 \end{equation*}%
2182 és $\overrightarrow{v}=(2,3,0)\wedge (0,0,2)$, és a dir,
2183 \begin{equation*}
2184 \overrightarrow{v}=\left\vert
2185 \begin{array}{ccc}
2186 2 & 3 & 0 \\
2187 0 & 0 & 2 \\
2188 \overrightarrow{i} & \overrightarrow{j} & \overrightarrow{k}%
2189 \end{array}%
2190 \right\vert
2191 =\left( 6,-4,0\right).
2192 \end{equation*}
2193 \end{example}
2195 \begin{exercise}Calculeu el vector director de les rectes:%
2196 \begin{multicols}{2}
2197 \begin{enumerate}[label=\emph{\alph*})]
2198 \item $\left\{
2199 \begin{array}{r}
2200 x-y-5=0 \\
2201 4x+y-5z-6=0%
2202 \end{array}%
2203 \right.$
2204 \item $\left\{
2205 \begin{array}{r}
2206 2x-z-5=0 \\
2207 3x+y-z-2=0%
2208 \end{array}%
2209 \right.$
2210 \item $\left\{
2211 \begin{array}{r}
2212 2x=0 \\
2213 2z+y=0%
2214 \end{array}%
2215 \right.$
2216 \item $\left\{
2217 \begin{array}{r}
2218 2x+3y+5z-2=0 \\
2219 4x+6y+10z-4=0%
2220 \end{array}%
2221 \right.$
2222 \end{enumerate}
2223 \end{multicols}
2224 \end{exercise}
2227 \subsubsection{Pas de l'equació implícita a l'equació paramètrica}\label{subseccio:pas-de-implica-a-parametrica-js}
2229 \paragraph{Mètode 1. Càlcul de punts i vector director}
2231 Per passar de l'equació implícita a l'equació paramètrica ho farem indirectament: calcularem el vector director de la recta i un punt. Vegem-ho amb un exemple.
2233 \begin{example}Trobeu les equacions paramètriques de la recta
2234 \begin{equation*}
2235 r \colon \left\{
2236 \begin{aligned}
2237 2x+y+2z+4 & = 0 \\
2238 x-y+4z+12 & = 0%
2239 \end{aligned}%
2240 \right.
2241 \end{equation*}%
2243 \begin{itemize}
2244 \item Per l'apartat anterior (\autoref{subseccio:vector-director-equacio-implicita}), tenim que el seu vector director és
2245 \begin{equation*}
2246 \begin{split}
2247 \overrightarrow{v} &=\left\vert
2248 \begin{array}{ccc}
2249 2 & 1 &2 \\
2250 1 & -1 & 4 \\
2251 \overrightarrow{i} & \overrightarrow{j} & \overrightarrow{k}%
2252 \end{array}%
2253 \right\vert\\
2254 & = -2\overrightarrow{k} + 4\overrightarrow{i} + 2\overrightarrow{j} +2\overrightarrow{i} -\overrightarrow{k} -8\overrightarrow{j}\\
2255 & = 6\overrightarrow{i} -6\overrightarrow{j} -3 \overrightarrow{k} = \overrightarrow{\left( 6,-6,-3\right)}.
2256 \end{split}
2257 \end{equation*}
2260 \item Per trobar un punt, substituïm una variable qualsevol, o $x$ o $y$ o $z$\footnote{Penseu que una recta té 2 equacions i 3 incògnites. Per tant, hi ha un grau de llibertat. D'aquesta manera només hem de substituir una sola variable.}. Triem $x$. Substituïm, $x=0$. Llavors hem de resoldre el sistema:
2261 \begin{equation*}
2262 \left\{
2263 \begin{aligned}
2264 y+2z+4 & = 0 \\
2265 -y+4z+12 & = 0%
2266 \end{aligned}%
2267 \right.
2268 \end{equation*}%
2270 Per resoldre aquest sistema podem aplicar qualsevol dels mètodes de resolució de sistemes d'equacions (vegeu \autoref{apendix:seccio:metodes-de-resolucio-sistemes}). Nosaltres aplicarem el mètode de reducció\footnote{També podríem resoldre aquest sistema usant la regla de Cràmer.}: sumant les equacions obtenim: $6z + 16 = 0$, el que implica que $z = \frac{-8}{3}$. Substituïnt el valor de $z$ a la primera equació i realitzant els càlculs, obtenim $y = \frac{4}{3}$. Per tant, un punt de la recta és $(0, \frac{4}{3}, \frac{-8}{3})$.
2272 \item Amb tota la informació anterior, l'equació paramètrica de la recta és:
2273 \begin{equation*}
2274 r \colon \left\{
2275 \begin{array}{l}
2276 x= 6\lambda \\
2277 y=\frac{4}{3} -6 \lambda \\
2278 z=\frac{-8}{3} -3 \lambda%
2279 \end{array}%
2280 \right. .
2281 \end{equation*}%
2284 \end{itemize}
2286 \end{example}
2289 \paragraph{Mètode 2. Parametritzant variables}
2291 En aquest cas, per passar de l'equació implítica a l'equació paramètrica, escollirem una variable qualsevol, que parametritzarem. I després resoldrem un sistema d'equacions de dues variables i dues incògnites. Vegem-ho amb un exemple.
2293 \begin{example}Trobeu les equacions paramètriques de la recta
2294 \begin{equation*}
2295 r \colon \left\{
2296 \begin{aligned}
2297 2x+y+2z+4 & = 0 \\
2298 x-y+4z+12 & = 0%
2299 \end{aligned}%
2300 \right.
2301 \end{equation*}%
2303 \begin{itemize}
2304 \item Triarem $y$ per a parametritzar. Per tant, $y = \lambda$. Amb la qual cosa el sistema queda:
2305 \begin{equation*}
2306 \left\{
2307 \begin{aligned}
2308 2x+2z & = -4-\lambda \\
2309 x+4z & = -12 +\lambda%
2310 \end{aligned}%
2311 \right.
2312 \end{equation*}%
2314 \item Ara hem de resoldre aquest sistema: podem aplicar directament la regla de Cràmer o bé aplicar alguns dels mètodes de resolució de sistemes $2 \times 2$ (reducció, igualació o substitució; vegeu \autoref{apendix:seccio:metodes-de-resolucio-sistemes}). Optem per la primera opció:
2315 \begin{itemize}
2316 \item El determinant de la matriu de coeficients és:
2317 \begin{equation*}
2318 \left\vert
2319 \begin{array}{rr}
2320 2 & 2 \\
2321 1 & 4 %
2322 \end{array}%
2323 \right\vert =8-2=6
2324 \end{equation*}%
2325 \item Per tant,
2326 \begin{equation*}
2327 x = \frac{\left\vert
2328 \begin{array}{rr}
2329 -4-\lambda & 2 \\
2330 -12 + \lambda & 4 %
2331 \end{array}%
2332 \right\vert}{6} = \frac{8-6\lambda}{6} = \frac{4}{3} - \lambda
2333 \end{equation*}%
2334 \begin{equation*}
2335 z = \frac{\left\vert
2336 \begin{array}{rr}
2337 2 & -4-\lambda \\
2338 1 & -12 + \lambda %
2339 \end{array}%
2340 \right\vert}{6} = \frac{-20+3\lambda}{6} = \frac{-10}{3} + \frac{1}{2}\lambda
2341 \end{equation*}%
2342 \end{itemize}
2344 \item Amb tota la informació anterior, l'equació paramètrica de la recta és:
2345 \begin{equation*}
2346 r \colon \left\{
2347 \begin{array}{l}
2348 x= \frac{4}{3} -\lambda \\
2349 y= \lambda \\
2350 z=\frac{-10}{3} + \frac{1}{2}\lambda%
2351 \end{array}%
2352 \right. .
2353 \end{equation*}%
2356 \end{itemize}
2358 \end{example}
2361 \subsubsection{Exercicis d'equacions de rectes}
2363 Tenim diverses equacions per a expressar una recta (\autoref{fig:relacions-equacions-recta-3d}). Practiquem el pas d'unes a les altres.
2365 %\pgfdeclarelayer{background}
2366 %\pgfdeclarelayer{foreground}
2367 %\pgfsetlayers{background,main,foreground}
2369 \begin{figure}[h!]
2370 \centering
2371 % TikZ picture. From:
2372 % page 147 of manual,
2373 % http://www.texample.net/tikz/examples/inertial-navigation-system/
2374 % http://tex.stackexchange.com/questions/29621/how-can-i-express-half-the-distance-between-two-nodes
2375 % http://tex.stackexchange.com/questions/42611/list-of-available-tikz-libraries-with-a-short-introduction/42674#42674
2376 \begin{tikzpicture}[auto]
2377 \tikzstyle{cloud} = [draw=blue, thick, ellipse, fill=blue!20, minimum height=1em,text width=6em, text centered];
2378 \tikzstyle{blockred} = [rectangle, draw=orange, thick, fill=orange!20, text centered, rounded corners, minimum height=1em, text width=6em];
2379 \tikzstyle{block} = [rectangle, draw=green,thick, fill=green!20, text centered, rounded corners, minimum height=1em, text width=6em];
2380 \tikzstyle{line}=[draw, thick, -latex',shorten >=2pt];
2381 \matrix[column sep=1cm, row sep=2cm]
2383 % row 1
2384 \node[block] (vectorial) {Eq. vectorial}; &
2385 \node[block] (parametrica) {Eq. paramètrica}; &
2386 \node[block] (continua) {Eq. contínua}; \\
2387 % row 2
2390 \node[blockred] (implicita) {Eq. implícita}; \\
2391 % row 2
2392 \node[cloud] (v) {Vector director}; &
2393 \node[cloud] (p) {Un punt de la recta}; &
2397 % row 3
2399 \node[cloud] (q) {Un punt de la recta}; &
2403 \tikzstyle{every node}=[font=\itshape]
2404 \tikzstyle{every path}=[line]
2405 \path[<->] (vectorial) -- (parametrica);
2406 \path[<->] (parametrica) -- (continua);
2407 \path (continua) -- node [midway, anchor=east] {producte creuat equacions} (implicita);
2408 \path[dashed] (p) -- (v);
2409 \path[loosely dashed] (implicita) |- node[midway] {substituir $x$, $y$, $z$} (p);
2410 \path[loosely dashed] (implicita) edge [bend right] node[midway, anchor=north west] {producte vectorial} (v);
2411 % rectangles
2412 \begin{pgfonlayer}{background}
2413 % Compute a few helper coordinates
2414 \path (v.south west)+(-1,-1) node (a) {};
2415 \path (p.north east)+(+1,+1) node (b) {};
2416 \path[fill=yellow!20,rounded corners, draw=black!50, dashed] (a) rectangle (b);
2417 \path (vectorial.south west)+(-0.5,-0.5) node (c) {};
2418 \path (continua.north east)+(+0.5,+0.5) node (d) {};
2419 \path[fill=red!10,rounded corners, draw=black!50, dashed] (c) rectangle (d);
2420 \end{pgfonlayer}
2421 % Fletxes als quadres
2422 % Punt mitjà i sumes de coordenades usant tikz calc library
2423 \path[dashed] (q) -| ($(v.east) !.5! (p.west)$);
2424 \path[double,<->] ($ (v.north) + (0,1) $) -- ($ (vectorial.south)+(0,-0.5) $);
2425 \end{tikzpicture}
2427 \caption{Relacions entre les equacions d'una recta}
2428 \label{fig:relacions-equacions-recta-3d}
2429 \end{figure}
2431 \begin{example}Trobeu totes les equacions de la recta que passa pel punt $P(3,-2,0)$ i que té per vector director $\overrightarrow{v}(1,0,-1)$.
2433 \begin{solution*}
2434 Tenim que:
2435 \begin{itemize}
2436 \item L'equació vectorial és
2437 \begin{equation*}
2438 \overrightarrow{OP} = \overrightarrow{(3,-2,0)} + \lambda \overrightarrow{(1,0,-1)}
2439 \end{equation*}
2441 \item Les equacions paramètriques són%
2442 \begin{equation*}
2443 r:\left\{
2444 \begin{array}{l}
2445 x=3+\lambda \\
2446 y=-2 \\
2447 z=-\lambda%
2448 \end{array}%
2449 \right.
2450 \end{equation*}%
2452 \item L'equació contínua és%
2453 \begin{equation*}
2454 r:x-3=\frac{y+2}{0}=\frac{z}{-1}
2455 \end{equation*}%
2457 \item I les equacions implícites són%
2458 \begin{equation*}
2459 r:\left\{
2460 \begin{array}{rll}
2461 0\cdot \left( x-3\right) & = & y+2 \\
2462 -1\cdot \left( x-3\right) & = & z%
2463 \end{array}%
2464 \right. \text{, és a dir, }r:\left\{
2465 \begin{array}{rll}
2466 y+2 & = & 0 \\
2467 -x-z+3 & = & 0%
2468 \end{array}%
2469 \right.
2470 \end{equation*}
2471 \end{itemize}
2473 \end{solution*}
2474 \end{example}
2476 \begin{exercise}\label{exercici:eq-recta}Trobeu totes les equacions de les rectes següents:
2477 \begin{enumerate}[label=\emph{\alph*})]
2478 \item Recta que té vector director $\overrightarrow{(2,-3,-1)}$ i passa per $(0,2,-10)$
2479 \item Recta que passa pels punts $(7,-4,0)$ i $(3,0,-5)$
2480 \item\label{exercici:eq-recta-itemc} Recta donada per l'equació $\overrightarrow{(x,y,z)} = \overrightarrow{(3,2,1)} + \lambda \overrightarrow{(-1,0,1)}$, amb $(x,y,z)$ un punt qualsevol de la recta.
2481 \item Recta donada per $r \colon \frac{x-3}{5} = y+3 = \frac{z+2}{-2}$
2482 \item La recta $s \colon \left\{
2483 \begin{aligned}
2484 x+3y -z & = 0 \\
2485 4x + 7z & = 0%
2486 \end{aligned}\right.$
2487 \end{enumerate}
2488 \end{exercise}
2490 \subsubsection{Rectes para\lgem{eles}}
2492 Donada una recta $r$ que té vector director $\overrightarrow{v}$ i passa per $P$, si volem trobar una recta para\lgem{ela} $s$ que passi per $Q$, només hem de notar que $s$ tendrà $\overrightarrow{v}$ com a vector director i passarà per $Q$.
2495 \begin{example}Calculeu l'equació de la recta para\lgem{ela} a $r \colon \frac{x}{-4}=\frac{y+2}{3}=\frac{z-3}{2}$ que passa pel punt $A(2,5,-1)$.
2498 Donat que la recta que cercan és para\lgem{ela} a la recta $r$, ambdues tenen el mateix vector director: $\overrightarrow{v}(-4,3,2)$. A més, sabem que la recta ha de passar pel punt $A(2,5,-1)$. Llavors, si subtituïm aquestes dues dades, per exemple, a l'equació contínua de la recta, obtindrem:%
2499 \begin{equation*}
2500 \frac{x-2}{-4}=\frac{y-5}{3}=\frac{z+1}{2}
2501 \end{equation*}
2502 \end{example}
2504 \begin{exercise}Donada la recta $r:\left\{ x=3+\lambda , \, y=-2,\, z=-\lambda \right\}$ en forma paramètrica, trobeu l'equació contínua de la
2505 recta para\lgem{ela} a $r$ que passa pel punt $A(0,-8,6)$.
2506 \end{exercise}
2508 \begin{exercise}Trobeu l'equació paramètrica de la recta $s$ que passa per $(0,4,4)$ i és para\lgem{ela} a $r \colon \left\{
2509 \begin{aligned}
2510 2x-y -2z & = 0 \\
2511 8x - 7z & = 0%
2512 \end{aligned}\right..$
2513 \end{exercise}
2518 \section{El pla a l'espai}\label{seccio:el-pla-a-lespai}
2520 Per definir un pla a l'espai\index{pla} necessitam tres punts $A$, $B$ i $C$ no alineats, o equivalentment, un punt $A$, per on passa el pla, i dos vectors $\overrightarrow{u}$ i $\overrightarrow{v}$ linealment independents\index{vector!linealment independent}\footnote{L'equivalència, resulta prenent $\overrightarrow{u} = \overrightarrow{OB}-\overrightarrow{OA}$ i $\overrightarrow{v} = \overrightarrow{OC}-\overrightarrow{OA}$.} (la definició de vectors linealment dependents és anàloga a la de línies d'una matriu; vegeu \autoref{def:dependencia-lineal-linies}) --- en el cas de només tenir dos vectors, tendríem infinits plans para\lgem{els}. Els vectors $\overrightarrow{u}$ i $\overrightarrow{v}$ s'anomenen \term{vectors directors} del pla\index{vector!director}.
2522 D'aquesta manera, si $\pi$ és el pla determinat per $A$, i els vectors directors $\overrightarrow{u}$ i $\overrightarrow{v}$, aleshores un punt $P$ que pertanyi a $\pi$ compleix que
2523 \begin{equation*}
2524 \overrightarrow{OP} = \overrightarrow{OA} + \overrightarrow{AP}
2525 \end{equation*}
2526 Però $\overrightarrow{AP}$ és suma dels vectors $\overrightarrow{AM}$ i $\overrightarrow{AN}$, que són múltiples de $\overrightarrow{u}$ i $\overrightarrow{v}$. És a dir,
2527 \begin{equation*}
2528 \begin{split}
2529 \overrightarrow{OP} & = \overrightarrow{OA} + \overrightarrow{AP}\\
2530 & = \overrightarrow{OA} + \overrightarrow{AM} + \overrightarrow{AN}\\
2531 & = \overrightarrow{OA} + \lambda \overrightarrow{u} + \mu \overrightarrow{v},
2532 \end{split}
2533 \end{equation*}
2534 per qualques $\lambda$, $\mu \in \mathbb{R}$ (\autoref{fig:repr-equacions-vectorials-pla-3d}). En resum,
2535 \begin{equation}\label{eq:equacions-vectorials-pla-3d}
2536 \overrightarrow{OP} = \overrightarrow{OA} + \lambda \overrightarrow{u} + \mu \overrightarrow{v},
2537 \end{equation}
2538 amb $\lambda$, $\mu \in \mathbb{R}$. Aquesta equació (\autoref{eq:equacions-vectorials-pla-3d}), s'anomena \term{l'equació vectorial del pla}\index{equació!vectorial!d'un pla}.
2540 \begin{figure}[h!]
2541 \centering
2542 % Generat amb TikZ
2543 % http://tex.stackexchange.com/questions/279917/rotating-axes-in-3d-for-better-viewing-planes
2544 \begin{tikzpicture}[rotate around y=-15, rotate around z=7]
2545 % Nota: els punts tenen coordenades (x,z,y)
2546 \coordinate (O) at (0,0,0);
2547 \coordinate (P0) at (3,2,2);
2548 \coordinate (P) at (5.5,2,4);
2550 % Punts M i N
2551 \coordinate (M) at (5.5,2,2);
2552 \coordinate (N) at (3,2,4);
2554 % Punts dels vectors directors
2556 \coordinate (V1) at (4,2,2);
2557 \coordinate (V2) at (3,2,3.2);
2559 % Punts del pla (a partir de A, P, M i N
2560 \coordinate (PLA0) at (2.3,2,0.9);
2561 \coordinate (PLA1) at (6.5,2,0.9);
2562 \coordinate (PLA2) at (2.3,2,4.5);
2563 \coordinate (PLA3) at (6.5,2,4.5);
2565 % eixos de coordenades
2566 \draw[->] (0,0,0) -- (4,0,0);
2567 \draw[->] (0,0,0) -- (0,3,0);
2568 \draw[->] (0,0,0) -- (0,0,3);
2569 \draw (4,0,0) node[anchor=west] {$x$};
2570 \draw (0,3,0) node[anchor=south] {$z$};
2571 \draw (0,0,3) node[anchor=north east] {$y$};
2573 % Pla
2574 \fill[color=green!10,thick,draw=black] (PLA0) -- (PLA1) -- (PLA3) -- (PLA2) -- cycle;
2575 \draw (PLA1) node[anchor=west] {$\pi$};
2577 % Punts: A, P i vectors de posició i AP
2578 \draw[color=orange,very thick,->,dashed] (O) -- (P0);
2579 \draw[color=green,very thick,->,dashed] (O) -- (P);
2580 \draw[color=red,ultra thick,->] (P0) -- (P);
2581 \draw (P0) node[anchor=south] {$A$};
2582 \draw (P) node[anchor=south] {$P$};
2584 % Llei del paral·lelogram
2585 \draw[thick, dotted,->] (P0) -- (M);
2586 \draw[thick, dotted,->] (P0) -- (N);
2587 \draw[thick, dotted,->] (M) -- (P);
2588 \draw[thick, dotted,->] (N) -- (P);
2591 % Punts M i N i els respectius vectors
2593 \draw[ultra thick,color=blue,->] (P0) -- (V1);
2594 \draw[ultra thick,color=blue,->] (P0) -- (V2);
2595 \draw (M) node[anchor=west] {$M$};
2596 \draw (N) node[anchor=east] {$N$};
2600 \end{tikzpicture}
2601 \caption{Representació de les equacions vectorials d'un pla}
2602 \label{fig:repr-equacions-vectorials-pla-3d}
2603 \end{figure}
2607 \subsection{Equacions paramètriques del pla}
2609 A partir de l'equació vectorial del pla (\autoref{eq:equacions-vectorials-pla-3d}), igualant coordenades s'obtenen les \term{equacions paramètriques del pla}\index{equació!paramètrica!d'un pla}: si $\pi$ és un pla determinat pel punt $A(x_1, y_1, z_1)$ i els vectors directors $\overrightarrow{u}(u_1, u_2, u_3)$ i $\overrightarrow{v}(v_1, v_2, v_3)$, aleshores les equacions paramètriques de $\pi$ són:
2610 \begin{equation}
2611 \left.
2612 \begin{array}{l}
2613 x=x_1+\lambda u_1+\mu v_1 \\
2614 y=y_1+\lambda u_2+\mu v_2 \\
2615 z=z_1+\lambda u_3+\mu v_3%
2616 \end{array}%
2617 \right\},
2618 \end{equation}%
2619 on $\lambda, \mu \in \mathbb{R}$. Recordeu que $\overrightarrow{u}$ i $\overrightarrow{v}$ són vectors no proporcionals entre si.
2621 \begin{claim}Si donam valors qualssevol als paràmetres $\lambda $ i $\mu $ i els substituim a l'expressió anterior, aleshores trobarem punts del pla en qüestió.
2622 \end{claim}
2624 \begin{example}\label{exemple-equacions-parametriques-pla}Sigui $\pi$ el pla que conté el punt $(2,0,-3) $ i que els vectors $(0,3,-1) $ i $(2,5,0) $ (els quals no són proporcionals entre si). Volem trobar dos punts de $\pi$ a més del que ja sabem.
2627 \bigskip
2628 L'equació paramètrica de $\pi$ és%
2629 \begin{equation*}
2630 \left.
2631 \begin{array}{l}
2632 x=2+\lambda \cdot 0+\mu \cdot 2 \\
2633 y=0+\lambda \cdot 3+\mu \cdot 5 \\
2634 z=-3+\lambda \cdot \left( -1\right) +\mu \cdot 0%
2635 \end{array}%
2636 \right\} \text{ i.e., }\left.
2637 \begin{array}{l}
2638 x=2+2\mu \\
2639 y=3\lambda +5\mu \\
2640 z=-3-\lambda%
2641 \end{array}%
2642 \right\}
2643 \end{equation*}
2645 Si donam valors a $\lambda $ i $\mu $ obtenim altres punts del pla:
2646 \begin{itemize}
2647 \item Fent $\lambda = 0$ i $\mu = 1$, obtenim que $x=4$, $y=5$ i $z=-3$. Per tant, $(4,5,-3)$ pertany a $\pi$.
2649 \item Fent $\lambda =1$ i $\mu =0$, obtenim que $x=2$, $y=3$ i $z = -4$. Per tant, $(2,3,-4) \in \pi$.
2650 \end{itemize}
2652 \end{example}
2655 \begin{exercise}Trobeu l'equació del pla que passa pel punt $(0,0,-8)$ i que és para\lgem{el} als vectors $\vec{u}(2,0,-5)$ i $\vec{v}(1,1,9)$. Trobeu tres punts més del pla. Trobeu un vector més que pertanyi al pla (a partir dels punts que heu trobat o bé a partir de combinació lineal de $\overrightarrow{u}$ i $\overrightarrow{v}$).
2656 \end{exercise}
2658 \subsection{Equació general del pla}
2660 El darrer tipus d'equació del pla és l'\term{equació general}\index{equació!general!del pla}, també anomenada \term{equació implítica}\index{equació!implícita!del pla}. L'equació general té la forma
2661 \begin{equation*}
2662 \pi \equiv Ax+By+Cz+D=0,
2663 \end{equation*}
2664 on $A$, $B$, $C$ i $D$ són nombres qualssevol. Per trobar l'equació general d'un pla $\pi$ determinat pels vectors $\vec{u}(u_1, u_2, u_3)$ i $\vec{v}(v_1, v_2, v_3)$ i que passa pel punt $P = (a, b, c)$, es pot emprar la fórmula següent (\autoref{eq:equacio-general-pla}):
2665 \begin{equation}\label{eq:equacio-general-pla}
2666 \left\vert
2667 \begin{array}{rrr}
2668 x-a & u_1 & v_1 \\
2669 y-b & u_2 & v_2 \\
2670 z-c & u_3 & v_3%
2671 \end{array}%
2672 \right\vert =0,
2673 \end{equation}%
2675 \begin{example}
2676 Seguint amb el pla de l'exemple anterior (\autoref{exemple-equacions-parametriques-pla}), tenim que tots els seus punts compleixen l'equació
2677 \begin{equation*}
2678 \pi \equiv \left\vert
2679 \begin{array}{rrr}
2680 x-2 & 0 & 2 \\
2681 y-0 & 3 & 5 \\
2682 z-\left( -3\right) & -1 & 0%
2683 \end{array}%
2684 \right\vert =0
2685 \end{equation*}%
2686 Calculant el determinant,%
2687 \begin{equation*}
2688 \pi \equiv -2y-6\left( z+3\right) +5\left( x-2\right) =0,
2689 \end{equation*}%
2690 és a dir,%
2691 \begin{equation*}
2692 \pi \equiv 5x-2y-6z-28=0
2693 \end{equation*}
2694 \end{example}
2696 \begin{exercise}Trobeu l'equació general del plans:
2697 \begin{enumerate}[label=\emph{\alph*})]
2698 \item El pla $\pi_1$ que té com a vectors directors $\overrightarrow{(3,-2,3)}$ i $\overrightarrow{(1,1,1)}$ i passa pel punt $(2,2,4)$
2699 \item El pla $\pi_2$ que passa pel punt $(0,1,-2)$ i és para\lgem{el} als vectors $\vec{u} = (0,2,4)$ i $\vec{v}=(4,4,2)$
2700 \item El pla que té vectors directors $\vec{u}(1,0,0)$ i $\vec{v}(0,0,1)$ i passa per $(0,2,2)$.
2701 \end{enumerate}
2702 \end{exercise}
2704 \subsubsection{Pas de l'equació general a la paramètrica}
2706 Notem que, si tenim un pla $\pi$ expressat mitjançant una equació paramètrica, aleshores és relativament senzill expressar $\pi$ mitjançant l'equació general. El motiu és que en l'equació paramètrica del pla tenim els vectors directors i un punt de $\pi$. Per tant, simplement aplicarem la fórmula \autoref{eq:equacio-general-pla}. Ara bé, si volem fer le procés invers llavors el procés és més llarg, ja que no {\em veiem} els vectors directors ni els punts per on passa $\pi$ de l'equació general.
2708 Existeixen dues maneres de passar de l'equació general a l'equació paramètrica, que podem usar indistintament.
2710 \paragraph{Mètode 1. Càlcul de punts}
2712 Donat $\pi \equiv Ax + By + Cz + D =0$ un pla qualsevol, si volem trobar la seva equació paramètrica el que hauríem de fer seria:
2713 \begin{enumerate}[label=\emph{\alph*})]
2714 \item Trobar tres punts del pla $P$, $Q$ i $R$
2715 \item Amb aquests punts trobar dos vectors del pla: per exemple $\overrightarrow{PQ}$ i $\overrightarrow{PR}$.
2716 \item Calcular l'equació paramètric amb els vectors anteriors i un punt del pla (per exemple $P$).
2717 \end{enumerate}
2719 \begin{example}Suposem que tenim el pla $\pi \colon 2x-5y -z +3 = 0$ i volem trobar la seva equació paramètrica.
2720 \begin{enumerate}[label=\emph{\alph*})]
2721 \item Trobem tres punts de $\pi$: substituirem qualssevol dues variables per valors que vulguem.
2722 \begin{itemize}
2723 \item Si prenem $y=0$ i $z=1$, tenim que $x=-1$. Per tant, el punt $P(-1,0,1)$ pertany a $\pi$
2724 \item Si prenem $x=0$ i $y=0$, tenim que $z = 3$. Per tant, el punt $Q(0,0,3)$ pertany a $\pi$
2725 \item Si prenem $x=5$ i $z=3$, tenim que $y=2$. Per tant, el punt $R(5,2,3)$ pertany $\pi$
2726 \end{itemize}
2727 \item Trobem dos vectors que pertanyen a $\pi$:
2728 \begin{itemize}
2729 \item $\vec{u} = \overrightarrow{(1,0,2)}$ prenent $P$ i $Q$ com a extrems
2730 \item $\vec{v} = \overrightarrow{(6,2,2)}$ prenent $P$ i $R$ com a extrems
2731 \end{itemize}
2732 \item Calculem l'equació paramètrica de $\pi$ (prenent $P$ com a punt de $\pi$):
2733 \begin{equation*}
2734 \pi \colon \left\{
2735 \begin{array}{l}
2736 x=-1+\lambda + 6 \mu \\
2737 y=2 \mu \\
2738 z=1+2 \lambda +2\mu %
2739 \end{array}%
2740 \right.
2741 \end{equation*}%
2743 \end{enumerate}
2744 \end{example}
2746 \paragraph{Mètode 2. Parametrització de variables}
2748 Donat $\pi \equiv Ax + By + Cz + D =0$ un pla qualsevol, per trobar la seva equació paramètrica procedirem de la manera següent:
2749 \begin{enumerate}[label=\emph{\alph*})]
2750 \item Triarem una variable qualsevol i la aïllarem
2751 \item Parametritzarem les altres dues variables
2752 \item Construirem l'equació paramètrica originada a partir de la informació anterior
2753 \end{enumerate}
2755 \begin{example}Suposem que tenim el pla $\pi \colon 2x-5y -z +3 = 0$ i volem trobar la seva equació paramètrica.
2756 \begin{enumerate}[label=\emph{\alph*})]
2757 \item Triem per exemple la variable $z$ (triem $z$ perquè és més bona d'aïllar, ja que el valor absolut del seu coeficient és 1). De l'equació original, tenim que $z = 3 +2x +5y$.
2758 \item Ara parametritzem les altres dues variables: $x = \lambda$ i $y = \mu$. Per tant, $z = 3 + 2\lambda + 5 \mu$.
2759 \item Si ajuntem les informacions anteriors, tenim que 'equació paramètrica de $\pi$ és:
2760 \begin{equation*}
2761 \pi \colon \left\{
2762 \begin{array}{l}
2763 x=\lambda\\
2764 y=\mu \\
2765 z=3 + 2\lambda + 5 \mu %
2766 \end{array}%
2767 \right.
2768 \end{equation*}%
2770 D'aquí veim que els vectors directors del pla són $\overrightarrow{(1,0,2)}$ i $\overrightarrow{(0,1,5)}$ i un punt de $\pi$ és $(0,0,3)$. Hem de notar que aquesta informació no ha de coincidir amb l'anterior, sinó que han de ser equivalents, ja que existeixen moltes equacions paramètriques equivalents d'un pla.
2772 \end{enumerate}
2773 \end{example}
2775 \subsubsection{Vector normal al pla a partir de l'equació general}\label{sec:subsubseccio:vector-normal-dun-pla}
2777 Donat un pla $\pi$, en aquesta secció trobarem un vector perpendicular a $\pi$ a partir de la seva equació general, el qual anomenarem \term{vector normal}\index{vector!normal!a un pla}.
2779 \begin{definition}[vector normal d'un pla] Donat el pla $\pi \equiv Ax + By + Cz + D = 0$, definirem el seu \term{vector normal}, que denotarem habitualment per $\vec{n}_\pi$ (o simplement $\vec{n}$), com
2780 \begin{equation*}
2781 \vec{n} = (A, B, C)
2782 \end{equation*}
2783 \end{definition}
2785 \begin{proposition}[perpendicularitat del vector normal]\label{proposicio-perpendicularitat-vector-normal} Donat un pla qualsevol $\pi$, $\vec{n}$ és perpendicular a $\pi$, és a dir, el vector normal sempre és perpendicular al seu pla (\autoref{fig:vector-normal-de-pla}).
2786 \end{proposition}
2788 \definecolor{qqttzz}{rgb}{0.,0.2,0.6}
2789 \definecolor{qqttqq}{rgb}{0.,0.2,0.}
2790 \definecolor{qqzzff}{rgb}{0.,0.6,1.}
2792 \begin{figure}[h!]
2793 \centering
2794 % Generat amb Geogebra. De http://tube.geogebra.org/material/show/id/21240
2796 \begin{tikzpicture}[line cap=round,line join=round,>=triangle 45,x=1.0cm,y=1.0cm,scale=0.44]
2797 \fill[color=qqttqq,fill=qqttqq,fill opacity=0.25] (16.314369994233928,2.1735516098406857) -- (8.829139829711197,0.7802834311953475) -- (15.952096881232668,-4.784102422268202) -- (23.330354512532672,-2.8046265446364393) -- cycle;
2798 \draw [color=qqzzff] (16.319127530674148,4.946870904020575)-- (16.309553188347557,-3.295520534200174);
2799 \draw [color=qqzzff] (9.160829444220798,-7.088913975103355)-- (16.309553188347557,-3.295520534200174);
2800 \draw [color=qqzzff] (16.309553188347557,-3.295520534200174)-- (23.171986395382174,-7.266980322667587);
2801 \draw [color=qqttqq] (16.314369994233928,2.1735516098406857)-- (8.829139829711197,0.7802834311953475);
2802 \draw [color=qqttqq] (8.829139829711197,0.7802834311953475)-- (15.952096881232668,-4.784102422268202);
2803 \draw [color=qqttqq] (15.952096881232668,-4.784102422268202)-- (23.330354512532672,-2.8046265446364393);
2804 \draw [color=qqttqq] (23.330354512532672,-2.8046265446364393)-- (16.314369994233928,2.1735516098406857);
2805 \draw [->,line width=2.8pt,color=qqttzz] (16.14782106613868,-0.8919428656307156) -- (17.130804935635858,2.539692755216607);
2806 \draw (22.219830619680312,-6.819780943114806) node[anchor=north west] {$x$};
2807 \draw (8.709421440136298,-7.04495442944054) node[anchor=north west] {$y$};
2808 \draw (15.070572428838272,6.127694520614853) node[anchor=north west] {$z$};
2809 \draw (8.371661210647698,0.7235308487972565) node[anchor=north west] {$\pi$};
2810 \draw (18.33558798056141,2.7500922257288556) node[anchor=north west] {$\overrightarrow{n}$};
2811 \end{tikzpicture}
2814 \caption{Vector normal a un pla}
2815 \label{fig:vector-normal-de-pla}
2816 \end{figure}
2818 \begin{demonstration}Basta veure que el vector normal és perpendicular a dos vectors que estan continguts al pla $\pi$, és a dir, els seus vectors directors.
2820 Per trobar els vectors directors del pla, calcularem tres punts del pla:
2821 \begin{itemize}
2822 \item Prenem $x=0$ i $y=0$, per tant, $z = \frac{-D}{C}$. Per tant, tenim que el punt $A(0,0,\frac{-D}{C})$ pertany a $\pi$
2823 \item Prenem $y=0$ i $z=0$, per tant, $x = \frac{-D}{A}$. Per tant, tenim que el punt $B(\frac{-D}{A},0,0)$ pertany a $\pi$
2824 \item Prenem $x=0$ i $z=0$, per tant, $y = \frac{-D}{B}$. Per tant, tenim que el punt $C(0,\frac{-D}{B},0)$ pertany a $\pi$
2825 \end{itemize}
2826 Clarament aquests punts no estan alineats.
2828 Per tant, els vectors $\overrightarrow{AB} = \overrightarrow{(\frac{-D}{A}, 0, \frac{D}{C})}$ i $\overrightarrow{AC} = \overrightarrow{(0, \frac{-D}{B}, \frac{D}{C})}$ estan a $\pi$. I, per tant, també ho estan $\vec{u} = \overrightarrow{(-DC, 0, DA)}$ i $\vec{v} = \overrightarrow{(0, -DC, DB)}$, que poden ser els vectors directors de $\pi$.
2830 És trivial veure que $\vec{n} = (A, B, C)$ és perpendicular a $\vec{u}$ i $\vec{v}$, calculant el producte escalar corresponent.
2831 \end{demonstration}
2833 \begin{example}Un vector perpendicular al pla $\pi \colon 5x-2y-6z-28 = 0$ és el vector $\vec{n}\left( 5,-2,-6\right) $.
2834 \end{example}
2836 \begin{exercise}Trobeu el vector normal al pla $x-5y+8z+4=0$.
2837 \end{exercise}
2839 L'aplicació inversa del vector normal d'un pla és trobar un pla que és perpendicular a un vector donat. És a dir, donat $\vec{u}$ un vector, trobar un pla $\pi$ tal que $\pi$ és perpendicular a $\vec{u}$. Sabem que un pla perpendicular a $\vec{u}$ serà aquell que tengui $\vec{u}$ com al seu vector normal. Tot seguit, veurem un exemple.
2841 \begin{example}Volem trobar un pla $\pi$ perpendicular al vector $\overrightarrow{v}\left(3,-1,7\right)$ i que passi pel punt $A(2,-4,0)$.
2843 \begin{solution*}
2844 Un pla perpendicular al vector $\overrightarrow{v}\left( 3,-1,7\right)$, és aquell que té l'equació general de la forma%
2845 \begin{equation*}
2846 \pi \equiv 3x-y+7z+D=0,
2847 \end{equation*}%
2848 ja que tendria $\vec{u}$ com al seu vector normal (vegeu \autoref{proposicio-perpendicularitat-vector-normal}).
2850 D'altra banda, sabem que el punt $A$ és de $\pi$, per tant, compleix les equacions del pla. Llavors:
2851 \begin{equation*}
2852 3\cdot 2-(-4)+7\cdot 0+D=0
2853 \end{equation*}%
2854 D'aquí es té que $D=-10$, i l'equació del pla és%
2855 \begin{equation*}
2856 \pi \equiv 3x-y+7z-10=0
2857 \end{equation*}
2858 \end{solution*}
2859 \end{example}
2861 \begin{exercise}Donat el pla $\pi \colon 3x - 6y + 2z -1 = 0$, trobeu la recta perpendicular $r$ a $\pi$ que passa pel punt $A(1,2,0)$. Trobeu un punt de $r$ que no sigui $A$.
2862 \end{exercise}
2864 \begin{exercise}Donada la recta $r \colon \frac{x-1}{2} = \frac{y+2}{3} = \frac{z-1}{0}$, trobeu el pla $\pi$ perpendicular a $r$ que passa per $A(1,0,-1)$.
2865 \end{exercise}
2867 \begin{exercise}Donat el pla $\pi$ que té com a vectors directors $\vec{u}(2,0,-1)$ i $\vec{v}(0,2,1)$ i que passa per $A(1,0,1)$, trobeu la recta perpendicular a $\pi$ que passa per $A$.
2868 \end{exercise}
2870 \begin{claim}Hem vist abans una fórmula per a calcular el vector director d'una recta a partir de les seves equacions implícites (\autoref{prop:v-d-a-partir-eq-implicita-3d}). És el moment de justificar aquest resultat amb l'ús dels vectors normals: notem que si $r$ és una recta donada per les seves equacions implícites
2871 \begin{equation*}
2872 \left\{
2873 \begin{aligned}
2874 Ax+By+Cz+D & = 0 \\
2875 A^{\prime}x+B^{\prime}y+C^{\prime}z+D^{\prime} & = 0%
2876 \end{aligned}%
2877 \right.,
2878 \end{equation*}%
2879 amb $A$, $B$, $C$, $D$, $A'$, $B'$, $C'$ i $D' \in \mathbb{R}$, aleshores el seu vector director $\overrightarrow{v_r}$ és el vector ortogonal als vectors normals dels plans que defineixen la recta $r$.
2881 És a dir, podem veure la recta $r$ com la intersecció de dos plans $\pi$ i $\rho$ d'equacions $\pi \colon Ax+By+Cz + D = 0$ i $\rho \colon A'x+B'y+C'z + D' = 0$. En aquest sentit, $\overrightarrow{v_r}$ tendrà la mateixa direcció que $\overrightarrow{n_\pi} \wedge \overrightarrow{n_\rho}$, on $\overrightarrow{n_\pi}$ i $\overrightarrow{n_\rho}$ són els vectors normals dels plans $\pi$ i $\rho$ respectivament (\autoref{fig:repr-vector-director-a-partir-plans}).
2883 \begin{figure}[h!]
2884 \centering
2885 % Generat amb TikZ
2886 % De http://tex.stackexchange.com/a/20009/61233
2887 \begin{tikzpicture}[x={(240:0.8cm)}, y={(-10:1cm)}, z={(0,1cm)},
2888 plane max z=3]
2889 %\draw[->, red] (0,0,0) -- (3,0,0);
2890 %\draw[->, yellow] (0,0,0) -- (0,3,0);
2891 %\draw[->, green] (0,0,0) -- (0,0,3);
2893 % Plane of equation 1x+1.5y+0z = 2
2894 \definePlaneByEquation{myplane}{1}{1.5}{0}{2}
2895 \drawPlane[thick,fill=blue]{myplane}
2897 % Plane of equation -4z = 0
2898 % It is determined by the four points:
2899 % (2,0,0), (0,4/3,0), (a,4/3+1.5a,0), (2+a,1.5a,0)
2900 % a \in \mathbb{R}
2901 %\definePlaneByEquation{myplane2}{0}{0}{-4}{0}
2902 %\drawPlane[thick,fill=green]{myplane2}
2904 \filldraw[color=green!80,thick,draw=black] (2,0,0) -- (0,1.3333333,0)--(1,2.8333333,0) -- (3,1.5,0) -- cycle;
2906 % Intersection line
2907 \draw[ultra thick] (2.75,-0.5,0) -- (-1,2,0);
2908 \draw (-1,2,0) node[anchor=south] {$r$};
2910 % Normal vectors: (-4,-6,0) and (0,0,-4)
2911 %% Medium point of (2,0,0) and (1,4/6,0)
2912 %\draw (1,0.66666,0) circle (2pt);
2913 %% + (0.5, 0.5,0);
2914 %\draw (1.5,1.16666,0) circle (2pt);
2915 %% Plus normal director (0,0,4)
2916 \draw[->,thick] (1.5,1.16666,0) -- (1.5,1.16666,1);
2917 %% Name of the normal vector
2918 \draw (1.5,1.16666,0.5) node[anchor=west] {$n_\pi$};
2920 %% The name of the plane
2921 \draw (0.5,2.08333333,0) node[anchor=west] {$\pi$};
2924 %% Medium point of (2,0,0) and (1,4/6,0) elevated +2
2925 %\draw (1.5,1.166666,2) circle (2pt);
2926 %% Plus normal director 1.2*(1,1.5,0)
2927 \draw[->,thick] (1.5,1.166666,2) -- (2.2,2.133333,2);
2928 %% Name of the normal vector
2929 \draw (1.85,1.649999,2) node[anchor=east] {$n_\rho$};
2932 %% The name of the plane 2
2933 \draw (2,0,2) node[anchor=east] {$\rho$};
2935 \end{tikzpicture}
2936 \caption{Vector director d'una recta com a producte vectorial dels vectors normals dels plans que la defineixen}
2937 \label{fig:repr-vector-director-a-partir-plans}
2938 \end{figure}
2939 \end{claim}
2941 \subsection{Plans para\lgem{els}}
2943 Donat un pla $\pi_0$, un pla \term{para\lgem{el}}\index{plans!para\lgem{els}} $\pi_1$ és un pla tal que té els {\em mateixos} vectors directors\footnote{Realment, no tenen perquè ser els mateixos vectors directors. En general, dos plans són para\lgem{els} quan els vectors directors de primer són combinació lineal dels vectors directors de segon i inversament. Ara bé, sempre podríem triar els mateixos vectors directors per als dos plans.} i passa per un punt que no pertany a $\pi_0$ (en el cas que passàs per un punt de $\pi_0$, els plans serien \term{coincidents}\index{plans!coincidents}, és a dir, el mateix; vegeu \autoref{seccio:posicio-relativa-plans-i-rectes}). Per tant, podem trobar les equacions paramètriques o generals de $\pi_1$, segons convengui, amb la informació proporcionada.
2946 \begin{example}Trobeu l'equació del pla para\lgem{el} a $\pi :3x-y+z-8=0$
2947 que passa pel punt $A(-2,-2,6)$.
2949 Si el pla que cercam és papa\lgem{el} al pla $\pi$, ambdós tendran el mateix vector normal. Per tant, la seva equació serà de la forma
2950 \begin{equation*}
2951 3x-y+z+D=0,
2952 \end{equation*}
2953 amb $D$ un nombre. Com que el pla que volem trobar passa pel punt $A$, llavors $A$ complirà l'equació del pla anterior:%
2954 \begin{equation*}
2955 3\cdot (-2)-(-2)+6+D=0;
2956 \end{equation*}%
2957 Per tant, $D=-2$. Llavors el pla que cercam és $3x-y+z-2=0$.
2958 \end{example}
2960 \begin{exercise}Trobeu l'equació del pla que passa pel punt $A(5,-1,0)$ i és para\lgem{el} al pla $-x+3y-8=0$.
2961 \end{exercise}
2963 \begin{exercise}Trobeu un pla para\lgem{el} a $\pi \colon -2x+3y+z-5=0$ que contengui a la recta $r \colon \frac{x-10}{3} = \frac{y+5}{-2} = \frac{z}{12}$
2964 \end{exercise}
2966 \begin{exercise}Determineu si els plans $\pi_1$ i $\pi_2$ són para\lgem{els}, on $\pi_1 \colon 3x - 2y + z + 1 = 0$ i $\pi_2$ està determinat pels vectors $\vec{u}(1,2,0)$, $\vec{v}(-1, 0, -1)$ i passa per $A(0,1,0)$. En cas contrari, trobeu-ne un de para\lgem{el} a cadascun d'ells.
2967 \end{exercise}
2970 \section{Posició relativa entre rectes i plans}\label{seccio:posicio-relativa-plans-i-rectes}
2972 \subsection{Posició relativa entre dues rectes}
2974 La \term{posició relativa entre dues rectes}\index{posició relativa!entre dues rectes} simplement és la com estan situades una respecte de l'altra. En aquest sentit, tenim que dues rectes poden tenir quatre posicions relatives entre elles:
2975 \begin{itemize}
2976 \item \term{coincidents}\index{rectes!coincidents}, quan són la mateixa recta
2977 \item \term{secants}\index{rectes!secants}, quan les rectes es tallen en un punt; és a dir, tenen un punt en comú.
2978 \item \term{para\lgem{eles}}\index{rectes!para\lgem{eles}}, quan no es tallen mai i tenen els seus vectors directors linealment dependents, és a dir, proporcionals.
2979 \item quan \term{es creuen}\index{rectes!que es creuen}, quan no es tallen mai i tenen el seus vectors linealment independents.
2980 \end{itemize}
2982 \medskip
2983 Existeixen dues proposicions sobre la posició relativa entre dues rectes, les quals són equivalents en termes teòrics.
2985 \begin{proposition}[posició relativa de dues rectes usant proporcionalitat de vectors] Donades dues rectes $r$ i $s$ amb vectors directors $\vec{u}=(u_1, u_2, u_3)$ i $\vec{w}=(w_1, w_2, w_3)$ i que passen pels punts $A=(a_1, a_2, a_3)$ i $B = (b_1, b_2, b_3)$ respectivament, tenim que la seva posició relativa es pot determinar amb l'arbre de decisió següent:
2986 \begin{itemize}
2987 \item Si $\vec{u}$ és proporcional a $\vec{w}$ (vegeu \autoref{resultat:proposicio-parallelisme-vectors}), aleshores les rectes poden ser coincidents o para\lgem{eles}.
2988 \begin{itemize}
2989 \item Si $\overrightarrow{AB}$ és proporcional a $\vec{u}$ (i per tant a $\vec{w}$), llavors $r$ i $s$ són coincidents
2990 \item Si $\overrightarrow{AB}$ no és proporcional a $\vec{u}$ (i per tant tampoc a $\vec{w}$), llavors $r$ i $s$ són para\lgem{eles}
2991 \end{itemize}
2992 \item Sinó, llavors les rectes són secants o bé es creuen.
2993 \begin{itemize}
2994 \item Si $\overrightarrow{AB}$ és propocional a $\vec{u}$ o a $\vec{w}$, llavors $r$ i $s$ són secants
2995 \item Si $\overrightarrow{AB}$ no és proporcional ni a $\vec{u}$ ni a $\vec{w}$, llavors $r$ i $s$ es creuen
2996 \end{itemize}
2997 \end{itemize}
2998 \end{proposition}
3000 \begin{proposition}[posició relativa de dues rectes usant matrius] Donades dues rectes $r$ i $s$ amb vectors directors $\vec{u}=(u_1, u_2, u_3)$ i $\vec{w}=(w_1, w_2, w_3)$ i que passen pels punts $A=(a_1, a_2, a_3)$ i $B = (b_1, b_2, b_3)$ respectivament, tenim que la seva posició relativa ve donada pels rangs següents:
3001 \begin{itemize}
3002 \item $rg \left(
3003 \begin{array}{rr}
3004 u_1 & w_1 \\
3005 u_2 & w_2 \\
3006 u_3 & w_3 %
3007 \end{array}%
3008 \right) =1$ i $rg \left(
3009 \begin{array}{rrr}
3010 u_1 & w_1 & b_1 -a_1\\
3011 u_2 & w_2 & b_2 -a_2\\
3012 u_3 & w_3 & b_3 -a_3%
3013 \end{array}%
3014 \right) =1$, $r$ i $s$ són coincidents
3015 \item $rg \left(
3016 \begin{array}{rr}
3017 u_1 & w_1 \\
3018 u_2 & w_2 \\
3019 u_3 & w_3 %
3020 \end{array}%
3021 \right) =1$ i $rg \left(
3022 \begin{array}{rrr}
3023 u_1 & w_1 & b_1 -a_1\\
3024 u_2 & w_2 & b_2 -a_2\\
3025 u_3 & w_3 & b_3 -a_3%
3026 \end{array}%
3027 \right) =2$, $r$ i $s$ són para\lgem{eles}
3028 \item $rg \left(
3029 \begin{array}{rr}
3030 u_1 & w_1 \\
3031 u_2 & w_2 \\
3032 u_3 & w_3 %
3033 \end{array}%
3034 \right) =2$ i $rg \left(
3035 \begin{array}{rrr}
3036 u_1 & w_1 & b_1 -a_1\\
3037 u_2 & w_2 & b_2 -a_2\\
3038 u_3 & w_3 & b_3 -a_3%
3039 \end{array}%
3040 \right) =2$, $r$ i $s$ es tallen
3041 \item $rg \left(
3042 \begin{array}{rr}
3043 u_1 & w_1 \\
3044 u_2 & w_2 \\
3045 u_3 & w_3 %
3046 \end{array}%
3047 \right) =2$ i $rg \left(
3048 \begin{array}{rrr}
3049 u_1 & w_1 & b_1 -a_1\\
3050 u_2 & w_2 & b_2 -a_2\\
3051 u_3 & w_3 & b_3 -a_3%
3052 \end{array}%
3053 \right) =3$, $r$ i $s$ es creuen
3055 \end{itemize}
3056 \end{proposition}
3058 \begin{exercise}Determineu la posició relativa d'aquestes rectes:
3059 \begin{enumerate}[label=\emph{\alph*})]
3061 \item $r \colon \left\{
3062 \begin{array}{l}
3063 x = \lambda \\
3064 y = -5 + \lambda \\
3065 z = 3 + \lambda
3066 \end{array}%
3067 \right.$ i $s \colon \frac{x-3}{2} = \frac{y}{2} = \frac{z+2}{2}$
3068 \item $r \colon \left\{
3069 \begin{array}{l}
3070 x = 2 + \lambda \\
3071 y = 1 + \lambda \\
3072 z = -1 - 3\lambda
3073 \end{array}%
3074 \right.$ i $s \colon \frac{x-2}{2} = \frac{y-1}{1} = \frac{z+1}{5}$
3075 \item $r \colon x = y-1 = \frac{z+2}{3}$ i $s \colon \frac{x-1}{2} = \frac{y-2}{2} = \frac{z+4}{6}$
3076 \item $r \colon \left\{
3077 \begin{array}{l}
3078 x = 2 + \lambda \\
3079 y = 1 + \lambda \\
3080 z = -1 - 3\lambda
3081 \end{array}%
3082 \right.$ i $s \colon \left\{
3083 \begin{array}{r}
3084 2x + 3y + 4z = 1\\
3085 x -2y + z = 10
3086 \end{array}%
3087 \right.$
3088 \item $r \colon \left\{
3089 \begin{array}{r}
3090 -y - z = 3\\
3091 2x -z = -1
3092 \end{array}%
3093 \right.$ i $s \colon \left\{
3094 \begin{array}{r}
3095 -y + z = 0\\
3096 x -3y + z = 1
3097 \end{array}%
3098 \right.$
3100 \end{enumerate}
3101 \end{exercise}
3103 \subsection{Posició relativa d'una recta i un pla}
3105 Per a determinar la posició relativa entre un pla i una recta\index{posició relativa!entre un pla i una recta} tenim dues opcions: estudiar la posició a partir de les equacions implícita de pla i recta o bé estudiar-la a partir de les posicions relatives entre el vector normal del pla i el vector director de la recta. Els dos estudis es resumeixen a les proposicions següents.
3107 La posició relativa d'una recta i un pla pot ser:
3108 \begin{itemize}
3109 \item la recta i el pla són \term{secants}, és a dir, es tallen a un únic punt\index{recta!secant a un pla}
3110 \item la recta és \term{para\lgem{ela}} al pla\index{recta!para\lgem{ela} a un pla}; en aquest cas, el vector director de la recta és linealment depenents als vectors directors del pla però cap punt de la recta pertany al pla (i vice-versa)
3111 \item la recta està \term{continguda} al pla\index{recta!continguda a un pla}; tots els punts de la recta pertanyen al pla.
3112 \end{itemize}
3114 \begin{proposition}[posició relativa entre pla i recta (versió eq. implítica)] Siguin $\pi \equiv A_1x + B_1y + C_1z + D_1 = 0$ i $r \equiv \left\{
3115 \begin{array}{r}
3116 A_2x + B_2y + C_2z + D_2 = 0\\
3117 A_3x + B_3y + C_3z + D_3= 0
3118 \end{array}%
3119 \right.$ un pla i una recta de l'espai qualssevol. Podem construir les matrius de $A$ i $M$:
3120 \begin{equation*}
3121 A = \left( \begin{array}{rrr}
3122 A_1 & B_1 & C_1 \\
3123 A_2 & B_2 & C_2 \\
3124 A_3 & B_3 & C_3%
3125 \end{array}%
3126 \right) \quad M = \left( \begin{array}{rrrr}
3127 A_1 & B_1 & C_1 & D_1 \\
3128 A_2 & B_2 & C_2 & D_2\\
3129 A_3 & B_3 & C_3 & D_3%
3130 \end{array}%
3131 \right)
3132 \end{equation*}
3133 Aleshores:
3134 \begin{itemize}
3135 \item Si $rg A = 3$ i $rg M = 3$, llavors el sistema és compatible determinat. Per tant, la recta talla al pla
3136 \item Si $rg A = 2$ i $rg M = 2$, llavors el sistema és compatible indeterminat. Per tant, la recta està continguda en el pla
3137 \item Si $rg A = 2$ i $rg M = 3$, llavors el sistema és incompatible. Per tant, la recta i el pla són para\lgem{els}
3138 \end{itemize}
3139 \end{proposition}
3141 \begin{proposition}[posició relativa entre pla i recta (versió vector normal)] Siguin $\pi$ un pla i $r$ una recta del pla. I sigui $\vec{v}$ el vector director de $r$ i $A$ un punt de la recta. Aleshores:
3142 \begin{itemize}
3143 \item Si $\vec{v}$ no és perpendicular a $\vec{n}$, llavors la recta i el pla es tallen
3144 \item Si $\vec{v}$ és perpendicular al vector normal del pla $\vec{n}$, la recta pot ser para\lgem{ela} o bé està continguda al pla:
3145 \begin{itemize}
3146 \item Si $A \in \pi$, llavors $r$ està continguda a $\pi$
3147 \item Si $A \not \in \pi$, llavors $r$ és para\lgem{ela} a $\pi$
3148 \end{itemize}
3149 \end{itemize}
3150 \end{proposition}
3152 \begin{example}Determineu la posició relativa de $r \colon \left\{
3153 \begin{array}{r}
3154 2x + y + 2= 0\\
3155 2x - z + 1= 0
3156 \end{array}%
3157 \right.$ i $\pi \colon x + y -z + 3 = 0$.
3159 \begin{solution*} Farem servir la primera proposició.
3160 \begin{itemize}
3161 \item Primer trobam les matrius de coeficients i ampliada:
3162 \begin{equation*}
3163 A = \left( \begin{array}{rrr}
3164 2 & 1 & 0 \\
3165 2 & 0 & -1 \\
3166 1 & 1 & -1%
3167 \end{array}%
3168 \right) \quad M = \left( \begin{array}{rrrr}
3169 2 & 1 & 0 & 2 \\
3170 2 & 0 & -1 & 1\\
3171 1 & 1 & -1 & 3%
3172 \end{array}%
3173 \right)
3174 \end{equation*}
3175 \item Calculem el rang de cada matriu. Després d'alguns càlculs tenim que $rg A = 3$ i $rg M = 3$. Per tant, el sistema és compatible determinat i, per tant, el pla i la recta són secants.
3176 \end{itemize}
3177 \end{solution*}
3178 \end{example}
3180 \begin{example}Siguin $r \equiv \frac{x-1}{3} = \frac{y-2}{9} = \frac{z}{6}$ i $\pi \colon 2x-4y+5z = 0$. Determineu la seva posició relativa.
3182 \begin{solution*}Farem servir la segona proposició.
3183 \begin{itemize}
3184 \item Tenim que el vector director de $r$, $\vec{v}(3,9,6)$, i el vector normal del pla, $\vec{n}(2,-4,5)$, són perpendiculars: $\vec{v} \cdot \vec{n} = 0$. Per tant, la regla pot ser continguda al pla o bé para\lgem{ela}.
3185 \item Per discriminar, vegem si el pla conté o no un punt de la recta: sabem que $A(1,2,0) \in r$. Però si substituïm a les equacions de $\pi$, tenim que: $2 \cdot 1 - 4 \cdot 2 + 5 \cdot 0 = -6 \neq 0$. Per tant, $r$ és para\lgem{ela} a $\pi$.
3186 \end{itemize}
3187 \end{solution*}
3188 \end{example}
3190 \begin{exercise}Determineu les posicions relatives entre aquests plans i aquestes rectes:
3191 \begin{enumerate}[label=\emph{\alph*})]
3192 % Javier Sánchez
3193 \item $r \colon \left\{
3194 \begin{array}{r}
3195 5x -3y + 2z -5= 0\\
3196 2x -y - z - 1= 0
3197 \end{array}%
3198 \right.$ i $\pi \equiv 4x-3y+7z-7=0$
3199 \item $r \colon \left\{
3200 \begin{array}{r}
3201 x +y - z +2= 0\\
3202 -x +3y - z + 1= 0
3203 \end{array}%
3204 \right.$ i $\pi \equiv x+z+1=0$
3205 \item $r \colon x = \frac{y+2}{2} = \frac{z-1}{-1}$ i $\pi \equiv 2x-5y+3z+3=0$
3206 \item $r \colon \left\{
3207 \begin{array}{r}
3208 2x +y - 2z +2= 0\\
3209 -x +3y - z + 1= 0
3210 \end{array}%
3211 \right.$ i $\pi \equiv x+4y-3z+3=0$
3212 \end{enumerate}
3213 \end{exercise}
3215 \begin{exercise}Trobeu el valor de $\alpha$ per a què la recta $r \equiv \frac{x-1}{6} = \frac{y-2}{\alpha} = \frac{z}{12}$ no talli al pla $\pi \colon 2x-4y+5z = 0$.
3216 \end{exercise}
3218 \begin{exercise}Donats el pla $\pi \colon x+y+mz = 1$ i la recta $r \colon \frac{x}{-1} = \frac{y-2}{-1} = \frac{z}{3}$, discutiu quina és la posició relativa de $r$ i $\pi$ segons els valors de $m$.
3219 \end{exercise}
3221 \subsection{Posició relativa entre dos plans}
3223 Finalment estudiarem la posició relativa entre dos plans\index{posició relativa!entre dos plans}. Suposarem que els plans sempre vénen donats mitjançant les seves equacions generals.
3225 Les posicions relatives entre dos plans poden ser:
3226 \begin{itemize}
3227 \item els plans són \term{coincidents}\index{plans!coincidents}; són el mateix pla
3228 \item els plans són \term{para\lgem{els}}\index{plans!para\lgem{els}}; no tenen cap punt en comú
3229 \item els plans són \term{secants}\index{plans!secants}; es tallen i tenen en comú una recta
3230 \end{itemize}
3232 \begin{proposition}[posició relativa entre dos plans (versió rangs de matrius)] Siguin $\pi_1 \equiv A_1 x + B_1 y + C_1 z + D_1 = 0$ i $\pi_2 \equiv A_2 x + B_2 y + C_2 z + D_2 = 0$ dos plans qualssevol. Considerem les matrius de coeficients i ampliada, $A$ i $M$ respectivament:
3233 \begin{equation*}
3234 A = \left( \begin{array}{rrr}
3235 A_1 & B_1 & C_1 \\
3236 A_2 & B_2 & C_2 %
3237 \end{array}%
3238 \right) \quad M = \left( \begin{array}{rrrr}
3239 A_1 & B_1 & C_1 & D_1 \\
3240 A_2 & B_2 & C_2 & D_2 %
3241 \end{array}%
3242 \right)
3243 \end{equation*}
3244 Llavors:
3245 \begin{itemize}
3246 \item Si $rg A = 2$, llavors $rg M = 2$ i, per tant, el sistema és compatible simplement indeterminat (2 equacions i 3 incògnites). Això significa que els plans es tallen
3247 \item Si $rg A = 1$:
3248 \begin{itemize}
3249 \item Si $rg M = 2$, llavors el sistema és incompatible. Per tant, no tenen punts en comú. Llavors els plans són para\lgem{els}
3250 \item Si $rg M = 1$, llavors el sistema és compatible doblement indeterminat (1 equació i 3 incògnites). Això significa que els plans són coincidents.
3251 \end{itemize}
3252 \end{itemize}
3253 \end{proposition}
3255 Disposem d'una proposició que estudia la posició relativa a partir de la proporcionalitat dels seus vectors normals.
3256 \begin{proposition}[posició relativa entre dos plans (versió vectors normals)] Siguin $\pi_1 \equiv A_1 x + B_1 y + C_1 z + D_1 = 0$ i $\pi_2 \equiv A_2 x + B_2 y + C_2 z + D_2 = 0$ dos plans qualssevol. Considerem $\vec{n}_1 = (A_1, B_1, C_1)$ i $\vec{n}_2 = (A_2, B_2, C_2)$ els seus vectors normals.
3257 \begin{itemize}
3258 \item Si $\vec{n}_1$ {\em no} és proporcional a $\vec{n}_2$, llavors els plans es tallen
3259 \item En cas contrari, llavors els plans són o bé coincidents o bé para\lgem{els}.
3261 Com que $\vec{n}_1$ i $\vec{n}_2$ són proporcionals, llavors $\frac{A_1}{A_2} = \frac{B_1}{B_2} = \frac{C_1}{C_2}$ és igual a un nombre. Diguem-li $R$.
3262 \begin{itemize}
3263 \item Si $\frac{D_1}{D_2} = R$, llavors $\pi_1$ és coincident amb $\pi_2$
3264 \item Sinó, $\pi_1$ és para\lgem{el} a $\pi_2$
3265 \end{itemize}
3266 \end{itemize}
3267 \end{proposition}
3270 \begin{exercise} Determineu la posició relativa dels plans següents:
3271 \begin{enumerate}[label=\emph{\alph*})]
3272 \item $\pi_1 = 3x-2y +4z = 2$ i $\pi_2 = 2x+3y-5z = -8$
3273 \item $\pi_1 = -x+2y -z = 0$ i $\pi_2 = x-2y+z = 0$
3274 \item $\pi_1 = x-z = -11$ i $\pi_2 = -2y-z = -11$
3275 \item $\pi_1 = 3x-2y +4z = 2$ i $\pi_2 = \left\{
3276 \begin{array}{l}
3277 x = 1 - \lambda + \mu\\
3278 y = \lambda + 2\mu\\
3279 z = -1 + \mu %
3280 \end{array}%
3281 \right.$
3282 \end{enumerate}
3283 \end{exercise}
3285 \section{Càlcul de la intersecció entre rectes i plans}
3287 En aquesta secció calcularem la intersecció entre rectes, plans i rectes i plans. No determinarem abans la seva posició relativa. Calcularem els punts (o rectes) d'intersecció {\em a pèl}.
3289 \subsection{Intersecció entre dos plans}
3291 De forma general, la intersecció entre dos plans és una recta, ja que les equacions generals de la recta no són res més que un sistema format per les equacions generals de dos plans.
3293 Vegem amb un exemple com trobar la intersecció entre una recta i un pla.
3295 \begin{example}Trobeu l'equació de la recta determinada per la intersecció dels plans $\pi_{1}\colon x-2y+z+3=0$ i $\pi_{2} \colon 2x-y+z-4=0$.
3297 La recta que cercam (veure (\autoref{subseccio:equacio-implicita-recta}) és%
3298 \begin{equation*}
3299 r:\left\{
3300 \begin{array}{c}
3301 x-2y+z+3=0 \\
3302 2x-y+z-4=0%
3303 \end{array}%
3304 \right. ,
3305 \end{equation*}%
3306 que, com es veu, ve definida per la intersecció de dos plans (si volem es pot passar a forma paramètrica (\autoref{subseccio:pas-de-implica-a-parametrica-js}).
3307 \end{example}
3309 \begin{exercise}Trobeu la intersecció entre els plans següents:
3310 \begin{enumerate}[label=\emph{\alph*})]
3311 \item $\pi_1 \colon x + 3y + z - 5=0$ i $\pi_2 \colon x - 2z - 2 =0$
3312 \item $\pi_1 \colon x + 2y + 3z - 5=0$ i $\pi_2 \colon x + 2y - 3z -2 = 0$
3313 \item $\pi_1 \colon 2x + y + 5z =0$ i $\pi_2 \colon y - 2z - 10 = 0$
3314 \item $\pi_1 \colon 2x + 4y + 8z - 10=0$ i $\pi_2 \colon 2x + 4y -8z - 10 = 0$
3315 \end{enumerate}
3316 \end{exercise}
3318 \subsection{Intersecció entre dues rectes}
3320 En general, la manera més senzilla de trobar el punt d'intersecció entre dues rectes és passar una de les rectes a forma paramètrica i substituir les equacions (paramètriques o no de l'altra)\footnote{D'aquesta manera evitarem resoldre, per exemple, sistemes d'equacions de 3 incògnites i 4 equacions en el cas en què les rectes estiguin expressades usant les equacions generals.}.
3322 \begin{example}Calculeu el punt d'intersecció entre les rectes $r \colon \left\{ x-2y+z+3=0,\right.$ $\left. 2x-y+z-4=0 \right\}$ i $s \colon \left\{ 2x-2y+3z-19=0, x-y+z-4=0\right\}$.
3324 \begin{solution*}\
3325 \begin{itemize}
3326 \item Passam en primer lloc la recta $r$ a forma paramètrica:%
3327 \begin{equation*}
3328 r:\left\{
3329 \begin{array}{l}
3330 x=-\lambda \\
3331 y=7+\lambda \\
3332 z=11+3\lambda%
3333 \end{array}%
3334 \right.
3335 \end{equation*}%
3336 \item Substituïm ara aquestes expressions a les equacions de $s$, i ens queda el sistema%
3337 \begin{equation*}
3338 \left.
3339 \begin{array}{r}
3340 2(-\lambda )-2(7+\lambda )+3(11+3\lambda )-19=0 \\
3341 -\lambda -(7+\lambda )+(11+3\lambda )-4=0%
3342 \end{array}%
3343 \right\} ,
3344 \end{equation*}%
3345 la solució del qual és, per a cadascuna de les dues equacions, $\lambda=0$. Notem la importància de què la solució sigui la mateixa a les dues equacions. En cas contrari, el sistema seria incompatible i, per tant, no tendria solució.
3346 \item Substituïm ara aquest valor de $\lambda$ a les equacions de la recta $r$ i en queda%
3347 \begin{equation*}
3348 r:\left\{
3349 \begin{array}{l}
3350 x=-0 \\
3351 y=7+0 \\
3352 z=11+3\text{$\cdot $}0%
3353 \end{array}%
3354 \right. =\left\{
3355 \begin{array}{l}
3356 x=0 \\
3357 y=7 \\
3358 z=11%
3359 \end{array}%
3360 \right.
3361 \end{equation*}%
3362 El punt d'intersecció és, aleshores, $A(0,7,11)$.
3363 \end{itemize}
3364 \end{solution*}
3365 \end{example}
3367 \begin{example}Calculeu el punt d'intersecció entre les rectes $r \colon \left\{ x-2y+z+3=0,\right.$ $\left. 2x-y+z-4=0\right\}$ i $s \colon \left\{ 2x-2y+3z=0, x-y+z-4=0\right\}$.
3369 \begin{solution*}\
3370 \begin{itemize}
3371 \item Passam la recta $r$ a forma paramètrica:%
3372 \begin{equation*}
3373 r:\left\{ x=-\lambda ,y=7+\lambda ,z=11+3\lambda \right\}
3374 \end{equation*}%
3375 \item Substituïm aquestes expressions a les equacions de $s$, i ens queda
3376 el sistema%
3377 \begin{equation*}
3378 \left.
3379 \begin{array}{r}
3380 2(-\lambda )-2(7+\lambda )+3(11+3\lambda )=0 \\
3381 -\lambda -(7+\lambda )+(11+3\lambda )-4=0%
3382 \end{array}%
3383 \right\} ,
3384 \end{equation*}%
3385 la solució del qual és, per a la primera de les equacions, $\lambda
3386 =19/5$, i $\lambda =0$ per a la segona. Per tant, el sistema no té solució, del que deduïm que les dues rectes no tenen punts en comú: o bé es creuen o bé són para\lgem{eles}. Per saber quina de les dues possibilitats és la bona, mirarem si els seus vectors directors són para\lgem{els} o no:%
3387 \begin{equation*}
3388 \overrightarrow{d_{r}}=(-1,1,3), \quad\overrightarrow{d_{s}}=\left\vert
3389 \begin{array}{rrr}
3390 2 & -2 & 3 \\
3391 1 & -1 & 1 \\
3392 \overset{\rightarrow }{i} & \overset{\rightarrow }{j} & \overset{\rightarrow }{k}%
3393 \end{array}%
3394 \right\vert =\left(1,1,0\right)
3395 \end{equation*}%
3396 Aleshores:%
3397 \begin{equation*}
3398 \frac{-1}{1}\neq \frac{1}{1}\neq \frac{3}{0}\Rightarrow \overrightarrow{d_{r}}\text{ i }\overrightarrow{d_{s}}\text{ no són para\lgem{els}}
3399 \end{equation*}%
3400 \item Per tant, com les rectes no s'intersequen i no tenen la mateixa direcció. Llavors es creuen.
3401 \end{itemize}
3402 \end{solution*}
3403 \end{example}
3405 \begin{exercise}Calculeu el punt d'intersecció entre aquestes rectes:
3406 \begin{enumerate}[label=\emph{\alph*})]
3407 \item $r \colon \left\{x+2y + 3z +1 =0, x-y+z =0 \right\}$ i
3409 $s \colon \left\{2x+y+4z +1 =0, x-y+z + 3 =0 \right\}$
3410 \item $r \colon \left\{ x-2y+z+3=0, 2x-y+z-4=0\right\}$ i
3412 $s \colon \left\{ 3x-3y+2z-1=0, x+y-7=0\right\}$
3413 \item $r \colon \frac{x-1}{2} = \frac{y}{3} = \frac{z}{-1}$ i $s \colon \left\{ 2x - 6y + z -1 = 0, x - z +3 = 0\right\}$
3414 \item $r \colon \frac{x}{0} = \frac{y-1}{1} = \frac{z}{-1}$ i $s \colon \frac{x+1}{1} = \frac{y+2}{2} = \frac{y+3}{0}$
3415 \end{enumerate}
3416 \end{exercise}
3418 \subsection{Punt d'intersecció entre una recta i un pla}
3420 De la mateixa manera que per dues rectes, la {\em tàctica} seria expressant la recta en forma paramètrica i substituir al pla.
3422 \begin{example}Calculeu el punt d'intersecció entre la recta%
3423 \begin{equation*}
3424 r \colon \left\{ x-2y+z+3=0, 2x-y+z-4=0\right\}
3425 \end{equation*}%
3426 i el pla $\pi :3x-y+2z-1=0$.
3428 \begin{solution*}\
3429 \begin{itemize}
3430 \item En primer lloc, passarem la recta a forma paramètrica:%
3431 \begin{equation*}
3432 r \colon \left\{
3433 \begin{array}{l}
3434 x=-\lambda \\
3435 y=7+\lambda \\
3436 z=11+3\lambda \\
3437 3x-y+2z-1=0%
3438 \end{array}%
3439 \right.
3440 \end{equation*}%
3442 \item Substituint a les equacions del pla:%
3443 \begin{equation*}
3444 3(-\lambda )-(7+\lambda )+2(11+3\lambda )-1=0 \Rightarrow \quad 2\lambda +14=0 \Rightarrow \quad \lambda =-7
3445 \end{equation*}%
3446 Llavors el punt d'intersecció és
3447 \begin{equation*}
3448 \left.
3449 \begin{array}{l}
3450 x=-(-7)=7 \\
3451 y=7+(-7)=0 \\
3452 z=11+3(-7)=-10%
3453 \end{array}%
3454 \right\} \Rightarrow (7,0,-10)
3455 \end{equation*}
3456 \end{itemize}
3457 \end{solution*}
3458 \end{example}
3460 \begin{exercise}Calculeu el punt d'intersecció entre les rectes i plans següents:
3461 \begin{enumerate}[label=\emph{\alph*})]
3462 \item $r:\frac{x-3}{2}=\frac{y+5}{3}=\frac{z+2}{5}$ i $\pi \colon 3x-y+2z-1=0$
3463 \item $r: \left\{x+2y+3z -1 = 0, y-z -3 = 0\right\}$ i $\pi \colon x-y+2z-1=0$
3464 \item $r: \left\{x+2y+3z -1 = 0, y-z -3 = 0\right\}$ i $\pi \colon x+2y+3z+3=0$
3465 \end{enumerate}
3466 \end{exercise}
3468 \newpage
3469 \section{Exercicis proposats}
3471 \subsection{Vectors}
3473 \begin{exercise}\label{exer:geom:antic-215}Quins dels vectors següents tenen la mateixa direcció?%
3474 \begin{equation*}
3475 \vec{a}\left( 1,-3,2\right) ,\quad \vec{b}\left( 2,0,1\right) ,\quad \vec{c}\left( -2,6,-4\right) ,\quad\vec{d}\left( 5,-15,10\right) ,\quad \vec{e}\left( 10,-30,5\right)
3476 \end{equation*}
3477 \end{exercise}
3479 \begin{exercise}\label{exer:geom:antic-216}Donats els vectors $\vec{u}\left( 1,-3,2\right)$ i $\vec{v}\left(2,0,1\right) $, calculeu:
3480 \begin{enumerate}[label=\emph{\alph*})]
3481 \item $\vec{u}\cdot \vec{v}$
3482 \item $\left\vert \vec{u}\right\vert$ i $\left\vert \vec{v}\right\vert$
3483 \item l'angle que formen entre si $\overrightarrow{u}$ i $\overrightarrow{v}$
3484 \item $\vec{u} \wedge \vec{v}$
3485 \end{enumerate}
3486 \end{exercise}
3488 \begin{exercise}\label{exer:geom:antic-217}Donats $\vec{u}=\vec{i}+m\overrightarrow{j}+\vec{k}$ i $\vec{v}=-2 \overrightarrow{i}+4\overrightarrow{j}+m\overrightarrow{k}$, calculeu $m$
3489 per a què els vectors siguin:
3490 \begin{enumerate}[label=\emph{\alph*})]
3491 \item para\lgem{els}
3492 \item ortogonals
3493 \end{enumerate}
3494 \end{exercise}
3496 \begin{exercise}\label{exer:geom:antic-218}Calculeu l'angle que formen entre si els vectors $\vec{u} = \overrightarrow{\left(1,2,3\right)}$ i $\vec{v} = \overrightarrow{\left( 2,-2,1\right)}$.
3497 \end{exercise}
3499 \begin{exercise}\label{exer:geom:antic-219}Calculeu $m$ per a què el vector $\vec{u}\left( 1,3,m\right)$ sigui ortogonal al vector $\vec{v}\left( 1,-2,3\right)$. És ortonormal?
3500 \end{exercise}
3502 \begin{exercise}\label{exer:geom:antic-220}Calculeu l'àrea del para\lgem{elogram} que formen els vectors $\vec{u}\left( 1,-3,2\right)$ i $\vec{v}\left( 2,0,1\right)$.
3503 \end{exercise}
3505 \begin{exercise}\label{exer:geom:antic-221}Trobeu un vector perpendicular a $\vec{u}\left( 2,3,1\right)$ i a $\vec{v}\left( -1,3,0\right)$ i que sigui unitari.
3506 \end{exercise}
3508 \begin{exercise}\label{exer:geom:antic-222}Trobeu un vector ortogonal a $\vec{u}(1,-1,0)$ i $\vec{v}(2,0,1)$ i el mòdul del qual sigui $\sqrt{24}$.
3509 \end{exercise}
3511 \begin{exercise}\label{exer:geom:antic-223}Calculeu $\left[ \overrightarrow{u},\overrightarrow{v},\overrightarrow{w} \right] $, amb $\vec{u}\left( 1,-1,0\right)$, $\vec{v}\left( 2,0,1\right)$ i $\vec{w}\left( 2,0,-2\right)$.
3512 \end{exercise}
3514 \begin{exercise}\label{exer:geom:antic-224}Calculeu el volum del para\lgem{elepípede} determinat pels vectors $\vec{u}\left( 1,-1,0\right)$ , $\vec{v}\left( 2,0,1\right)$ i $\vec{w}\left( 2,0,-2\right)$.
3515 \end{exercise}
3517 \begin{exercise}\label{exer:geom:antic-225}Calculeu el valor de $m$ perquè $\vec{u}\left( 2,-3,1\right)$, $\vec{v}\left( 1,m,3\right)$ i $\vec{w}\left( -4,5,-1\right)$ siguin coplanaris.
3518 \end{exercise}
3520 \begin{exercise}\label{exer:geom:antic-226}Donat el vector $\vec{v}\left( -2,2,-4\right)$, trobeu les coordenades dels vectors següents:
3521 \begin{enumerate}[label=\emph{\alph*})]
3522 \item unitari i de la mateixa direcció que $\vec{v}$
3523 \item para\lgem{el} a $\vec{v}$ i de mòdul 6.
3524 \end{enumerate}
3525 \end{exercise}
3527 \begin{exercise}\label{exer:geom:antic-227}Trobeu un vector ortogonal a $\vec{u}\left( 2,3,-1\right)$ i a $\vec{v}\left( 1,4,2\right)$ la tercera component del qual sigui 1.
3528 \end{exercise}
3530 \begin{exercise}\label{exer:geom:antic-228}Calculeu les coordenades d'un vector $\vec{u}$ que sigui ortogonal a $\vec{v}\left( 1,2,3\right)$ i $\vec{w}\left( 1,-1,1\right)$ i tal que $\left[ \overrightarrow{u},\overrightarrow{v},\overrightarrow{w}\right] =19$.
3531 \end{exercise}
3533 \subsection{Punts}
3535 \begin{exercise}\label{exer:geom:antic-229}Comproveu si els punts $A\left( 1,-2,1\right)$, $B\left( 2,3,0\right) $ i $C\left( -1,0,-4\right)$ estan alineats o no.
3536 \end{exercise}
3538 \begin{exercise}\label{exer:geom:antic-230}Trobeu el punt simètric del punt $A\left( -2,3,0\right)$ respecte del punt $M\left( 1,-1,2\right)$.
3539 \end{exercise}
3541 \begin{exercise}\label{exer:geom:antic-231}Calculeu $a$ i $b$ per a què els punts $A\left( 1,2,-1\right)$, $B\left( 3,0,-2\right)$ i $C\left( 4,a,b\right)$ estiguin alineats.
3542 \end{exercise}
3544 \subsection{Rectes i plans}
3546 \begin{exercise}\label{exer:geom:antic-232}Associeu els conceptes de punt, vector, recta i pla al pla o a l'espai amb qualcuna o qualcunes de les expresions següents:%
3547 \begin{multicols}{2}
3548 \begin{enumerate}[label=\emph{\alph*})]
3549 \item $\overrightarrow{A}\left( 2,-3,1\right)$
3551 \item $\left\{
3552 \begin{array}{c}
3553 x+y=2 \\
3554 y+z=3%
3555 \end{array}%
3556 \right.$
3558 \item $\left\{
3559 \begin{array}{c}
3560 x=-2\lambda \\
3561 y=2+\lambda \\
3562 z=3%
3563 \end{array}%
3564 \right.$
3566 \item $A\left( 2,-3,1\right)$
3568 \item $x+y=2$
3570 \item $\left\{
3571 \begin{array}{c}
3572 x=-2\lambda +\mu \\
3573 y=2+\lambda \\
3574 z=3%
3575 \end{array}%
3576 \right.$
3578 \item $\left\{
3579 \begin{array}{c}
3580 x=2 \\
3581 y=3%
3582 \end{array}%
3583 \right.$
3585 \item $\frac{x-1}{0}=y+3=\frac{z}{-6}$
3587 \end{enumerate}
3588 \end{multicols}
3589 \end{exercise}
3591 \begin{exercise}\label{exer:geom:antic-233}Escriviu l'equació de la recta $r$ que passa pels punts $A\left( -3, 2, 1\right)$ i $B\left( -\tfrac{5}{2},\tfrac{3}{2},0\right)$. Podeu trobar les altres equacions de la recta?
3592 \end{exercise}
3594 \begin{exercise}\label{exer:geom:antic-234}Trobeu l'equació de la recta que passa pel punt $A\left( -4, 2, 5\right)$ i és para\lgem{ela} al vector director $\rightarrow{(1,0,-1)}$.
3595 \end{exercise}
3597 \begin{exercise}\label{exer:geom:antic-234b}Trobeu l'equació de la recta que passa pel punt $A\left( -4,2,5\right)$ i és paraç\lgem{ela} a l'eix $OZ$.
3598 \end{exercise}
3600 \begin{exercise}\label{exer:geom:antic-235}Comproveu si els punts $A\left( 1,-2,1\right)$, $B\left( 2,3,0\right)$, $C\left( -1,0,-4\right)$ i $D\left( 4,0,-5\right)$ es troben en un mateix pla o no. En cas d'estar en el mateix pla, digueu quin és.
3601 \end{exercise}
3603 \begin{exercise}Trobeu les equacions paramètrica i contínua la recta%
3604 \begin{equation*}
3605 \left.
3606 \begin{array}{rrr}
3607 -x+3y-z+10 & = & 0 \\
3608 2x+y-z-6 & = & 0%
3609 \end{array}%
3610 \right\}
3611 \end{equation*}
3612 \end{exercise}
3614 \subsection{Plans}
3616 \begin{exercise}\label{exer:geom:antic-239}Calculeu les equacions paramètrica i general dels plans següents:
3617 \begin{enumerate}[label=\emph{\alph*})]
3618 \item passa pel punt $P\left( 2,-3,1\right)$ i el vector normal del qual és $\vec{n}\left( 5,-3,-4\right)$
3619 \item perpendicular a la recta $\frac{x}{2}=\frac{y+1}{-1}=\frac{z}{3}$ i que passa pel punt $\left( 1,0,1\right)$
3620 \end{enumerate}
3621 \end{exercise}
3623 \begin{exercise}\label{exer:geom:mat-especiales-1}Trobeu l'equació implícita dels plans següents:
3624 \begin{itemize}
3625 \item Pla que passa pels punts $P_1 = (1, 0, -1)$, $P_2 = (1, 3, 0)$ i $P_3 = (2, -1, 3)$
3626 \item Pla que passa pel punt $Q=(3,0,1)$ i és para\lgem{el} al pla $3x-2y+5z +1=0$
3627 \end{itemize}
3628 \end{exercise}
3630 \begin{exercise}\label{exer:geom:antic-241}Determineu l'equació del pla que conté el punt $P\left( 2,1,2\right)$ i la recta $x-2=\frac{y-3}{-1}=\frac{z-4}{-3}$.
3631 \end{exercise}
3633 \begin{exercise}\label{exer:geom:antic-242}Comproveu que les rectes $r:\frac{x-1}{2}=y=z-2$ i%
3634 \begin{equation*}
3635 s:\left\{
3636 \begin{array}{r}
3637 x-2z=5 \\
3638 x-2y=11%
3639 \end{array}%
3640 \right.
3641 \end{equation*}%
3642 són para\lgem{eles}, i troba l'equació del pla que les conté.
3643 \end{exercise}
3645 \begin{exercise}\label{exer:geom:antic-243}Determineu el valor de $a$ per a què les rectes $r$ i $s$ siguin coplanàries:%
3646 \begin{equation*}
3647 r:x=y-a=\frac{z}{0},\text{ }s:\left\{
3648 \begin{array}{l}
3649 x=1+\lambda \\
3650 y=1-\lambda \\
3651 z=-1+\lambda%
3652 \end{array}%
3653 \right.
3654 \end{equation*}%
3655 Trobeu l'equació del pla que les conté.
3656 \end{exercise}
3658 \begin{exercise}\label{exer:geom:antic-245}Trobeu l'equació del pla que passa pels punts $A\left( 1,3,2\right) $ i $B\left( -2,5,0\right) $ i és para\lgem{el} a la recta%
3659 \begin{equation*}
3660 r:\left\{ x=3-\lambda ,\text{ }y=2+\lambda ,\text{ }z=-2-3\lambda \right\}
3661 \end{equation*}
3662 \end{exercise}
3664 \begin{exercise}\label{exer:geom:antic-246}Trobeu l'equació del pla que conté la recta%
3665 \begin{equation*}
3666 r:\left\{ x=2+3\lambda ,\text{ }y=-1-\lambda ,\text{ }z=\lambda \right\}
3667 \end{equation*}%
3668 i és para\lgem{el} a $s:\frac{x-3}{5}=\frac{y+1}{2}=\frac{z}{-3}$.
3669 \end{exercise}
3671 \begin{exercise}\label{exer:geom:antic-247}Calculeu el valor de $m$ per a què els punts $A\left( m,0,1\right)$, $B\left( 0,1,2\right)$, $C\left( 1,2,3\right)$ i $D\left( 7,2,1\right)$
3672 estiguin en un mateix pla. Quina és l'equaci\'{o} d'aquest pla?
3673 \end{exercise}
3675 \begin{exercise}\label{exer:geom:antic-248}Donat el pla $\pi \colon 2x-3y+z=0$ i la recta $r \colon x-1=\frac{y-2}{-1}=\frac{z+1}{2}$, trobeu l'equació del pla que conté la recta $r$ i és perpendicular al pla $\pi$.
3676 \end{exercise}
3678 \begin{exercise}\label{exer:geom:antic-249}Escriviu l'equació del pla que passa pels punts $A\left( 1,-3,2\right)$ i $B\left( 0,1,1\right) $ i és para\lgem{el} a la recta%
3679 \begin{equation*}
3680 r:\left\{
3681 \begin{array}{r}
3682 3x-2y+1=0 \\
3683 2y+3z-3=0%
3684 \end{array}%
3685 \right.
3686 \end{equation*}
3687 \end{exercise}
3689 \begin{exercise}\label{exer-js-geometria-1} Trobeu la recta que passa pel punt $A=(1,1,-1)$, és para\lgem{ela} al pla $\pi \equiv x - y + z = 5$ i talla a l'eix de coordenades $OZ$.
3690 \end{exercise}
3692 \begin{exercise}\label{exer-js-geometria-3} Donats el punt $P=(2,1,2)$ i la recta resultant de la intersecció dels plans $4x-y = 12$ i $z - x = 2$, trobeu l'àrea del triangle determinat pel punt $P$, el punt de la recta més proper a $P$ i el punt $Q=(1,0,-1)$.
3693 \end{exercise}
3695 \begin{exercise}\label{exer-js-geometria-4} Donada la recta de l'equació $\frac{x}{2} = 1-y = \frac{2z+2}{6}$ i el pla $\pi$ d'equació $x+3y-3z = -3$, trobeu:
3697 \begin{enumerate}[label=\emph{\alph*})]
3698 \item El pla que conté a $r$ i és perpendicular a $\pi$
3699 \item El volum del tetraedre determinat per $\pi$ i els plans coordenats
3700 \end{enumerate}
3702 \end{exercise}
3704 \begin{exercise}Sigui $\pi$ el pla d'equació $6x+4y-2z = 2$ i $r$ la recta d'equació $\left\{
3705 \begin{array}{l l}
3706 3x+2y & = 0 \\
3707 -y+z & = -1%
3708 \end{array}%
3709 \right.$. Estudieu si els punts $P=(1,0,0)$, $Q=(2,-3,-4)$, $R=(0,1,1)$ i $S=(0,0,-1)$ pertanyen al pla $\pi$ o a la recta $r$.
3710 \end{exercise}
3712 \begin{exercise}Trobeu el punt d'intersecció de $r \colon \frac{x-1}{2} = \frac{y}{3} = \frac{z+1}{0}$ i $\pi \colon 3x - y + z = 5$. Trobeu el pla que passa pel punt anterior i és perpendicular a la recta
3713 \begin{equation*}
3714 s \colon \left\{
3715 \begin{array}{r}
3716 2x+y-3z=1 \\
3717 x+y-2z=0%
3718 \end{array}%
3719 \right.
3720 \end{equation*}%
3721 \end{exercise}
3723 \begin{exercise}Trobeu el pla $\pi$ que és perpendicular a la recta
3724 \begin{equation*}
3725 r \colon \left\{
3726 \begin{array}{r}
3727 x-2y+z=1 \\
3728 -2x+y-z=3%
3729 \end{array}%
3730 \right.
3731 \end{equation*}%
3732 Trobeu el pla para\lgem{el} a $\pi$ que passa pel punt $(-1, 0, 10)$.
3733 \end{exercise}
3735 \begin{exercise}Trobeu la intersecció dels plans $\pi_1 \colon 2x -y + z + 1 = 0$ i $\pi_2 \colon x + 5y - z = 5$. En cas que la intersecció sigui una recta, expresseu-la en forma contínua i trobeu dos punts de la recta.
3736 \end{exercise}
3738 \begin{exercise}Trobeu el punt d'intersecció $A$ de les rectes $r \colon \frac{x-1}{-1} = \frac{y}{0} = \frac{z+2}{-1}$ i
3739 \begin{equation*}
3740 s \colon \left\{
3741 \begin{array}{r}
3742 x+z=1 \\
3743 -y+z=-1%
3744 \end{array}%
3745 \right.
3746 \end{equation*}%
3747 Trobeu el pla que conté $r$ i $s$ i passa per $A$.
3748 \end{exercise}
3751 \subsection{Posicions relatives}
3753 \begin{exercise}\label{exer:geom:antic-236}Estudieu la posició relativa de les rectes següents, i trobeu el seu punt de tall quan sigui possible:
3754 \begin{enumerate}[label=\emph{\alph*})]
3755 \item $r \colon \frac{x-1}{3}=\frac{y+2}{2}=\frac{z-1}{4}$ i $s \colon \frac{x+2}{-1}=\frac{y-3}{2}=\frac{z-2}{3}$
3756 \item $r \colon \frac{x-1}{-1}=\frac{y-3}{2}=\frac{z-2}{1}$ i $s \colon \frac{x-4}{4}=\frac{y-4}{1}=\frac{z-5}{2}$
3757 \item $r \colon \frac{x}{2}=y-1=\frac{z+1}{3}$ i $s \colon \left\{ x-2y-1=0, 3y-z+1=0\right\}$
3758 \item $r \colon \frac{x-1}{2}=\frac{y}{3}=\frac{z}{4}$ i $s \colon \left\{x=3+4\lambda , y=3+6\lambda , z=4+8\lambda \right\}$
3759 \end{enumerate}
3760 \end{exercise}
3762 \begin{exercise}Trobeu les posicions relatives d'aquests plans. En cas de que es tallin, trobeu l'equació contínua de la recta corresponent:
3763 \begin{enumerate}[label=\emph{\alph*})]
3764 \item $\pi_1 \colon 3x + 3y + z -1=0$ i $\pi_2 \colon x -5y + 5z =0$
3765 \item $\pi_1 \colon x + 2y + z =0$ i $\pi_2 \colon x + 2y + z -2 = 0$
3766 \item $\pi_1 \colon 2x + 4y + z =0$ i $\pi_2 \colon 10x + 20y + 5z +2= 0$
3767 \item $\pi_1 \colon 2x + 8z - 10=0$ i $\pi_2 \colon 2x +8y - 10 = 0$
3768 \end{enumerate}
3769 \end{exercise}
3771 \begin{exercise}\label{exer:geom:antic-244}Estudieu la posició relativa de la recta $r:\frac{x-3}{2}=y+1=\frac{z}{-1}$ i el pla $\pi :x-y+z-3=0$.
3772 \end{exercise}
3774 \begin{exercise}\label{exer:geom:antic-250}Estudieu la posició relativa de la recta $r:\left\{ x=3, y=2\right\}$ i el pla $z=1$.
3775 \end{exercise}
3777 \begin{exercise}Digueu si la recta $r$ està continguda al pla $\pi$:
3778 \begin{enumerate}[label=\emph{\alph*})]
3779 \item $r \colon \frac{x}{6}=\frac{y}{2}=\frac{z}{4}$ i $\pi \colon -x + y + 3z = 0$
3780 \item $r \colon \left\{ x-y+z + 1 = 0, 2x -z = -1 \right\}$ i $\pi \colon -2x + z = 0$
3781 \item $r \colon \frac{x-1}{-1}=y+1=\frac{z+1}{0}$ i $\pi \colon -x + y + 5=0$
3782 \end{enumerate}
3783 \end{exercise}
3785 \begin{exercise}Digueu si la recta $r$ talla al pla $\pi$:
3786 \begin{enumerate}[label=\emph{\alph*})]
3787 \item $r \colon \frac{x-1}{2}=\frac{y}{3}=\frac{z}{0}$ i $\pi \colon -x + 2y - 3z + 5 = 0$
3788 \item $r \colon \frac{x}{2}=\frac{y+1}{1}=\frac{z+1}{0}$ i $\pi \colon -x +2y +2 = 0$
3789 \item $s \colon \left\{
3790 \begin{array}{r}
3791 -2x +y =3 \\
3792 -y + 2z=2%
3793 \end{array}%
3794 \right.$ i $\pi \colon \left\{
3795 \begin{array}{ll}
3796 x & = -1 + 2\lambda - \mu \\
3797 y & - \lambda + \mu \\
3798 z & 1 - \mu%
3799 \end{array}%
3800 \right.$
3801 \end{enumerate}
3803 En cas afirmatiu, trobeu el punt de tall.
3804 \end{exercise}
3806 \begin{exercise}Donat el pla $\pi \colon x - 2y = 5$, trobeu una recta para\lgem{ela}, una recta secant i una recta perpendicular a $\pi$.
3808 En cada cas, trobeu una recta para\lgem{ela} que passi pel punt $A(-10, 0, 10)$.
3809 \end{exercise}
3811 \begin{exercise}\label{exer:geom:antic-237}Calculeu el valor de $a$ per a què les rectes $r$ i $s$ es tallin, i trobeu el seu punt de tall: $r \colon x=y=z-a$, $s \colon \frac{2x-1}{3}=\frac{y+3}{-2}=\frac{z-2}{0}$.
3812 \end{exercise}
3814 \begin{exercise}\label{exer:geom:antic-238}Calculeu els valors de $m$ i $n$ per a què les rectes $r$ i $s$ siguin para\lgem{eles}: $r \colon \left\{
3815 \begin{array}{l}
3816 x=5+4\lambda \\
3817 y=3+\lambda \\
3818 z=-\lambda%
3819 \end{array}%
3820 \right.$, $s \colon \frac{x}{m}=\frac{y-1}{3}=\frac{z+3}{n}$.
3821 \end{exercise}
3823 \begin{exercise}\label{exer:geom:antic-240}Calculeu $m$ i $n$ per a què els plans $\alpha \colon mx+y-3z-1=0$ i $\beta \colon 2x+ny-z-3=0$ siguin para\lgem{els}. Poden ser coincidents?
3824 \end{exercise}
3826 \begin{exercise}\label{exer:geom:antic-251}Estudieu les posicions relatives del pla $\pi \colon x+ay-z=1$ i la recta%
3827 \begin{equation*}
3828 r:\left\{
3829 \begin{array}{r}
3830 2x+y-az=2 \\
3831 x-y-z=a-1%
3832 \end{array}%
3833 \right.
3834 \end{equation*}%
3835 segons els valors de $a$.
3836 \end{exercise}
3838 \begin{exercise}\label{exer-js-geometria-2} Estudieu la posició relativa dels plans $\pi_1 \equiv x + 3y - 2z = 7$, $\pi_2 \equiv x+2y-az=5$ i $\pi_3 \equiv ax+z = b$, segons els valors d'$a$ i de $b$. Quan es tallen en una recta? N'hi ha alguna d'elles que passi pel punt $(-1,4,2)$?
3839 \end{exercise}
3841 \begin{exercise}Trobeu en funció de $a$, la posició relativa de la recta $r \colon \frac{x}{3} = \frac{y+2}{-1} = \frac{z-1}{0}$ i la recta $s$ determinada pels punts $A(a, -2, 1)$ i $B(1, 0, 1)$.
3842 \end{exercise}
3844 \section{Exercicis resolts de geometria de l'espai}
3846 \begin{example}Donada la recta següent%
3847 \begin{equation*}
3848 r:\left\{
3849 \begin{array}{r}
3850 x-2y+z+3=0 \\
3851 2x-y+z-4=0,%
3852 \end{array}%
3853 \right.
3854 \end{equation*}%
3855 trobeu un punt qualsevol de la recta i calculeu el seu vector director.
3857 \begin{solution*}
3858 Qualsevol punt de la recta ha de complir l'equació que la defineix. Per a trobar-ne un, donem un valor qualsevol a les variables. Per exemple, podem prendre $y=0$ i substituir aquest valor en l'equació de la recta:%
3859 \begin{equation*}
3860 \left.
3861 \begin{array}{r}
3862 x-2\text{$\cdot $}0+z+3=0 \\
3863 2x-0+z-4=0%
3864 \end{array}%
3865 \right\} ;\quad \left.
3866 \begin{array}{r}
3867 x+z+3=0 \\
3868 2x+z-4=0%
3869 \end{array}%
3870 \right\}
3871 \end{equation*}%
3872 Les solucions d'aquest sistema són $x=7$, $z=-10$, per la qual cosa el punt que cercam és
3873 \begin{equation*}
3874 \left( 7,0,-10\right)
3875 \end{equation*}%
3876 El vector director de la recta és%
3877 \begin{equation*}
3878 \begin{split}
3879 \overrightarrow{d} &= \overrightarrow{n_{1}}\wedge \overrightarrow{n_{2}}= \overrightarrow{(1,-2,1)} \wedge \overrightarrow{(2,-1,1)} \\
3880 &= \left\vert
3881 \begin{array}{rrr}
3882 1 & -2 & 1 \\
3883 2 & -1 & 1 \\
3884 \overset{\rightarrow }{i} & \overset{\rightarrow }{j} & \overset{\rightarrow}{k}
3885 \end{array}%
3886 \right\vert \\
3887 &= \overrightarrow{(-1,1,3)}
3888 \end{split}
3889 \end{equation*}
3890 \end{solution*}
3891 \end{example}
3894 \begin{example}Trobeu un punt qualsevol del pla $\pi \colon-x+5y+2z-1=0$ i un vector perpendicular a ell.
3895 \begin{solution*}
3896 Un punt qualsevol del pla ha de satisfer la seva equació. Per tant, donem valors qualssevol a les variables, per exemple $y=0$ i $z=0$, i substituim aquests valors a l'equació del pla:%
3897 \begin{equation*}
3898 -x+5\cdot 0+2\cdot 0-1=0; -x-1=0; x=-1
3899 \end{equation*}%
3900 Per tant, un punt del pla és%
3901 \begin{equation*}
3902 \left( -1,0,0\right)
3903 \end{equation*}%
3904 Un vector perpendicular al pla és el seu vector normal: $\overrightarrow{n}=\left(-1,5,2\right)$.
3905 \end{solution*}
3906 \end{example}
3909 \begin{example}Passeu a forma paramètrica la recta
3910 \begin{equation*}
3911 r:\left\{ \begin{array}{r}
3912 x-2y+z+3=0 \\
3913 2x-y+z-4=0%
3914 \end{array}%
3915 \right.
3916 \end{equation*}
3917 \begin{solution*}Per poder escriure una recta en forma paramètrica necessitem un vector director d'ella i un punt qualsevol del seus punts.
3919 Calculem el vector director de la recta:%
3920 \begin{equation*}
3921 \begin{split}
3922 \overrightarrow{d} & =\overrightarrow{n_{1}}\wedge \overrightarrow{n_{2}} = \overrightarrow{(1,-2,1)} \wedge \overrightarrow{(2,-1,1)} = \\
3923 & =\left\vert \begin{array}{rrr}
3924 1 & -2 & 1 \\
3925 2 & -1 & 1 \\
3926 \overset{\rightarrow }{i} & \overset{\rightarrow }{j} & \overset{\rightarrow}{k}
3927 \end{array}%
3928 \right\vert = \overrightarrow{(-1,1,3)}
3929 \end{split}
3930 \end{equation*}%
3931 Cerquem ara un punt de la recta. Aquest punt ha de complir les equacions $x-2y+z+3=0$ i $2x-y+z-4=0.$ Facem, per exemple, $x=0$. Aleshores, substituint aquest valor a les dues equacions anteriors queda el sistema d'equacions%
3932 \begin{equation*}
3933 \left.
3934 \begin{array}{r}
3935 -2y+z=-3 \\
3936 -y+z=4%
3937 \end{array}%
3938 \right\} ,
3939 \end{equation*}%
3940 la solució del qual és $y=7,z=11$. Així, un punt de la recta és $A(0,7,11)$.
3942 L'equació en forma paramètrica és, aleshores, %
3943 \begin{equation*}
3944 r:\left\{
3945 \begin{array}{l}
3946 x=0+(-1)\lambda \\
3947 y=7+1\lambda \\
3948 z=11+3\lambda%
3949 \end{array}%
3950 \right. =\left\{
3951 \begin{array}{l}
3952 x=-\lambda \\
3953 y=7+\lambda \\
3954 z=11+3\lambda%
3955 \end{array}%
3956 \right.
3957 \end{equation*}
3958 \end{solution*}
3959 \end{example}
3961 \newpage
3963 \section{Solucions}
3965 \begin{itemize}
3967 \item[\ref{exer:geom:antic-215}] $\vec{a}$, $\vec{c}$ i $\vec{d}$ són para\lgem{els}.
3969 \item[\ref{exer:geom:antic-216}] \begin{enumerate*}[label=\emph{\alph*})] \item $4$, \item El mòdul de $\vec{a}$ és $\sqrt{14}$ i el mòdul de $\vec{b}$ és $\sqrt{5}$, \item l'angle que formen és, aproximadament, de $61,43\degree$ \end{enumerate*}
3971 \item[\ref{exer:geom:antic-217}] \begin{enumerate*}[label=\emph{\alph*})] \item $m = -2$, \item $m=\frac{2}{5}$\end{enumerate*}
3973 \item[\ref{exer:geom:antic-218}] $\cos^{-1} \frac{1}{\sqrt{280}} \sim 86,57\degree$
3975 \item[\ref{exer:geom:antic-219}] $m = \frac{5}{3}$
3977 \item[\ref{exer:geom:antic-220}] $\sqrt{54}$
3979 \item[\ref{exer:geom:antic-221}] $\frac{1}{\sqrt{91}} \overrightarrow{(-3, -1, 9)}$
3981 \item[\ref{exer:geom:antic-222}] $\overrightarrow{(-2,-2,4)}$ o $\overrightarrow{(2,2,-4)}$
3983 \item[\ref{exer:geom:antic-223}] $\left[ \overrightarrow{u},\overrightarrow{v},\overrightarrow{w}\right] = -6$
3985 \item[\ref{exer:geom:antic-224}] $\left\vert \left[ \overrightarrow{u},\overrightarrow{v},\overrightarrow{w}\right] \right\vert = \lvert -6 \rvert = 6$
3987 \item[\ref{exer:geom:antic-225}] $m = -4$
3989 \item[\ref{exer:geom:antic-226}] \begin{enumerate*}[label=\emph{\alph*})] \item $1/\sqrt{24} \overrightarrow{(-2,2,-4)}$, \item $\overrightarrow{\left(-12/\sqrt{24}, 12/\sqrt{24}, -24/\sqrt{24}\right)}$ \end{enumerate*}
3991 \item[\ref{exer:geom:antic-227}] $\overrightarrow{(2,-1,1)}$
3993 \item[\ref{exer:geom:antic-228}] $\overrightarrow{(95/30, 57/45, -57/30)}$
3995 \item[\ref{exer:geom:antic-229}] No estan alineats
3997 \item[\ref{exer:geom:antic-230}] El punt és $A' = (4, -5, 4)$
3999 \item[\ref{exer:geom:antic-231}] $a = -1$ i $b= \frac{-5}{2}$
4001 \item[\ref{exer:geom:antic-232}] \begin{enumerate*}[label=\emph{\alph*})] \item vector \item punt al pla (és una intersecció de dues rectes) i recta a l'espai (és la intersecció de dos plans a l'espai) \item recta en forma paramètrica a l'espai \item punt a l'espai \item recta al pla i pla a l'espai \item pla a l'espai en forma paramètrica \item punt al pla \item recta a l'espai en forma contínua. \end{enumerate*}
4003 \item[\ref{exer:geom:antic-233}] \begin{enumerate*}[label=\emph{\alph*})] \item Forma vectorial $r \equiv (-3,2,1) + \lambda \overrightarrow{(-2,1,-1)}$, \item Forma paramètrica $r \equiv \left\{
4004 \begin{array}{l}
4005 x=-3-2\lambda \\
4006 y=2+\lambda \\
4007 z=1-\lambda%
4008 \end{array}%
4009 \right.$ \item Forma contínua $r \equiv \frac{x+3}{-2} = \frac{y-2}{1} = \frac{z-1}{-1}$, \item Forma implícita $r \equiv \left\{
4010 \begin{array}{l}
4011 x+2y-1 = 0 \\
4012 -y -z + 3 = 0%
4013 \end{array}%
4014 \right.$ \end{enumerate*}
4016 \item[\ref{exer:geom:antic-234}] \begin{enumerate*}[label=\emph{\alph*})] \item Forma vectorial $r \equiv (-4,2,5) + \lambda \overrightarrow{(0,0,1)}$, \item Forma paramètrica $r \equiv \left\{
4017 \begin{array}{l}
4018 x=-4 \\
4019 y=2 \\
4020 z=5+\lambda%
4021 \end{array}%
4022 \right.$ \item Forma contínua $r \equiv \frac{x+4}{0} = \frac{y-2}{0} = \frac{z-5}{1}$, \item Forma implícita $r \equiv \left\{
4023 \begin{array}{l}
4024 x+4 = 0 \\
4025 y-2 = 0%
4026 \end{array}%
4027 \right.$ \end{enumerate*}
4029 \item[\ref{exer:geom:antic-235}] No estan en el mateix pla.
4031 \item[\ref{exer:geom:antic-236}] \begin{enumerate*}[label=\emph{\alph*})] \item S'encreuen, \item s'encreuen, \item s'encreuen, \item coincidents \end{enumerate*}
4033 \item[\ref{exer:geom:antic-237}] Es tallen
4035 \item[\ref{exer:geom:antic-238}] $m=12$ i $n = -3$
4037 \item[\ref{exer:geom:antic-239}] \begin{enumerate*}[label=\emph{\alph*})] \item $\pi \colon 5x -3y +4z -23 = 0$, \item $2x -y +3z +1 = 0$ \end{enumerate*}
4039 \item[\ref{exer:geom:mat-especiales-1}]\begin{enumerate*}[label=\emph{\alph*})] \item $\pi \equiv 13x -y -3z -16 = 0$, \item $\pi \equiv 3x -2y +5z -14 =0$\end{enumerate*}
4041 \item[\ref{exer:geom:antic-240}] Per ser para\lgem{els} $m = 6$ i $n = \frac{1}{3}$. No són coincidents.
4043 \item[\ref{exer:geom:antic-241}] $\pi \colon 2x -y +z -5 =0$.
4045 \item[\ref{exer:geom:antic-242}] L'equació del pla que les conté és $2x +16y-20z +38 = 0$.
4047 \item[\ref{exer:geom:antic-243}] $a=-2$; el pla que les conté és $\pi \colon x-y-2z -2 = 0$.
4049 \item[\ref{exer:geom:antic-244}] $r$ és para\lgem{ela} a $\pi$.
4052 \item[\ref{exer:geom:antic-245}] $4x +7y +z -27 =0$
4054 \item[\ref{exer:geom:antic-246}] $x+14y+11z +12 = 0$
4056 \item[\ref{exer:geom:antic-247}] $m=-1$; $\pi \colon -x +4y -3z +2=0$
4058 \item[\ref{exer:geom:antic-248}] $-5x -3y +z +12 = 0$
4060 \item[\ref{exer:geom:antic-249}] $11x -4y +5z -1 = 0$
4062 \item[\ref{exer:geom:antic-250}] Secants.
4064 \item[\ref{exer:geom:antic-251}] Si $a \neq -1$ o $a \neq 2$, llavors són secants. Si $a=-1$, llavors són para\lgem{els}. Si $a=2$, llavors $r$ està continguda a $\pi$.
4066 \end{itemize}